Medicine KS 2

Download as pdf or txt
Download as pdf or txt
You are on page 1of 204

Medicine II

Medicine Marking Scheme →


https://drive.google.com/file/d/1THMyD_hWmjdxYzDAUCrnWmI7EGaqFd3S/view?us
p=sharing

Answers are from the following books - - Archith Boloor : Exam Preparatory Manual
for Undergraduates - Manthappa : Manipal Prep Manual of Medicine - Matthew :
Medicine Prep Manual for Undergraduates.

Hematology and Oncology


SHOCK / DIC

NOT SURE IF THIS IS A PART OF PAPER I OR II


A 50 year old male admitted in ICU for fever with shaking chills develops
hypotension and starts bleeding from venipuncture sites. Blood culture is
positive for gram negative bacilli. a. What is the most likely diagnosis? b.
Discuss the clinical and laboratory evaluation of this patient. c. Outline the
treatment plan for the same patient.
Septic Shock complicated by Disseminated Intravascular Coagulation (DIC)
Clinical Features
Spectrum of thrombosis and hemorrhage is seen in all cases of DIC; either of the two
or both may be encountered. Most cases present with hemorrhage.
In sepsis-induced DIC, bleeding manifestations are more common than thrombosis
Epistaxis, gingival bleeding
Purpura fulminans is characteristic of sepsis-induced DIC.
Waterhouse-Friedrichsen syndrome - an occult thrombosis of adrenal vein resulting
in adrenal hemorrhage
Intracranial hemorrhage may be seen.
Microangiopathic hemolytic anemia
ARDS
Laboratory Evaluation
ESR → Reduced (Due to reduced fibrinogen)
Platelets → Reduced
PT and aPTT → Increased (could be normal in 40% cases)
Plasma fibrinogen level → Reduced (relates most closely with bleeding)
Fibrin degradation products (FDPs) → Increased
D-dimer → Increased
Schistocytes seen on peripheral smear
Management consists of the following
Management of sepsis
Initial resuscitation → Volume resuscitation with normal saline 30 mL/kg over first 30
min → if mean arterial pressure > 70 mmHg start inotropes.
Diagnosis
Blood culture → Before the initiation of antimicrobial therapy, at least two blood
cultures should be obtained, one drawn percutaneously and other drawn through
each vascular access device if inserted longer than 48 hours
Antibiotic therapy → Start IV antibiotic therapy within first hour of recognition of
sepsis after obtaining appropriate cultures.
Source control → Evaluate patients for source of infection amenable to source
control measures
Vasopressors → To be initiated if fluid challenge fails to restore adequate blood
pressure and organ perfusion
Inotropes → In patients with low cardiac output despite fluid resuscitation,
dobutamine can be used to increase cardiac output.
Mechanic ventilation → In sepsis induced lung injury high tidal volumes (> 6 mL/kg
body weight) have to be avoided. A minimum amount of PEEP should be set to
prevent lung collapse at end-expiration.
Management of DIC
Replacement of coagulation factors and platelets by fresh whole blood, fresh frozen
plasma, cryoprecipitate and platelet concentrates.
Dose
Cryoprecipitate → 1 unit/10kg
Fresh frozen plasma → 15-20 mL/kg
Platelet concentrates → 1-2 units/10kg
Target
Fibrinogen → 100-150 mg/dL
Platelets → >50,000/cu.mm.
Hemoglobin → 6-10 g/dL
PT and aPTT of less than 1.5 times the control
Inhibition of coagulation process
Role of heparin is controversial. However, it is often given in patients with thrombotic
manifestations. Average requirement is 1000 units/hour by continuous IV infusion. It
should be given after correction of bleeding risk.
Anti-fibrinolytic agents are contraindicated as they may cause enhanced precipitation
of fibrin in microcirculation and macrocirculation leading to fatal disseminated
thrombosis.
Diagnosis and treatment of disseminated intravascular coagulation
Clinical Features
Spectrum of thrombosis and hemorrhage is seen in all cases of DIC; either of the two
or both may be encountered. Most cases present with hemorrhage.
In sepsis-induced DIC, bleeding manifestations are more common than thrombosis
Epistaxis, gingival bleeding
Purpura fulminans is characteristic of sepsis-induced DIC.
Waterhouse-Friedrichsen syndrome - an occult thrombosis of adrenal vein resulting
in adrenal hemorrhage
Intracranial hemorrhage may be seen.
Microangiopathic hemolytic anemia
ARDS
Laboratory Evaluation
ESR → Reduced (Due to reduced fibrinogen)
Platelets → Reduced
PT and aPTT → Increased (could be normal in 40% cases)
Plasma fibrinogen level → Reduced (relates most closely with bleeding)
Fibrin degradation products (FDPs) → Increased
D-dimer → Increased
Schistocytes seen on peripheral smear
Management of DIC
Replacement of coagulation factors and platelets by fresh whole blood, fresh frozen
plasma, cryoprecipitate and platelet concentrates.
Dose
Cryoprecipitate → 1 unit/10kg
Fresh frozen plasma → 15-20 mL/kg
Platelet concentrates → 1-2 units/10kg
Target
Fibrinogen → 100-150 mg/dL
Platelets → >50,000/cu.mm.
Hemoglobin → 6-10 g/dL
PT and aPTT of less than 1.5 times the control
Inhibition of coagulation process
Role of heparin is controversial. However, it is often given in patients with thrombotic
manifestations. Average requirement is 1000 units/hour by continuous IV infusion. It
should be given after correction of bleeding risk.
Anti-fibrinolytic agents are contraindicated as they may cause enhanced precipitation
of fibrin in microcirculation and macrocirculation leading to fatal disseminated
thrombosis.
Disseminated intravascular coagulation
Disseminated intravascular coagulation (DIC) is a widespread acute or chronic
thrombohemorrhagic disorder in which a combination of thrombosis and
hemorrhage develops as a secondary complication of a wide variety of disorders
Causes
Clinical Features
Spectrum of thrombosis and hemorrhage is seen in all cases of DIC; either of the two
or both may be encountered. Most cases present with hemorrhage.
In sepsis-induced DIC, bleeding manifestations are more common than thrombosis
Epistaxis, gingival bleeding
Purpura fulminans is characteristic of sepsis-induced DIC.
Waterhouse-Friedrichsen syndrome - an occult thrombosis of adrenal vein resulting
in adrenal hemorrhage
Intracranial hemorrhage may be seen.
Microangiopathic hemolytic anemia
ARDS
Laboratory Evaluation
ESR → Reduced (Due to reduced fibrinogen)
Platelets → Reduced
PT and aPTT → Increased (could be normal in 40% cases)
Plasma fibrinogen level → Reduced (relates most closely with bleeding)
Fibrin degradation products (FDPs) → Increased
D-dimer → Increased
Schistocytes seen on peripheral smear
Management of DIC
Replacement of coagulation factors and platelets by fresh whole blood, fresh frozen
plasma, cryoprecipitate and platelet concentrates.
Dose
Cryoprecipitate → 1 unit/10kg
Fresh frozen plasma → 15-20 mL/kg
Platelet concentrates → 1-2 units/10kg
Target
Fibrinogen → 100-150 mg/dL
Platelets → >50,000/cu.mm.
Hemoglobin → 6-10 g/dL
PT and aPTT of less than 1.5 times the control
Inhibition of coagulation process
Role of heparin is controversial. However, it is often given in patients with thrombotic
manifestations. Average requirement is 1000 units/hour by continuous IV infusion. It
should be given after correction of bleeding risk.
Anti-fibrinolytic agents are contraindicated as they may cause enhanced precipitation
of fibrin in microcirculation and macrocirculation leading to fatal disseminated
thrombosis.
ANEMIA
Classify causes of anemia in adults. How will you investigate and interpret the
reports in an elderly patient with severe anemia?
Causes of anemia

Investigations and Interpretations


Peripheral smear in the diagnosis of anemia
Causes and treatment of microcytic hypochromic anemia
Causes
Iron deficiency anemia
Treatment of underlying cause of iron deficiency anemia
Treatment of iron deficiency
Oral iron therapy →
Dose is 6 mg/kg/day.
Commonly used preparations are-
Ferrous sulphate (200 mg TID) = 180 mg ferrous iron
Ferrous gluconate (300 mg BD) = 70 mg ferrous iron
Ferrous fumarate (325 mg BD or TID)
Side effects → Metallic taste, nausea, dyspepsia, constipation, black tarry stools. Can
be reduced by taking with food.
Response → Appears within 7-10 days of treatment in the form of increased
reticulocyte count.
Parenteral iron therapy
Used if oral iron is not well tolerated or the compliance is not good
Calculation of dose = 2.3 x Body weight (kg) x (Normal Hb level - Patient’s Hb) + 500
mg
Route → Slow IV infusion or IM injection
Preparations
Iron sorbitol → IM route
Iron dextran → IM route. Using Z-tract technique
Iron sucrose → Slow IV infusion
Ferric carboxymaltose →Lesser side effects.
Red cell transfusion
Acute and severe conditions
Features of cardiovascular failure
Require immediate intervention
Thalassemia
Hyper transfusion to maintain Hb level between 10–12 g/dL is probably adequate. It
decreases the effect of chronic anemia and prevents abnormal growth and
development. Super-transfusion wherein Hb level is maintained at 12 g/dL is
designed to completely suppress hematopoiesis
With repeated transfusions, iron overload inevitably develops and can result in heart
and liver failure. Iron overload is treated using iron chelators like deferoxamine
(injected parenterally if serum ferritin is > 1500 mcg/L) or deferasirox and
deferiprone (both oral).
Splenectomy is indicated in children with with massive symptomatic splenomegaly
and those with progressively increasing requirement of blood transfusion.
The definitive management is bone marrow/stem cell transplantation.
Anemia of chronic disease
Treat the underlying disorder
Recombinant erythropoietin therapy may be tried if anemia is not corrected after
treatment of underlying disorder
Sideroblastic anemia
Withdrawal of the causative agent.
Occasional cases may respond to pyridoxine or folic acid
Supportive treatment with transfusions
Erythropoietin
IRON DEFICIENCY ANEMIA
Treatment of iron deficiency anemia
Treatment consists of the following
Treatment of underlying cause of iron deficiency anemia
Treatment of iron deficiency
Oral iron therapy →
Dose is 6 mg/kg/day.
Commonly used preparations are-
Ferrous sulphate (200 mg TID) = 180 mg ferrous iron
Ferrous gluconate (300 mg BD) = 70 mg ferrous iron
Ferrous fumarate (325 mg BD or TID)
Side effects → Metallic taste, nausea, dyspepsia, constipation, black tarry stools. Can
be reduced by taking with food.
Response → Appears within 7-10 days of treatment in the form of increased
reticulocyte count.
Parenteral iron therapy
Used if oral iron is not well tolerated or the compliance is not good
Calculation of dose = 2.3 x Body weight (kg) x (Normal Hb level - Patient’s Hb) + 500
mg
Route → Slow IV infusion or IM injection
Preparations
Iron sorbitol → IM route
Iron dextran → IM route. Using Z-tract technique
Iron sucrose → Slow IV infusion
Ferric carboxymaltose →Lesser side effects.
Red cell transfusion
Acute and severe conditions
Features of cardiovascular failure
Require immediate intervention
Mention TWO causes for microcytic hypochromic anemia with low ferritin in a male
patient aged 65 years
Causes
Chronic blood loss
Malabsorption syndromes → Coeliac disease
MEGALOBLASTIC ANEMIA
A 56 year old lady is brought with severe anemia. Discuss the clinical features
and investigations that would help in making a diagnosis. Add a note on
treatment of megaloblastic anemia.
Clinical features
Symptoms
Palpitations
Angina
Symptoms of cardiac failure
Intermittent claudication
Fatigue
Irritability
Syncope
Throbbing in head and ears
Sleep disturbance
Dyspnea
Dizziness
Headache
Lack of concentration
Transient cerebral ischemia
Signs
Pallor of skin, palms, oral mucous membranes, nail beds and palpebral conjunctivae.
The palmar crease become as pale as the surrounding skin when Hb < 7 g/dL
Tachycardia, wide pulse pressure
Oedema
Cervical venous hum, hyperdynamic precordium
Ejection systolic murmur, best heard over pulmonary area
Signs of cardiac failure
Investigations
Treatment of megaloblastic anemia
Supportive therapy
Blood transfusion should be given in significantly symptomatic or severely anemic
patients. Adequate precautions are to be taken to avoid circulatory overloading,
especially in elderly patients. Intravenous furosemide 20-40 mg may be given prior to
transfusion.
Treatment of infections
Treatment of cardiac failure
Specific therapy
Vitamin B12 therapy
Initial dose → Six IM injections of hydroxocobalamin 1000 mcg given at 3-7 days
intervals
Maintenance dose → 1000 mcg to be given IM every 3 months
Folate therapy → Ensure that B12 is normal before correcting folate deficiency to
prevent aggravation of cobalamin neuropathy
Oral dose of 5 mg folate daily for 3 weeks will treat acute deficiency and 5 mg once
weekly is adequate maintenance therapy.
Etiology and investigation of megaloblastic anemia
Etiology
Investigations
Peripheral blood
Hb and Hematocrit → Reduced
Red cell indices → MCV > 100 fL (normal 80-100)
Pancytopenia
Peripheral smear
RBCs →
Macrocytic and oval
Anisopoikilocytosis → Marked variation in size and shape
Evidence of dyserythropoiesis → Basophilic stippling, Cabot ring and Howell-Jolly
bodies
WBCs
Decreased count
Hyper-segmented neutrophils (more than 5 nuclear lobes)
Platelets
Decreased
Reticulocyte count → Normal or low
Bone marrow
Markedly hypercellular
Megaloblastic type of erythropoiesis
Giant metamyelocytes and band forms
Bone marrow iron → Markedly increased
Biochemical tests (Both B12 and Folic acid deficiency)
Deoxyuridine suppression test
Serum homocysteine levels → Increased
Serum bilirubin → Indirect bilirubin is mildly increased
Plasma LDH → Marked increase
Serum vitamin B12 and folate decreased
Diagnostic tests for B12 deficiency
Methylmalonic acid is increased in B12 deficiency whereas its normal in folic acid
deficiency
Schilling test for Vitamin B12 absorption
Diagnostic test for folate deficiency
RBC folic acid levels → Reduced
FIGLU in urine → Excessively increased
Diagnosis and treatment of megaloblastic anemia
Diagnosis
Peripheral blood
Hb and Hematocrit → Reduced
Red cell indices → MCV > 100 fL (normal 80-100)
Pancytopenia
Peripheral smear
RBCs →
Macrocytic and oval
Anisopoikilocytosis → Marked variation in size and shape
Evidence of dyserythropoiesis → Basophilic stippling, Cabot ring and Howell-Jolly
bodies
WBCs
Decreased count
Hyper-segmented neutrophils (more than 5 nuclear lobes)
Platelets
Decreased
Reticulocyte count → Normal or low
Bone marrow
Markedly hypercellular
Megaloblastic type of erythropoiesis
Giant metamyelocytes and band forms
Bone marrow iron → Markedly increased
Biochemical tests (Both B12 and Folic acid deficiency)
Deoxyuridine suppression test
Serum homocysteine levels → Increased
Serum bilirubin → Indirect bilirubin is mildly increased
Plasma LDH → Marked increase
Serum vitamin B12 and folate decreased
Diagnostic tests for B12 deficiency
Methylmalonic acid is increased in B12 deficiency whereas its normal in folic acid
deficiency
Schilling test for Vitamin B12 absorption
Diagnostic test for folate deficiency
RBC folic acid levels → Reduced
FIGLU in urine → Excessively increased
Treatment
Treatment consists of treating the underlying cause wherever possible
Specific therapy
Vitamin B12 deficiency
Vitamin B12 therapy (Cyanocobalamin, hydroxycobalamin or methylcobalamin may
be used)
Dosage
Initial dose → Six IM injections of hydroxycobalamin 1000 mcg given at 3-7 days
internal
Maintenance dose → 1000 mcg IM every 3 months
Folate deficiency
Dosage
Initial dose → Oral 5 mg folate daily for 3 weeks
Maintenance → 5 mg once weekly
Always check for normal B12 levels before initiating folate therapy to prevent
cobalamin neuropathy
Supportive therapy
Blood transfusions
Four important investigations in the evaluation of megaloblastic anemia //
Laboratory diagnosis of megaloblastic anemia (x2)
Peripheral blood
Hb and Hematocrit → Reduced
Red cell indices → MCV > 100 fL (normal 80-100)
Pancytopenia
Peripheral smear
RBCs →
Macrocytic and oval
Anisopoikilocytosis → Marked variation in size and shape
Evidence of dyserythropoiesis → Basophilic stippling, Cabot ring and Howell-Jolly
bodies
WBCs
Decreased count
Hyper-segmented neutrophils (more than 5 nuclear lobes)
Platelets
Decreased
Reticulocyte count → Normal or low
Bone marrow
Markedly hypercellular
Megaloblastic type of erythropoiesis
Giant metamyelocytes and band forms
Bone marrow iron → Markedly increased
Biochemical tests (Both B12 and Folic acid deficiency)
Deoxyuridine suppression test
Serum homocysteine levels → Increased
Serum bilirubin → Indirect bilirubin is mildly increased
Plasma LDH → Marked increase
Serum vitamin B12 and folate decreased
Diagnostic tests for B12 deficiency
Methylmalonic acid is increased in B12 deficiency whereas its normal in folic acid
deficiency
Schilling test for Vitamin B12 absorption
Diagnostic test for folate deficiency
RBC folic acid levels → Reduced
FIGLU in urine → Excessively increased
A 35 years old man admitted with history of easy fatigability. On examination
has moderate pallor, Jaundice and pigmentation on the palm and the knuckles.
His laboratory investigation shows Hemoglobin 4.5gm/dL, Total leucocyte
count 3.5 x 1000 /cu.mm , Platelet count 56,000/cu.mm. RBC indices revels
MCV of 112fl with LDH 1024 IU/L. a. What is your diagnosis? b. What are the
risk factors for this illness? c. Discuss the investigations necessary to confirm
your diagnosis d. How do you treat this patient?
Megaloblastic anemia
Risk factors
Investigations
Peripheral blood
Hb and Hematocrit → Reduced
Red cell indices → MCV > 100 fL (normal 80-100)
Pancytopenia
Peripheral smear
RBCs →
Macrocytic and oval
Anisopoikilocytosis → Marked variation in size and shape
Evidence of dyserythropoiesis → Basophilic stippling, Cabot ring and Howell-Jolly
bodies
WBCs
Decreased count
Hyper-segmented neutrophils (more than 5 nuclear lobes)
Platelets
Decreased
Reticulocyte count → Normal or low
Bone marrow
Markedly hypercellular
Megaloblastic type of erythropoiesis
Giant metamyelocytes and band forms
Bone marrow iron → Markedly increased
Biochemical tests (Both B12 and Folic acid deficiency)
Deoxyuridine suppression test
Serum homocysteine levels → Increased
Serum bilirubin → Indirect bilirubin is mildly increased
Plasma LDH → Marked increase
Serum vitamin B12 and folate decreased
Diagnostic tests for B12 deficiency
Methylmalonic acid is increased in B12 deficiency whereas its normal in folic acid
deficiency
Schilling test for Vitamin B12 absorption
Diagnostic test for folate deficiency
RBC folic acid levels → Reduced
FIGLU in urine → Excessively increased
Treatment
Treatment consists of treating the underlying cause wherever possible
Specific therapy
Vitamin B12 deficiency
Vitamin B12 therapy (Cyanocobalamin, hydroxycobalamin or methylcobalamin may
be used)
Dosage
Initial dose → Six IM injections of hydroxycobalamin 1000 mcg given at 3-7 days
internal
Maintenance dose → 1000 mcg IM every 3 months
Folate deficiency
Dosage
Initial dose → Oral 5 mg folate daily for 3 weeks
Maintenance → 5 mg once weekly
Always check for normal B12 levels before initiating folate therapy to prevent
cobalamin neuropathy
Supportive therapy
Blood transfusions
HEMOLYTIC ANEMIAS
22 year old with recurrent anemia, jaundice, splenomegaly-give differential diagnosis
and discuss treatment of any one condition.
Differential diagnosis -
Hemolytic anemia → eg. thalassemia, sickle cell disease, auto-immune hemolytic
anemia
Lymphoma
Hepatitis → Acute fulminant hepatitis can cause aplastic anemia, along with
splenomegaly.
Treatment of hemolytic anemia
Good nutrition and folic acid supplementation (5 mg daily) to be given to all patients
with hemolysis
If sickle cell disease is the cause, timely immunization against S. pneumoniae,
seasonal influenza, Neisseria meningitidis, H influenzae type B and hepatitis B virus
Splenectomy is indicated in children with with massive symptomatic splenomegaly
and those with progressively increasing requirement of blood transfusion.
Blood transfusions should be given for clear indications in sickle cell disease.
Hypertransfusion to be done in thalassemia to maintain Hb level between 10-12
mg/dL.
With repeated transfusions, iron overload inevitably develops and can result in heart
and liver failure. Iron overload is treated using iron chelators like deferoxamine
(injected parenterally if serum ferritin is > 1500 mcg/L) or deferasirox and
deferiprone (both oral).
Hydroxyurea may be used in hemoglobinopathies like sickle cell disease and
thalassemia to increase fetal hemoglobin.
Treatment of chronic leg ulcers in sickle cell disease should be carried out by
elevation of limb, daily dressings with zinc sulphate and exchange transfusions in
extreme cases.
The definitive management is bone marrow/stem cell transplantation.
In hemolytic anemia due to G6PD deficiency the triggering agent should be
removed.
45 year old female who had no premorbid illness, has severe anemia, mild jaundice
and mild splenic enlargement. What is the likely diagnosis? How do you investigate
her further?
Auto-immune hemolytic anemia (Common in middle-aged females)
Investigations
Hb decreased
CBC → RBCs decreased
Peripheral smear → Schistocytes and macrocytes seen
Reticulocytes increased
Direct Coombs test is positive
Auto-immune thrombocytopenia and/or neutropenia may also be present (Evan’s
syndrome)
Treatment
Corticosteroids (e.g. Prednisolone 1 mg/kg daily) are effective in about 80% of
patients.
Start prednisolone 60 mg daily
Gradually taper the dose
When a dose of 20-30 mg daily achieves a persistent remission (Stable Hb level and
decreased reticulocyte count), it is recommended to given prednisolone on alternate
days
Avoid blood transfusion as far as possible, because autoantibodies may cause
difficulty in crossmatching the blood
Danazol can be used along with prednisolone as first line therapy allowing for a
shorter duration of prednisolone therapy
Splenectomy may be necessary if
no response to corticosteroid
the remission is not maintained when dose of prednisolone is reduced
IVIG may be used as temporary measure before performing splenectomy
Rituximab may be used in refractory cases
Clinical features and investigations of hereditary spherocytosis
Clinical features
Age of presentation → Anytime from neonatal period to adulthood
Family history → Autosomal dominant disorder. Hence strong family history.
Anemia - mild to moderate
Jaundice - intermittent attacks
Splenomegaly - moderate splenic enlargement
Children - growth retardation due to hemolysis and bone changes due to marrow
hyperplasia
Complications - cholelithiasis, aplastic crises, hemolytic crises
Investigations
Anemia
Spherocytes on peripheral smear. MCHC increased.
Raised serum bilirubin and urinary urobilinogen
Increased osmotic fragility
Negative Coombs test
Treatment
Splenectomy is the treatment of choice. It should be preceded by pneumococcal and
Hemophilus influenzae immunization and followed by lifelong penicillin prophylaxis
Folic acid supplementation in patients without splenectomy
Regular blood transfusions are required in few patients with severe disease
Cholecystectomy for symptomatic gallstones
BLEEDING DISORDERS & THROMBOCYTOPENIA
How do you investigate a case of thrombocytopenia to come to diagnosis of
Immune Thrombocytopenic Purpura (ITP)?
ITP is a diagnosis of exclusion.
Investigations
Platelet count markedly reduced and is below 80,000/cu.mm
Tourniquet Test (Hess test) → Positive (>10 petechiae seen)
Bleeding time → Increased
Bone marrow examination → It is important in chronic ITP to rule out
thrombocytopenia resulting from bone marrow failure. Bone marrow shows
moderate increase in number of both immature and mature forms of
megakaryocytes. A decrease in the number of megakaryocytes argues against the
diagnosis of ITP
Antiplatelet antibodies → Positive antibodies against GP IIb-IIIa and GP Ib/IX.
Negative test doesn’t rule out ITP
In chronic ITP, secondary causes such as HIV, hepatitis C virus infection, etc. should
be excluded
Mention two causes of thrombocytopenia with splenomegaly. Discuss clinical
features and management of Idiopathic Thrombocytopenic Purpura.
Cirrhosis
Lymphoma
Acute leukemia
Clinical features
Occur due to thrombocytopenia
No physical signs other than those due to bleeding and anemia
Menorrhagia
Epistaxis
Intracranial hemorrhage
GI bleeding and hemorrhage is uncommon
Splenomegaly and lymphadenopathy are uncommon in primary ITP
Majority of patients recover within 6 months
Investigations
Platelet count markedly reduced and is below 80,000/cu.mm
Tourniquet Test (Hess test) → Positive (>10 petechiae seen)
Bleeding time → Increased
Bone marrow examination → It is important in chronic ITP to rule out
thrombocytopenia resulting from bone marrow failure. Bone marrow shows
moderate increase in number of both immature and mature forms of
megakaryocytes. A decrease in the number of megakaryocytes argues against the
diagnosis of ITP
Antiplatelet antibodies → Positive antibodies against GP IIb-IIIa and GP Ib/IX.
Negative test doesn’t rule out ITP
In chronic ITP, secondary causes such as HIV, hepatitis C virus infection, etc. should
be excluded
Treatment
Indications
Overt hemorrhage
Platelet counts < 20,000/cu.mm.
Organ failure or life-threatening bleeding irrespective of the circulating platelet
count
Platelet transfusion → if platelet counts < 20,000/cu.mm or bleeding manifestations
First line therapy → Oral corticosteroids
Indications
To reduce remissions
To maintain remission in chronic ITP
Dosage
Initial dose is 1-2 mg/kg of prednisolone/day.
The initial dose is continued for at least two weeks and then reduced and slowly
stopped
About 66% respond but relapse is common when dose is reduced
Second line therapy → Splenectomy
Third line therapy →
High dose corticosteroid
IVIG
RhO(D) immune globulin (anti-D)
Immunosuppressive therapy → Vincristine, vinblastine, Azathioprine,
cyclophosphamide, cyclosporine and combination chemotherapy, mycophenolate
mofetil
Plasmapheresis
TPO mimetic drugs → Eltrombopag or romiplostim
Rituximab
Emergency treatment → Necessary in life-threatening bleeding.
IV administration of methylprednisolone (30 mg/kg/day, maximum 1 g) over 20-30
minutes along
Platelet transfusion
This is followed by IVIG (1 g/kg)
Idiopathic thrombocytopenic purpura.
Clinical features
Occur due to thrombocytopenia
No physical signs other than those due to bleeding and anemia
Menorrhagia
Epistaxis
Intracranial hemorrhage
GI bleeding and hemorrhage is uncommon
Splenomegaly and lymphadenopathy are uncommon in primary ITP
Majority of patients recover within 6 months
Investigations
Platelet count markedly reduced and is below 80,000/cu.mm
Tourniquet Test (Hess test) → Positive (>10 petechiae seen)
Bleeding time → Increased
Bone marrow examination → It is important in chronic ITP to rule out
thrombocytopenia resulting from bone marrow failure. Bone marrow shows
moderate increase in number of both immature and mature forms of
megakaryocytes. A decrease in the number of megakaryocytes argues against the
diagnosis of ITP
Antiplatelet antibodies → Positive antibodies against GP IIb-IIIa and GP Ib/IX.
Negative test doesn’t rule out ITP
In chronic ITP, secondary causes such as HIV, hepatitis C virus infection, etc. should
be excluded
Treatment
Indications
Overt hemorrhage
Platelet counts < 20,000/cu.mm.
Organ failure or life-threatening bleeding irrespective of the circulating platelet
count
Platelet transfusion → if platelet counts < 20,000/cu.mm or bleeding manifestations
First line therapy → Oral corticosteroids
Indications
To reduce remissions
To maintain remission in chronic ITP
Dosage
Initial dose is 1-2 mg/kg of prednisolone/day.
The initial dose is continued for at least two weeks and then reduced and slowly
stopped
About 66% respond but relapse is common when dose is reduced
Second line therapy → Splenectomy
Third line therapy →
High dose corticosteroid
IVIG
RhO(D) immune globulin (anti-D)
Immunosuppressive therapy → Vincristine, vinblastine, Azathioprine,
cyclophosphamide, cyclosporine and combination chemotherapy, mycophenolate
mofetil
Plasmapheresis
TPO mimetic drugs → Eltrombopag or romiplostim
Rituximab
Emergency treatment → Necessary in life-threatening bleeding.
IV administration of methylprednisolone (30 mg/kg/day, maximum 1 g) over 20-30
minutes along
Platelet transfusion
This is followed by IVIG (1 g/kg)
Management of Idiopathic thrombocytopenic purpura
Investigations
Platelet count markedly reduced and is below 80,000/cu.mm
Tourniquet Test (Hess test) → Positive (>10 petechiae seen)
Bleeding time → Increased
Bone marrow examination → It is important in chronic ITP to rule out
thrombocytopenia resulting from bone marrow failure. Bone marrow shows
moderate increase in number of both immature and mature forms of
megakaryocytes. A decrease in the number of megakaryocytes argues against the
diagnosis of ITP
Antiplatelet antibodies → Positive antibodies against GP IIb-IIIa and GP Ib/IX.
Negative test doesn’t rule out ITP
In chronic ITP, secondary causes such as HIV, hepatitis C virus infection, etc. should
be excluded
Treatment
Indications
Overt hemorrhage
Platelet counts < 20,000/cu.mm.
Organ failure or life-threatening bleeding irrespective of the circulating platelet
count
Platelet transfusion → if platelet counts < 20,000/cu.mm or bleeding manifestations
First line therapy → Oral corticosteroids
Indications
To reduce remissions
To maintain remission in chronic ITP
Dosage
Initial dose is 1-2 mg/kg of prednisolone/day.
The initial dose is continued for at least two weeks and then reduced and slowly
stopped
About 66% respond but relapse is common when dose is reduced
Second line therapy → Splenectomy
Third line therapy →
High dose corticosteroid
IVIG
RhO(D) immune globulin (anti-D)
Immunosuppressive therapy → Vincristine, vinblastine, Azathioprine,
cyclophosphamide, cyclosporine and combination chemotherapy, mycophenolate
mofetil
Plasmapheresis
TPO mimetic drugs → Eltrombopag or romiplostim
Rituximab
Emergency treatment → Necessary in life-threatening bleeding.
IV administration of methylprednisolone (30 mg/kg/day, maximum 1 g) over 20-30
minutes along
Platelet transfusion
This is followed by IVIG (1 g/kg)
Thrombocytopenia
Definition → Decrease in the platelet count below the lower limit of 150,000/mcL
Causes

Clinical features
Bleeding into skin - petechiae, purpura, ecchymoses
Bleeding into mucous membranes - epistaxis, hemorrhagic bullae in oral mucosa,
genitourinary bleeding and gastrointestinal bleeding
Fundal hemorrhage and intracranial bleeding
Investigations
Platelet count - reduced
Hess test (capillary fragility test) - may be positive (>10 petechiae)
Bleeding time - prolonged
Bone marrow -
normal or increased megakaryocytes indicates increased platelet destruction
decreased megakaryocytes indicate reduced production
CLOTTING DISORDERS
Clinical features of hemophilia A
Depends on level of factor VIII activity
Excessive bleeding
Post-traumatic bleeding → characteristically delayed
Severity of bleeding → Mild to Severe
Petechiae are not seen in hemophilia (feature of platelet disorder)
Hemarthrosis (Bleeding into joints)
Frequent and spontaneous hemorrhages into large joints
Acute stage → Affected joint is swollen, hot and tender and movements are severely
restricted
Consequences → Recurrent bleeding into joints will lead to deformities and atrophy
of muscles around joints
Bleeding into muscles
Common sites → Calf and psoas muscles
Consequences → Psoas hematomas may compress femoral nerve resulting in sensory
disturbances over thigh and weakness of quadriceps. Calf hematomas can lead to
contracture and shortening of the Achilles tendon.
Other manifestations
Easy bruising
Cerebral hemorrhages
Hematuria and ureteric colic due to passage of clots
PANCYTOPENIA
Discuss the causes and clinical features of aplastic anemia. Add a note on its
investigations and management (x2)
Clinical features
Anemia - Progressive weakness, fatigue, pallor and dyspnea
Neutropenia - Recurrent infections like sore throat, oral and pharyngeal ulcers, fever
with chills and sweating
Thrombocytopenia - Resulting in bleeding manifestations in the form of petechiae,
bruises and ecchymoses. These include epistaxis, menorrhagia, bleeding from gums,
GI tract, retinal hemorrhages and cerebral hemorrhage
Lymphadenopathy and hepatosplenomegaly are rare.
Fanconi Anemia
Short stature
Ectopic kidney, horse-shoe kidney
Hydrocephalus
Investigations
Hb decreased
PCV decreased
Reticulocytes decreased (0.5-1%) - characteristic feature
Peripheral smear - pancytopenia
Bone marrow study - marked hypocellularity with replacement of more than 70% of
marrow cells by fat.
Serum iron and transferrin saturation - increased
Treatment
Removal of causative factor wherever possible
Providing supportive care while awaiting marrow recovery. This includes
Prevention and treatment of infections
Treatment of hemorrhage
Treatment of anemia by RBC transfusions
Growth factors → G-CSF, TPO receptor agonists
Severe aplastic anemia
Patients under age of 40 years and HLA identical sibling/donor available → Bone
marrow transplantation
Patients above age of 40 years / HLA identical donor not available →
Immunosuppressive therapy
Eltrombopag, horse ATG, cyclosporine and prednisone in combination produce a
hematological response rate of 60-80%
Androgens (e.g. oxymetholone) are sometimes useful in patients not responding to
immunosuppression
Causes of pancytopenia
Complications of pancytopenia
Pancytopenia is a condition in which there is a decrease in the number of all types of
blood cells, including red blood cells, white blood cells, and platelets. Pancytopenia
can be caused by a variety of underlying medical conditions and can lead to several
complications, including:
Increased risk of infection: Pancytopenia can lead to a decreased number of white
blood cells, which are essential for fighting off infections. As a result, patients with
pancytopenia are at an increased risk of developing infections.
Anemia: Pancytopenia can cause a decrease in the number of red blood cells, leading
to anemia. Anemia can cause symptoms such as fatigue, weakness, and shortness of
breath.
Bleeding disorders: Pancytopenia can lead to a decrease in the number of platelets,
which can cause bleeding disorders and an increased risk of bleeding and bruising.
Delayed wound healing: The decreased number of platelets can also lead to delayed
wound healing.
Treatment of aplastic anemia
Treatment
Removal of causative factor wherever possible
Providing supportive care while awaiting marrow recovery. This includes
Prevention and treatment of infections
Treatment of hemorrhage
Treatment of anemia by RBC transfusions
Growth factors → G-CSF, TPO receptor agonists
Severe aplastic anemia
Patients under age of 40 years and HLA identical sibling/donor available → Bone
marrow transplantation
Patients above age of 40 years / HLA identical donor not available →
Immunosuppressive therapy
Eltrombopag, horse ATG, cyclosporine and prednisone in combination produce a
hematological response rate of 60-80%
Androgens (e.g. oxymetholone) are sometimes useful in patients not responding to
immunosuppression
TRANSFUSION AND RELATED COMPLICATIONS
Immunological complications related to blood/blood products transfusion

TRALI
Transfusion related acute lung injury is syndrome characterized by acute respiratory
distress following transfusion.
It is caused by anti-HLA and/or anti-granulocyte antibodies in donor plasma that
agglutinate and degranulate recipient granulocytes withing the lung.
Clinical features
Symptoms within 6 hours of transfusion
Breathlessness
Associated fever, cyanosis and hypotension
Bilateral crepitations
CXR → Bilateral pulmonary edema
Treatment
Mild → Oxygen supplementation
Severe → Mechanical ventilation and ICU support
Indications and complications of transfusion therapy
Indications
RBC transfusions
Given to patients to raise hematocrit levels in patients with severe anemia or to
replace losses during acute bleeding episodes
Three types
Whole blood : Contains all components RBCs, plasma and platelets. Used during
surgery.
Packed RBCs (PRBCs) : Each unit has a volume of 300 mL of which approximately 200
mL is RBCs. Given to patients with severe anemia who have normal blood volume.
Autologous PRBCs : Patients scheduled for elective surgery may donate their blood
beforehand for transfusion during surgery. These units may be stored up to 35 days
Fresh Frozen Plasma (FFP)
FFP is prepared from single units of whole blood or from plasma collected by
apheresis technique. It can be stored up to 1 year
Contains all coagulation factors present in blood
ABO compatibility required before transfusion but Rh typing not required
Dose : 10-15 ml/kg (3-5 units) will correct coagulation abnormality
Indications
Bleeding due to excessive warfarin, Vitamin K deficiency or deficiency of multiple
coagulation factors (e.g. DIC, liver disease, dilutional coagulopathy)
Treatment of TTP-HUS
Factor XIII, VIII or IX deficiency
Platelet concentrate (PC) or Platelet Rich Plasma (PRP)
One unit is made from four or five donation of whole blood, or from a single platelet
apheresis technique
Shelf life up to 5 days of collection
ABO compatibility is required
One unit increases platelet count by 4000-8000/cu.mm.
Indications
Bleeding due to thrombocytopenia or platelet dysfunction
Low platelets < 10,000 /cu.mm.
Prior to surgery in thrombocytopenic patients
Cryoprecipitate
Prepared by concentrating plasma near freezing point
Contains fibrinogen, factor VIII, factor IX and von Willebrand’s factor
Indications
Fibrinogen deficiency
von Willebrand’s disease and hemophilia A and B
LEUKEMIAS
Classification of acute myeloid leukemia
Hematological features and treatment of Chronic myeloid leukemia
Hematological features
Hb < 11 g/dL
TLC → Markedly increased. Almost always more than 20,000/mcL, often exceeding
1,00,000/mcL
Peripheral smear
RBC - normocytic normochromic anemia
WBC -
left shift (shift to immaturity) with granulocytes at all stages of development
(neutrophiles, metamyelocytes, myelocytes, promyelocytes, and occasional
myeloblasts)
Predominant cells are neutrophils and myelocytes
Blasts < 10% of circulating WBCs (Blasts will increase in blast crisis phase)
Platelets - count may be normal, increased or decreased
Bone marrow study → Markedly hypercellular due to marked hyperplasia of all
granulocytic elements
Treatment
Goals
Complete molecular remission
Achieve prolonged, durable, non-neoplastic hematopoiesis
Eradication of any residual cells containing BCR-ABL1 transcript
Chemotherapy
Imatinib mesylate is the first-line treatment
Dose = 300-400 mg/day
If there is failure of response or progress on imatinib, options include
Second generation tyrosine kinase inhibitors - dasatinib and nilotinib
Allogenic bone marrow transplantation
Classical cytotoxic drugs such as hyroxyurea, or interferon-alpha or busulphan
Splenectomy
Stem cell transplantation
Indications
Patients under the age of 60 years who have suitable donor
Those who do not respond to 2nd line tyrosine kinase inhibitors or those who
present with accelerated phase or blast crisis
Monitoring
Bone marrow biopsy every 6 months to determine cytogenetic response
Significance of cytogenetic study in chronic myeloid leukemia
Confirmation of diagnosis
Staging of disease
Monitor effectiveness of treatment
Predicting the likelihood for relapse
Write a note on clinical features (symptoms and signs) of chronic lymphocytic
leukemia
Demographics
Age: Most of the patients at the time of diagnosis are over 50 years of age
Sex: More common in males than in females with a ratio of 2:1.
Symptoms
Asymptomatic: In about 25% of patients and are detected either because of
nonspecific symptoms or routine blood examination for some other disease
Non-specific symptoms
Fatigue
Loss of weight
Anorexia
Slow developing anemia
Recurrent infections
Signs
Painless generalized lymphadenopathy
Splenomegaly
Hepatomegaly
Acute promyelocytic leukemia
Acute promyelocytic leukemia (M3)
Uncommon variant of AML associated with severe coagulation complications.
It has favorable prognosis.
Clinical features
Symptoms similar to other acute myeloid leukemias such as fatigue, loss of weight.
Easy bruising or bleeding
Recurrent infections due to neutropenia
Bone pain or sternal tenderness
Hepatosplenomegaly
Generalized lymphadenopathy
Disseminated intravascular coagulation is characteristically seen in APL
Investigations
Low Hb
Severe normochromic anemia
Numerous blast cells on peripheral smear
Bone marrow aspirate - Blast cells > 20%
CXR - mediastinal widening
Management
Induction therapy is with all-trans-retinoic acid (ATRA) and anthracyclines or a
combination of ATRA with arsenic trioxide.
Allogeneic transplantation: Necessary if the leukemia is not eliminated at the
molecular level, only in relapsed/refractory cases.
In relapsed cases, arsenic trioxide (induces apoptosis via activation of the caspase
cascade) has been also found to be effective.
Myelodysplastic syndrome
Myelodysplastic syndromes (MDS) are a heterogeneous group of acquired clonal
stem cell disorders characterized by:
Progressive cytopenias (low blood cell counts)
Dysplasia in one or more major cell lines (erythroid, myeloid and megakaryocytic
forms)
Ineffective hematopoiesis
Increased risk of development of AML
A subtype of myeloid neoplasms.
Clinical Features
Usually found in patients above 60 years and slightly more common in males
Detect incidentally in 50% of patients
Symptoms are due to cytopenias which may be single-lineage or multilineage
Extramedullary hematopoiesis may cause hepatomegaly and splenomegaly
About 10-40% cases progress to AML
Diagnosis
Dysplasia in at least 10% of cells of any one of the myeloid lineages
Peripheral smear
RBC - Mild to moderate degree of macrocytic or dimorphic anemia
WBC - Normal or low. Neutropenia with few blasts. Number of blasts decide the type
of MDS
Platelets - Variable thrombocytopenia, presence of large hypogranular or giant
platelets is seen
Bone marrow → Varying degree of dyspoietic (disordered) differentiation affecting all
non-lymphoid lineages associated with cytopenias
Cytogenetic study of marrow
Treatment
Supportive therapy
PRBC for anemia
Platelet transfusion for bleeding due to thrombocytopenia
Antibiotic therapy for infections
EPO and G-CSF may be useful in some patients
Thalidomide, lenalidomide, 5-azacitidine, and decitabine may reduce requirements of
blood transfusion and to retard the progression of MDS to AML.
Allogenic hematopoietic stem cell transplantation. Curative. However rarely
performed because MDS is most common during seventh or eighth decade of life
LYMPHOMA
Clinical features of Non-Hodgkin’s lymphoma
Age → NHL can occur at any age, but the peak incidence is around 60 years
Most common presentation
Painless, firm lymph node enlargement
Extranodal involvement is more common in T-cell lymphoma and involves the bone
marrow, gut, thyroid, lung, skin, testis, brain and more rarely bone
Waldeyer’s ring and epitrochlear lymph nodes are frequently involved
Pressure effects - gut obstruction, ascites, SVC obstruction, spinal cord compression
Hepatosplenomegaly
Patients with lymphoblastic lymphoma present with an anterior mediastinal mass
Burrkitt lymphoma typically disseminates to the bone marrow and meninges and
involves extranodal sites
B symptoms → Weight loss, night sweats, fever
Autoimmune hemolytic anemia or immune thrombocytopenia
Paraneoplastic complications
Neurologic → GBS, demyelinating polyneuropathy, autonomic dysfuntion
Kidney → Glomerulonephritis
Skin → Pemphigus
MULTIPLE MYELOMA
Current Treatment of multiple myeloma in 50 year old male
General measures
High fluid intake of about 3 L/day to treat renal impairment and hypercalcemia
Prompt treatment of infections with antibiotics
Treatment of anemia may require transfusions and erythropoietin also helps
Analgesic for relief from bone pain
Allopurinol 300 mg daily should be given to reduce/prevent hyperuricemia
Hyper viscosity syndrome is managed by plasmapheresis
Bisphosphonates to reduce skeletal events such as pathological fractures, cord
compression and bone pain
Autologous stem cell transplantation
Treatment of choice in multiple myeloma in eligible patients (Age < 65 years without
renal failure
Steps
Induction therapy with one of the following
Standard Risk → Lenalidomide, Dexamethasone, Bortezumib
High risk → Lenalidomide, Dexamethasone, Carfilozomib
After three cycles look for remission
In case of remission, HDT therapy using melphalan 200 mg/sq.m. IV autologous stem
cell transplantation
Chemotherapy
Older patients → Thalidomide combined with the alkylating agent (melphalan or
cyclophosphamide or chlorambucil) and prednisolone
Younger Patients → Lenalidomide, Dexamethasone, and Bortezumib
Radiotherapy
Effective for local problems like severe bone pain, pathological fractures and
tumorous lesions.
As an emergency treatment of spinal cord compression complicating extradural
plasmacytomas.
Multiple myeloma - clinical features and diagnosis
OTHERS
Erythrocyte Sedimentation Rate
Rate at which RBCs settle down when anticoagulated whole blood is allowed to
stand.
Normal values
Men → 1-10 mm 1st hour
Women → 5-20 mm 1st hour
Factors affecting ESR
Plasma factors → Increased in plasma proteins (especially fibrinogen), reduces the
repulsive forces of RBCs and cause them to stack together like tires or rouleaux.
These rouleaux have more mass/surface area ration than single RBCs and therefore
sediment faster and increase the ESR.
Elevated fibrinogen and raised ESR is observed in pregnancy, old age, and end-stage
renal disease.
In acute inflammation ESR is raised
ESR is increased in conditions associated with monoclonal (multiple myeloma) or
polyclonal (chronic infections like TB) increase in immunoglobulins.
Red cell factors
Anemia increases the ESR and polycythemia decreases it
Microcytes sediment slower than macrocytes
Red cells with abnormal morphology like sickle cells or spherocytes, do not exhibit
rouleaux formation and have low ESR
Febrile neutropenia a. How do you define febrile neutropenia? b. How do you
classify it according to severity? c. Discuss the pathophysiology d. What
complications are expected? And how to manage such patients including
empiric antibiotic therapy and give reasons for the choice?
Neutropenic fever is defined as a fever/pyrexia of 38.3°C (≥101 F) for more than 1
hour in a patient with a neutrophil count <500/mm3.
Classification
Severity Neutrophil Count (cells/cu.mm)
Mild ≥ 500
Moderate 100-499
Severe < 100
The pathophysiology of febrile neutropenia is complex and involves a number of
different mechanisms. One of the main factors that can lead to the development of
febrile neutropenia is the use of chemotherapy or other medications that suppress
the body's immune system. These medications can reduce the production of
neutrophils, leading to a decrease in their number in the blood. This can increase the
risk of infections, which can then cause a fever.
Complications
Hypotension
Systemic circulatory shutdown
Organ failure
Treatment
Antibiotic therapy

Anti-fungal → If no response to antibiotics after 72-96 hours, treatment with


amphotericin B or voriconazole should be given to cover fungal infections
Growth factors → G-CSF, GM-CSF are given to improve blood counts
Four Causes of Hypersplenism
Hypersplenism features
Splenomegaly
Any combination of anemia, leukopenia and/or thrombocytopenia
Compensatory bone marrow hyperplasia
Improvement after splenectomy
Causes
CML or myelofibrosis
Infections such as malaria, HIV/AIDS and TB
Autoimmune disorders such as rheumatoid arthritis and SLE
Gaucher’s disease and Niemann Pick Disease
Sickle cell crisis
Indications for use of anticoagulants. Mention the types of anticoagulants and
examples
Role of anticoagulants in medicine

GIT incl. Hepatology and Pancreas


ESOPHAGUS
A female patient aged 55 years presents to outpatient department with history
of progressive difficulty in swallowing since 3 months. a. Mention possible
causes in this patient b. How do you investigate this patient? c. Discuss the
management of achalasia of the esophagus
Possible causes →
Esophageal lumen → Foreign body
Esophageal wall → Strictures, achalasia cardia, carcinoma oesophagus, Plummer-
Vinson syndrome, Diffuse esophageal spasm
Outside the esophageal wall → Thyroid swelling, secondaries in neck, mediastinal
abscess, aortic aneurysm
Others → Pharyngitis, Sjogren’s syndrome
Investigations
GERD - causes and management
Causes
Defective anti-reflux barriers
Hiatus hernia (Sliding type)
Abnormalities of LES
Cigarette smoking, chocolate, caffeine, alcohol, fatty food
Cardiomyotomy and vagotomy
Drugs : nitrates, CCBs, beta-agonists
Crural diaphragm
Increased intra-abdominal pressure : Pregnancy, obesity, ascites, weight lifting,
straining
Delayed esophageal clearance of refluxed acid
Impaired peristalsis
Reduced salivation
Body position
Defective gastric emptying
Gastric outlet obstruction
Anticholinergic drugs
Fatty food
Clinical features
Triad
Heartburn
Acid regurgitation
Epigastric pain
Cough and wheezing may also be present
Alarm symptoms
Dysphagia
Odynophagia
Anemia
Persistent heartburn
Family history of cancer
GI bleed - malena, hematochezia
Investigations
Endoscopy to detect erosive esophagitis or complications like Barrett’s esophagus
(confirmed by biopsy)
Barium radiography : May be used in the diagnosis of hiatus hernia
24 hour pH-monitoring : Gold standard
Esophageal manometry : To rule out motility disorders
Resting ECG and stress ECG : To rule out ischemic heart disease
Treatment
Life style modifications
Avoid alcohol, smoking, caffeine, chocolate, mint, spicy foods
Avoid late night meals just before lying down
Avoid weight lifting, stooping and bending at waist
Elevation of the head end of the bed
Weight reduction
Frequent feeds of small volume
Medical treatment
Antacids : Liquid antacids buffer acid and increase LESP. They are used in the dose of
10-15 mL, 1 and 3 hours after meal and at bedtime as needed.
PPIs : Omeprazole (20-40 mg/day), Lansoprazole ((15-30 mg/day)
H2 antagonists : Ranitidine (150 mg QID), cimetidine (400 mg QID)
Surgical treatment
Indications
Failure to respond to medical therapy
Patient not willing to take long term PPIs or intolerant to PPIs
Severe symptoms
Procedure →
Laparoscopic fundoplication
Surgical resection of esophageal strictures
UPPER GI BLEED
Discuss acute upper gastrointestinal hemorrhage under following headings: a.
Etiological factors b. Investigations c. Emergency management of acute
nonvariceal upper gastro-intestinal hemorrhage
Causes of upper GI bleed
Esophageal causes
Esophageal varices
Erosive esophagitis
Esophageal carcinoma
Mallory-Weiss Syndrome
Gastroduodenal Causes
Peptic ulcer
Erosive gastritis
Stress ulcers occurring with shock, sepsis or trauma
Curling ulcers in the proximal duodenum with severe burns or trauma
Carcinoma stomach
Other causes
Coagulation defects (Hemophilia)
Cameron’s lesions
Hemobilia
Hemosuccus pancreaticus
Investigations
General investigations
Complete blood count
Chronic bleeding leads to anemia
Low platelet suggest hematological disorder
Blood urea nitrogen and creatine → BUN increases to greater extent than creatine
because of increased intestinal absorption of urea after breakdown of blood proteins
Liver function tests - Evidence of chronic liver disease
Prothrombin time - Prolonged in patients with chronic liver disease or in coagulation
disorders
Cross-matching - Potential infusion of packed red cells
Specific investigations to find the site of bleed
Management
Medical resuscitation and replenishment of intravascular volume
IV access → The first step is to gain IV access using at least one large bore IV catheter
Fluid resuscitation → Adequate resuscitation and hemodynamic stabilization is
essential prior to endoscopy( saline). Vasopressors if persistent hypotension.
Blood products →
if Hb < 7 g/dL → Blood transfusion to maintain Hb at level ≥9 g/dL
if active bleeding and low platelet count (<50,000) → Platelet transfusion
If prolonged prothrombin and INR > 1.5 → FFP
Initial clinical assessment
Circulatory status → Monitor pulse, BP, postural hypotension, urine output and level
of consciousness
Evidence of liver disease → In patients with decompensated cirrhosis
Endoscopy
IV PPI → Esmoprazole 80 mg bolus, 8 mg/hr drip
Pre-endoscopic pharmacotherapy for non-variceal UGIB
Suppresses acid, facilitates clot formation and stabilization
Endoscopic hemostasis therapy
Epinephrine injection, fibrin glue, human thrombin, sclerosants
Thermal electrocoagulation, laser therapy
Mechanical hemoclips, banding, endoloops, staples, sutures
Post-endoscopy management for non-variceal UGIB
Patients with ulcers requiring endoscopic therapy should receive PPI x 72 hours
Determine H. pylori status in all ulcer patients.
Discharge patients on PPI (once to twice daily)
In patients with cardiovascular disease on low dose aspirin: Restart as soon as
bleeding has resolved.
A chronic alcoholic patient has come to the emergency department with the
history of hematemesis. fie has been having ascites and pedal oedema for the
last three months. a. What is the likely etiology? b. What are the clinical
features? c. What are the possible complications? d. Describe the management
of this patient.
Acute esophageal variceal bleed due to portal hypertension secondary to chronic
liver disease
Clinical features
Hematemesis
Red with clots - when bleeding is severe
Black - less severe bleeding
Malena - black, tarry, foul smelling stools containing altered blood
Hematochezia - Severe/Massive acute upper GI bleeding
Occasionally symptoms of blood loss only.
Acute loss → Dizziness, extreme pallor and shock
Chronic loss → Anemia
Complications
Anemia
Shock
Perforation
Re-bleeding
Management
Investigations
Fiberoptic endoscopy : performed within 8 hours of bleeding reveals bleeding stie
and the presence of varices. This helps in excluding other causes of bleeding.
USG : To confirm the patency of portal vein
Hemoglobin : To assess the blood loss
Blood grouping and cross matching
Liver function tests
AST and ALT elevated
AST : ALT ratio > 2
GGT elevated
PT/INR : Might be increased
Albumin : Might be decreased
Serum ammonia
Management of variceal bleed
Resuscitation
Immediate hospitalization and nursing
NPO until bleeding stops
Two large bore IV access
Consider airway protection by intubation
Assess general condition : pulse, blood pressure and maintain fluid and electrolyte
balance
Obtain blood for grouping and cross matching, hemoglobin, prothrombin time,
blood urea, electrolytes, creatine, LFTs and blood cultures
Immediately restore blood volume with blood transfusion.
Correction of coagulation factor deficiency by fresh blood or fresh frozen plasma
Platelet transfusions to raise platelet count above 50,000 /cu.mm.
IV/IM vitamin K
General measures
Grade cirrhosis according to Child-Pugh Score
H2 antagonists and PPIs to prevent stress ulcers
Prophylactic antibiotics : Prevent SBP. Usually quinolones are used (e.g. ciprofloxacin
500 mg BD)
Measures to prevent encephalopathy :
Lactulose therapy : 15-30 mL three times orally per day. Dose is increased gradually
till there are 2-3 loose stools per day
Lactitol (30 mg daily)
Rifaximin (550 mg BD)
Treatment of ascites by ascitic tap (paracentesis) or by administration of
spironolactone or amiloride
Monitor for alcohol withdrawal and give thiamine
Causes, clinical assessment and management of upper gastrointestinal bleed
Causes of upper GI bleed
Esophageal causes
Esophageal varices
Erosive esophagitis
Esophageal carcinoma
Mallory-Weiss Syndrome
Gastroduodenal Causes
Peptic ulcer
Erosive gastritis
Stress ulcers occurring with shock, sepsis or trauma
Curling ulcers in the proximal duodenum with severe burns or trauma
Carcinoma stomach
Other causes
Coagulation defects
Cameron’s lesions
Hemobilia
Hemosuccus pancreaticus
Clinical assessment
Features suggestive of peptic ulcer → Peptic ulcer disease
Jaundice, pedal edema, abdominal distension, splenomegaly, ascites, dilated
abdominal veins → Esophageal varices
Signs of liver cell failure e.g. spider nevi, palmar erythema, jaundice, gynecomastia →
Esophageal varices
Alcohol or drug ingestion, trauma, burns or sepsis → Gastric erosion
Heavy alcohol intake or vomiting → Mallory Weiss tear
Dysphagia and weight loss prior to bleeding, Tenderness or a mass → Malignant
(stomach or esophageal cancer)
Management
Investigations
General investigations
Complete blood count
Chronic bleeding leads to anemia
Low platelet suggest hematological disorder
Blood urea nitrogen and creatine → BUN increases to greater extent than creatine
because of increased intestinal absorption of urea after breakdown of blood proteins
Liver function tests - Evidence of chronic liver disease
Prothrombin time - Prolonged in patients with chronic liver disease or in coagulation
disorders
Cross-matching - Potential infusion of packed red cells
Specific investigations to find the site of bleed

Treatment
Medical resuscitation and replenishment of intravascular volume
IV access → The first step is to gain IV access using at least one large bore IV catheter
Fluid resuscitation → Adequate resuscitation and hemodynamic stabilization is
essential prior to endoscopy( saline). Vasopressors if persistent hypotension.
Blood products →
if Hb < 7 g/dL → Blood transfusion to maintain Hb at level ≥9 g/dL
if active bleeding and low platelet count (<50,000) → Platelet transfusion
If prolonged prothrombin and INR > 1.5 → FFP
Initial clinical assessment
Circulatory status → Monitor pulse, BP, postural hypotension, urine output and level
of consciousness
Evidence of liver disease → In patients with decompensated cirrhosis
Endoscopy
Pre-endoscopic pharmacotherapy for non-variceal UGIB
IV PPI → Esmoprazole 80 mg bolus, 8 mg/h drip
Suppresses acid, facilitates clot formation and stabilization
Endoscopic hemostasis therapy
Epinephrine injection, fibrin glue, human thrombin, sclerosants
Thermal electrocoagulation, laser therapy
Mechanical hemoclips, banding, endoloops, staples, sutures
Post-endoscopy management for non-variceal UGIB
Patients with ulcers requiring endoscopic therapy should receive PPI x 72 hours
Determine H. pylori status in all ulcer patients.
Discharge patients on PPI (once to twice daily)
In patients with cardiovascular disease on low dose aspirin: Restart as soon as
bleeding has resolved.
Acute esophageal variceal bleeding
HEPATOBILIARY SYSTEM
A 50 year old man, a known case of cirrhosis of liver due to ethanol
consumption is admitted with history of worsening abdominal distension and
abdominal pain. A) Give 3 differential diagnosis for the same. (3 marks) B)
Enumerate the clinical signs of liver cell failure (3 marks) C) Treatment of portal
hypertension and hepatic encephalopathy (2+2= 4 marks)
Differential diagnosis
Ascites
Spontaneous bacterial peritonitis
Acute pancreatitis
Clinical signs of liver cell failure
Alopecia
Icterus
Parotid swelling
Fetor hepaticus
Axillary hair loss
Dupuytren’s contracture
Palmar erythema
Spider nevi
Gynecomastia
Testicular atrophy
Loss of pubic hair
Asterixis
Ankle edema
Treatment of portal hypertension
Reduction of portal pressure
Non-selective beta blockers : propranolol or nadolol
TIPS (Transjugular intrahepatic portosystemic shunt) : Catheter is passed from the
right jugular vein retrograde to the hepatic vein into the liver
Main complication - encephalopathy
Contraindications - Pulmonary hypertension, heart failure, systemic infections,
tricuspid regurgitation
Portosystemic shunt surgeries : If TIPS contraindicated or fails
Portocaval shunt
Splenorenal shunt
Liver transplantation
Treatment of complications
Variceal bleed
Ascites
Encephalopathy
Treatment of hepatic encephalopathy
Diet
Restrict dietary proteins
Zinc supplementation
Maintain good nutrition
Stop alcohol
Increase nitrogen excretion
Lactulose therapy : 15-30 mL three times orally per day. Dose is increased gradually
till there are 2-3 loose stools per day
Lactitol (30 mg daily)
Rifaximin (550 mg BD)
Flumazenil may be effective in some patients
Liver transplantation
Prompt treatment of infections with antibiotics
Four causes of tender enlarged Liver with Differential diagnosis
Causes
Acute Budd-Chiari syndrome → abdominal pain, ascites and hepatomegaly
Pyogenic liver abscess
Amoebic liver abscess
Viral hepatitis
Congestive Cardiac Failure
Malignancies → Hepatocellular carcinoma, secondaries to liver
Causes of obstructive jaundice
Fatty Liver
Abnormal accumulation of triglycerides within cytosol of the parenchymal cells
Causes
Alcoholic fatty liver disease (Alcoholic steatosis)
Non-alcoholic fatty liver disease (Non-alcoholic steatosis)
Drugs : Amiodarone, methotrexate, glucocorticoids, didanosine, zidovudine
Nutritional : Marasmus, TPN, Rapid weight loss/obesity
Metabolic : Diabetes, lipodystrophy, pregnancy
Clinical features
Asymptomatic
Occasionally, discomfort in RUQ, nausea and jaundice
Most common feature : Hepatomegaly
Investigations
AST/ALT
Alcoholic fatty liver disease > 1 (usually > 2)
Non-alcoholic fatty liver disease < 1
GGT : Raised in alcoholic fatty liver disease
ALP : May be raised or may be normal
USG : Hyperechoic
CT : Fatty infiltration produces low density liver
Liver biopsy : Shows accumulation of fat in perivenular hepatocytes and later in entire
hepatic lobule
Treatment
Alcoholic FLD
Complete cessation of alcohol consumption in case of alcoholic fatty liver disease
results in normalization of biochemical findings and histological changes
NAFLD
Weight loss, control of diabetes, hyperlipidemia
Drugs : Metformin, thiazolidinedione (e.g. pioglitazone), liraglutide, atorvastatin,
orlistat
Liver transplantation for end stage cirrhosis
Alcoholic hepatitis
Clinical features
Asymptomatic or presents with fever, rapid onset of jaundice and abdominal
discomfort and proximal muscle wasting
Portal hypertension, spider nevi, ascites and variceal bleeding can occur without
cirrhosis
Tender hepatomegaly and splenomegaly
Investigations
Biochemical findings
AST and ALT raised to 2-7 times of normal (usually < 400 IU)
AST:ALT ratio > 2
Elevated GGT
Raised bilirubin
Mildly elevated ALP
Decreased albumin
Hematological
Increased PT/INR
Leukocytosis
Liver biopsy : Ballooning degeneration of hepatocytes with leukocyte infiltration.
Mallory bodies often seen
Treatment
Stop alcohol consumption for life
Bed rest
Nutrition : Using fine bore nasogastric tube or sometimes IV (> 3000 kCal/day;
multivitamins mainly B and C)
Encephalopathy : Rifaximin and lactulose can be used
Ascites : Loop diuretics, sodium restriction, paracentesis
Corticosteroids may be tried in severe cases (in the absence of any infection)
Antibiotics (pentoxifylline) in severe case and antifungal prophylaxis
Investigations and treatment of non-alcoholic fatty liver disease // Non-alcoholic
steatohepatitis (NASH)
Causes
Obesity, hypertension, type 2 diabetes mellitus, hyperlipidemia and insulin resistance
Rare : Amiodarone, methotrexate, tamoxifen
Clinical features
Asymptomatic
Occasionally, discomfort in RUQ, nausea and jaundice
Most common feature : Hepatomegaly
Investigations
AST/ALT
Non-alcoholic fatty liver disease < 1
GGT : Raised in alcoholic fatty liver disease
ALP : May be raised or may be normal
USG : Hyperechoic
CT : Fatty infiltration produces low density liver
Liver biopsy : Shows accumulation of fat in perivenular hepatocytes and later in entire
hepatic lobule
Treatment
NAFLD
Weight loss, control of diabetes, hyperlipidemia
Drugs : Metformin, thiazolidinedione (e.g. pioglitazone), liraglutide, atorvastatin,
orlistat
Liver transplantation for end stage cirrhosis
Discuss the clinical features, complications and management of hepatic
encephalopathy. Add a note on the precipitating factors for hepatic
encephalopathy // 45 year old chronic alcoholic is admitted with history of
fever and altered sensorium for 2 days. Clinical examination shows deep
jaundice, pallor flapping tremors and moderate ascites with guarding a. What is
the probable diagnosis? b. What other clinical symptoms and signs you elicit to
arrive at clinical diagnosis? c. How do you manage this patient? d. List all other
complications patient can develop
Hepatic encephalopathy or portosystemic encephalopathy is defined as a
neuropsychiatric syndrome (alteration in mental status and cognitive function)
occurring secondary to chronic liver disease. It could be either acute and potentially
reversible, or chronic and progressive
Precipitating Factors
Increased protein load
Increased dietary proteins
GI bleeding
Fluid and electrolytes disturbances due to
Large volume paracentesis
Diuretics
Vomiting/Diarrhea
Portosystemic shunt operations
Acute infections : SBP
Drugs e.g. sedatives, antidepessants
Uremia
HCC or Portal vein thrombosis
Clinical Features
Clinical grading of hepatic encephalopathy
Grade I : Euphoria or depression, mild confusion, slurred speech, disordered sleep
rhythm, asterixis absent
Grade II : Lethargy, moderate confusion, asterixis present
Grade III : Marked confusion, incoherent speech, sleeping but arousable, asterixis
present
Grade IV : Coma, asterixis absent
Complications
Seizures
Permanent neurological deficit
Progressive, irreversible coma
Increased risk of cardiovascular collapse, respiratory failure, sepsis
Death
Management

Treatment of hepatic encephalopathy


Diet
Restrict dietary proteins
Zinc supplementation
Maintain good nutrition
Stop alcohol
Increase nitrogen excretion
Lactulose therapy : 15-30 mL three times orally per day. Dose is increased gradually
till there are 2-3 loose stools per day
Lactitol (30 mg daily)
Rifaximin (550 mg BD)
Flumazenil may be effective in some patients
Liver transplantation
Prompt treatment of infections with antibiotics
Definition and causes of fulminant hepatic failure
Hepatorenal syndrome
VIRAL HEPATITIS
Describe the Etiology, Clinical features, Laboratory features and management
of Viral Hepatitis // Discuss the etiology, clinical features investigations and
management of Acute Viral Hepatitis
Etiology
Hepatitis A, B, C, E
Delta hepatitis (Hepatitis D Virus)
HSV
VZV
EBV
Cytomegalovirus
Adenovirus
Yellow fever virus
Clinical features
Symptomatic pre-icteric phase
Usually lasts for 1-2 weeks
Prodromal symptoms are non-specific
Anorexia, nausea, vomiting, poor appetite
Fatigue, malaise, headache, distaste
Upper vague abdominal pain
Dark urine
Symptomatic Icteric phase
Liver enlarged and tender
Splenomegaly and cervical lymphadenopathy
Dark urine and pale stool
Recovery phase
Appetite improves, jaundice decreases, stools and urine become normal and liver
decreases
Duration of this phase is variable, ranging 2-12 weeks.
Investigations
Urine
Bilirubinuria and increased urinary urobilinogen slight microscopic hematuria and
mild proteinuria
Hematological tests
Leukopenia with a relative lymphocytosis
ESR raised
Biochemical
AST and ALT raised
Bilirubin raised → Both conjugated and unconjugated
ALP raised
Serological tests
Hepatitis A → IgM anti-HAV (acute infection) and IgG anti-HAV
Hepatitis B → HBsAg, HBeAg, anti-HBC IgM (Window period), HBV DNA levels
Hepatitis C → Anti-HCV, HCV-RNA
Hepatitis D → Anti-HDV IgM, HDV RNA
Hepatitis E → Anti-HEV IgM, HEV RNA
Treatment
General measures
Avoid drugs which are metabolized in the liver
Avoid alcohol
No dietary modifications are required
Hepatitis A
No specific treatment
Hepatitis B
Symptomatic treatment
Monitor HBV markers
Entecavir or Tenofovir to be given if HBeAg persists beyond 12 weeks
Hepatitis C
Interferon-alpha used to prevent chronic disease
Hepatitis D
Pegylated Interferon-alpha and Adefovir may be used
Classify hepatitis. Discuss etiopathogenesis, clinical features, complication and
management of chronic viral hepatitis.
Classification

Etiopathogenesis
Immune tolerant phase
Asymptomatic and frequent in children
Active viral replication in liver but little or no evidence of disease activity
HBsAg and HBeAg positive and very high levels of serum HBV DNA
Normal liver function tests.
Liver biopsy shows no inflammation or fibrosis
May lasts for decades
Immune-active phase
Vigorous immune response
Liver biopsy shows chronic hepatitis with moderate or severe necroinflammation and
fibrosis
Evidence of active HBV replication: High levels of HBV DNA and HBeAg
Persistent or intermittent elevation of serum aminotransferases (ALT/AST)
Carrier phase with low replication
Most patients with chronic HBV infection will eventually enter inactive carrier phase
HBsAg positive in the serum >6 months.
HBeAg negative and HBe antibody positive
Undetectable or low levels (below 400 iu/L) of HBV DNA in the serum
Normal aminotransferase (ALT) levels
Liver biopsy does not show any significant hepatitis.
Chronic HBeAg negative state
HBV DNA levels high, liver enzymes are raised and presence of active
histological activity
Patients harbor HBV variants with mutations that prevent production or have low
HBeAg
Clinical features
Asymptomatic or may develop end-stage liver disease
Symptoms: Fatigue, malaise and anorexia
During end-stage liver disease: Symptoms due to complication of cirrhosis
Extrahepatic manifestations: Arthralgias, arthritis, vasculitis, polyarteritis nodosa
Mild hepatomegaly
Long-standing cases may develop hepatocellular carcinoma
Investigations
Serum aminotransferases - Mildly elevated (ALT > AST)
Serum bilirubin - normal or raised
Serum proteins - Hypalbuminemia in severe cases
Prothrombin time - prolonged
Serological markers for Hepatitis B
Positive HBsAg
Positive IgG anti-HBc, negative IgM anti-HBc
Positive HBe antigen or rarely, positive anti-HBe
Positive HBV-DNA
Serological markers for Hepatitis C
HCV antibody
HCV RNA
Treatment of Hepatitis B
Criteria
Serum HBV-DNA above 2000 iu/mL (about >10,000 copies/mL).
Serum ALT level greater than two times normal.
Moderate to severe active necroinflammation and/or fibrosis in the liver biopsy
Current drug of choice → Tenofovir alafenamide 25 mg orally once daily
Actions
HBV DNA < 2000 IU/mL
HBeAg negative (Seroconversion)
HBsAg positive to negative (In only 10% patients)
Treatment of Hepatitis C
Criteria
Detectable HCV RNA or
Cirrhosis or
Decompensated cirrhosis
Drugs used for treatment
Without cirrhosis → Velpatasvir + Sofosbuvir
With cirrhosis → Velpatasvir + Sofosbuvir + Ribavirin
Mention the acute and chronic complications of Hepatitis B virus infection
Acute Complications
Fulminant hepatitis
Progression to chronic hepatitis
Chronic complications
Scarring of the liver (cirrhosis)
Hepatocellular carcinoma
Liver Failure
Co-hepatitis (Hepatitis D)
Polyarteritis nodosa
Encephalopathy
Hepatitis B prevention
Vaccination
Recombinant vaccine is used
Dosage
Number of doses → Three at 0, 1, 6 months or 0, 1, 2 months
Site → IM Deltoid
Dose → 20 mcg (1mL) in Adults (Half in children)
Post exposure prophylaxis

Causes and clinical feature of chronic hepatitis.


Zollinger-Ellison syndrome
Extras
SEROLOGICAL MARKERS FOR HEPATITIS B
PANCREATITIS
In brief discuss causes, clinical features and management of acute pancreatitis
Acute pancreatitis is defined clinically be a patient presenting with two of the three
criteria listed below
Symptoms : Epigastric pain consistent with disease
Lab findings : Elevated amylase/lipase > 3 times the upper limit of normal
Radiological imaging : CT or MRI findings consistent with diagnosis
Causes (I GET PP SMASHED)
I : Idiopathic
G : Gallstones → specifically at Ampulla of Vater
E : Ethanol → alcohol triggers release of pancreatic enzymes
T : Trauma (e.g. seatbelt injury in children)
P : Pancreas divisum
P : Posterior duodenal ulcer rupture
S : Steroids abuse
M : Mumps (other infections are coxsackievirus, mycoplasma pneumoniae)
A : Autoimmune disease (Typically due to IgG4 related disease (+ Riedel's thyroiditis,
retroperitoneal fibrosis, noninfectious aortitis))
S : Scorpion sting
H : Hypertriglyceridemia/Hyperchylomicronemia, Hypercalcemia
E : ERCP
D : Drugs (e.g. sulfa drugs, NRTIs, protease inhibitors, Didanosine, Corticosteroids,
Alcohol, Valproic Acid, Azathioprine, Furosemide, HCTZ, GLP-1 analogs, 6MP)
Clinical features
Symptoms
Severe upper abdominal epigastric pain radiating to the back partially relieved on
stooping and bending forward (Mohammedan Prayer Sign)
Vomiting - frequent and effortless due to reflex pylorospasm
Low grade fever
Hematemesis and malena may occur due to necrosis of duodenum
Signs
Febrile, tachypneic
Cyanosis
Faint jaundice
Features of shock - feeble pulse, tachycardia, hypotension, cold extremities
Abdominal findings
Tenderness in the epigastrium
Upper abdominal guarding and rigidity
Distension of the abdomen
Mass in epigastrium
Cullen’s sign - Bluish ecchymotic discoloration seen around umbilicus
Grey turner’s sign - Bluish discoloration in the flanks
Investigations
CBC → Hb may be low. TLC is raised above 15,000 cells/cu.mm due to inflammation
Blood urea, creatine to rule out renal failure
Serum amylase
Normal → 40-80 Somogyi units
Levels suggestive of acute pancreatitis → >400 units
Levels diagnostic of acute pancreatitis → > 1000 units
Serum lipase - more specific but difficult to measure
Serum calcium levels - Hypocalcemia due to hypoalbuminemia or due to usage of
calcium by the process of saponification
Total proteins are low, especially albumin
Plain X-ray abdomen
Sentinel loop sign - one dilated jejunal loop of intestine which is seen in region of
pancreas
Colon cut-off sign - Mild distension of transverse colon with collapsed descending
colon
Abdominal ultrasound - Edematous pancreas, fluid in the abdomen
Contrast enhanced CT (CECT)
Done after 3-5 days in patient who fail to respond to conservative treatment
If CT scan demonstrates infected necrosis, an urgent CT guided FNAC is done and
the report is sent for gram staining
If it comes out to be positive the patient has to be treated urgently
Treatment
Nil per oral (Bowel rest) + Aspiration with Ryle’s tube
IV fluids - Ringer lactate (preferred)
Analgesic to control pain
No role of empirical antibiotics (given if severe pancreatitis / infected necrosis /
necrosis > 50%)
Antibiotic of choice : Meropenem
Nutrition
Early phase : Total Parenteral nutrition
Start enteral nutrition early (reduces infections)
Naso-jejunal feeding is preferred
Other drugs : PPIs, glucagon and octreotide
Anticoagulation : With LMWH for prevention of DVT
ERCP : No routine use. Indications -
Biliary pancreatitis
Pain > 48 hours
Features of obstruction / jaundice
Patients with documented stones in CBD
Cholecystectomy
Done in all patients with gallstones before discharge
What is the differential diagnosis of a 50 year old alcoholic with severe
abdominal pain Outline the clinical features, complications and treatment of
acute pancreatitis. // Complications of pancreatitis
Rest same as previous question
Complications →
Local
Pseudocyst → It represents a localized peripancreatic collection of pancreatic juice
and debris developing in the lesser sac following inflammatory rupture of the
pancreatic duct.
Pancreatic ascites - May develop due to leakage of fluid from disrupted pancreatic
duct into the peritoneal cavity
Leaking pseudocyst
Necrosis → Necrosis may result in an acute necrotic collection that contains a
variable amount of fluid and necrosis or walled-off necrosis which consists of a
mature, encapsulated collection
Abscess
Pancreatic ascites → Localized collection of pus close to the pancreas and containing
little or no pancreatic tissue
Disruption of main pancreatic duct
Bowel infarction
Massive intraperitoneal hemorrhage
Systemic
Systemic Inflammatory Response Syndrome
Pleural effusion
ARDS
Hypotension and shock
Upper GI bleeding
Obstructive jaundice, splenic or portal vein thrombosis
Oliguria, azotemia, renal artery/vein thrombosis
DIC
55 year old male chronic alcoholic was admitted with acute central abdominal pain.
What is the differential diagnosis and how do you confirm the diagnosis?
Acute Pancreatitis
Investigations
CBC → Hb may be low. TLC is raised above 15,000 cells/cu.mm due to inflammation
Blood urea, creatine to rule out renal failure
Serum amylase
Normal → 40-80 Somogyi units
Levels suggestive of acute pancreatitis → >400 units
Levels diagnostic of acute pancreatitis → > 1000 units
Serum lipase - more specific but difficult to measure
Serum calcium levels - Hypocalcemia due to hypoalbuminemia or due to usage of
calcium by the process of saponification
Total proteins are low, especially albumin
Plain X-ray abdomen
Sentinel loop sign - one dilated jejunal loop of intestine which is seen in region of
pancreas
Colon cut-off sign - Mild distension of transverse colon with collapsed descending
colon
Abdominal ultrasound - Edematous pancreas, fluid in the abdomen
Contrast enhanced CT (CECT)
Done after 3-5 days in patient who fail to respond to conservative treatment
If CT scan demonstrates infected necrosis, an urgent CT guided FNAC is done and
the report is sent for gram staining
If it comes out to be positive the patient has to be treated urgently
Treatment
Nil per oral (Bowel rest) + Aspiration with Ryle’s tube
IV fluids - Ringer lactate (preferred)
Analgesic to control pain
No role of empirical antibiotics (given if severe pancreatitis / infected necrosis /
necrosis > 50%)
Antibiotic of choice : Meropenem
Nutrition
Early phase : Total Parenteral nutrition
Start enteral nutrition early (reduces infections)
Naso-jejunal feeding is preferred
Other drugs : PPIs, glucagon and octreotide
Anticoagulation : With LMWH for prevention of DVT
ERCP : No routine use. Indications -
Biliary pancreatitis
Pain > 48 hours
Features of obstruction / jaundice
Patients with documented stones in CBD
Cholecystectomy
Done in all patients with gallstones before discharge
A 41 year old female presented with periumbilical abdominal pain radiating to
back since last 2 days. a. What is the most likely diagnosis? b. Describe the
etiology, complications and management of the condition
Acute pancreatitis
Causes (I GET PP SMASHED)
I : Idiopathic
G : Gallstones → specifically at Ampulla of Vater
E : Ethanol → alcohol triggers release of pancreatic enzymes
T : Trauma (e.g. seatbelt injury in children)
P : Pancreas divisum
P : Posterior duodenal ulcer rupture
S : Steroids abuse
M : Mumps (other infections are coxsackievirus, mycoplasma pneumoniae)
A : Autoimmune disease (Typically due to IgG4 related disease (+ Riedel's thyroiditis,
retroperitoneal fibrosis, noninfectious aortitis))
S : Scorpion sting
H : Hypertriglyceridemia/Hyperchylomicronemia, Hypercalcemia
E : ERCP
D : Drugs (e.g. sulfa drugs, NRTIs, protease inhibitors, Didanosine, Corticosteroids,
Alcohol, Valproic Acid, Azathioprine, Furosemide, HCTZ, GLP-1 analogs, 6MP)
Complications
Local
Pseudocyst
Leaking pseudocyst
Necrosis
Abscess
Pancreatic ascites
Disruption of main pancreatic duct
Bowel infarction
Massive intraperitoneal hemorrhage
Systemic
Systemic Inflammatory Response Syndrome
Pleural effusion
ARDS
Hypotension and shock
Upper GI bleeding
Obstructive jaundice, splenic or portal vein thrombosis
Oliguria, azotemia, renal artery/vein thrombosis
DIC
Investigations
CBC → Hb may be low. TLC is raised above 15,000 cells/cu.mm due to inflammation
Blood urea, creatine to rule out renal failure
Serum amylase
Normal → 40-80 Somogyi units
Levels suggestive of acute pancreatitis → >400 units
Levels diagnostic of acute pancreatitis → > 1000 units
Serum lipase - more specific but difficult to measure
Serum calcium levels - Hypocalcemia due to hypoalbuminemia or due to usage of
calcium by the process of saponification
Total proteins are low, especially albumin
Plain X-ray abdomen
Sentinel loop sign - one dilated jejunal loop of intestine which is seen in region of
pancreas
Colon cut-off sign - Mild distension of transverse colon with collapsed descending
colon
Abdominal ultrasound - Edematous pancreas, fluid in the abdomen
Contrast enhanced CT (CECT)
Done after 3-5 days in patient who fail to respond to conservative treatment
If CT scan demonstrates infected necrosis, an urgent CT guided FNAC is done and
the report is sent for gram staining
If it comes out to be positive the patient has to be treated urgently
Treatment
Nil per oral (Bowel rest) + Aspiration with Ryle’s tube
IV fluids - Ringer lactate (preferred)
Analgesic to control pain
No role of empirical antibiotics (given if severe pancreatitis / infected necrosis /
necrosis > 50%)
Antibiotic of choice : Meropenem
Nutrition
Early phase : Total Parenteral nutrition
Start enteral nutrition early (reduces infections)
Naso-jejunal feeding is preferred
Other drugs : PPIs, glucagon and octreotide
Anticoagulation : With LMWH for prevention of DVT
ERCP : No routine use. Indications -
Biliary pancreatitis
Pain > 48 hours
Features of obstruction / jaundice
Patients with documented stones in CBD
Cholecystectomy
Done in all patients with gallstones before discharge
Mention the causes and discuss the differential diagnosis of sudden onset of
Hematemesis in a 60 year old male. What are the complications of Pancreatitis?
Peptic ulcer disease: Hematemesis can be caused by a peptic ulcer, which is a sore in
the lining of the stomach or small intestine. Ulcers can be caused by the bacterium
Helicobacter pylori or by the use of nonsteroidal anti-inflammatory drugs (NSAIDs).
Esophageal varices: Esophageal varices are swollen veins in the esophagus that can
burst and cause bleeding. They are often a complication of liver disease or portal
hypertension.
Gastritis: Gastritis is inflammation of the stomach lining that can cause bleeding. It
can be caused by H. pylori infection, alcohol abuse, or the use of certain medications.
Mallory-Weiss tear: A Mallory-Weiss tear is a tear in the lining of the esophagus or
stomach that can cause bleeding. It can be caused by severe vomiting or coughing.
Other causes: Other possible causes of hematemesis include cancer of the digestive
system, blood clotting disorders, and trauma.
Complications
Local
Pseudocyst
Leaking pseudocyst
Necrosis
Abscess
Pancreatic ascites
Disruption of main pancreatic duct
Bowel infarction
Massive intraperitoneal hemorrhage
Systemic
Systemic Inflammatory Response Syndrome
Pleural effusion
ARDS
Hypotension and shock
Upper GI bleeding
Obstructive jaundice, splenic or portal vein thrombosis
Oliguria, azotemia, renal artery/vein thrombosis
DIC
Describe the etiology, clinical features, complications and treatment of acute
pancreatitis
Causes (I GET PP SMASHED)
I : Idiopathic
G : Gallstones → specifically at Ampulla of Vater
E : Ethanol → alcohol triggers release of pancreatic enzymes
T : Trauma (e.g. seatbelt injury in children)
P : Pancreas divisum
P : Posterior duodenal ulcer rupture
S : Steroids abuse
M : Mumps (other infections are coxsackievirus, mycoplasma pneumoniae)
A : Autoimmune disease (Typically due to IgG4 related disease (+ Riedel's thyroiditis,
retroperitoneal fibrosis, noninfectious aortitis))
S : Scorpion sting
H : Hypertriglyceridemia/Hyperchylomicronemia, Hypercalcemia
E : ERCP
D : Drugs (e.g. sulfa drugs, NRTIs, protease inhibitors, Didanosine, Corticosteroids,
Alcohol, Valproic Acid, Azathioprine, Furosemide, HCTZ, GLP-1 analogs, 6MP)
Complications
Local
Pseudocyst → It represents a localized peripancreatic collection of pancreatic juice
and debris developing in the lesser sac following inflammatory rupture of the
pancreatic duct.
Pancreatic ascites - May develop due to leakage of fluid from disrupted pancreatic
duct into the peritoneal cavity
Leaking pseudocyst
Necrosis → Necrosis may result in an acute necrotic collection that contains a
variable amount of fluid and necrosis or walled-off necrosis which consists of a
mature, encapsulated collection
Abscess
Pancreatic ascites → Localized collection of pus close to the pancreas and containing
little or no pancreatic tissue
Disruption of main pancreatic duct
Bowel infarction
Massive intraperitoneal hemorrhage
Systemic
Systemic Inflammatory Response Syndrome
Pleural effusion
ARDS
Hypotension and shock
Upper GI bleeding
Obstructive jaundice, splenic or portal vein thrombosis
Oliguria, azotemia, renal artery/vein thrombosis
DIC
Investigations
CBC → Hb may be low. TLC is raised above 15,000 cells/cu.mm due to inflammation
Blood urea, creatine to rule out renal failure
Serum amylase
Normal → 40-80 Somogyi units
Levels suggestive of acute pancreatitis → >400 units
Levels diagnostic of acute pancreatitis → > 1000 units
Serum lipase - more specific but difficult to measure
Serum calcium levels - Hypocalcemia due to hypoalbuminemia or due to usage of
calcium by the process of saponification
Total proteins are low, especially albumin
Plain X-ray abdomen
Sentinel loop sign - one dilated jejunal loop of intestine which is seen in region of
pancreas
Colon cut-off sign - Mild distension of transverse colon with collapsed descending
colon
Abdominal ultrasound - Edematous pancreas, fluid in the abdomen
Contrast enhanced CT (CECT)
Done after 3-5 days in patient who fail to respond to conservative treatment
If CT scan demonstrates infected necrosis, an urgent CT guided FNAC is done and
the report is sent for gram staining
If it comes out to be positive the patient has to be treated urgently
Treatment
Nil per oral (Bowel rest) + Aspiration with Ryle’s tube
IV fluids - Ringer lactate (preferred)
Analgesic to control pain
No role of empirical antibiotics (given if severe pancreatitis / infected necrosis /
necrosis > 50%)
Antibiotic of choice : Meropenem
Nutrition
Early phase : Total Parenteral nutrition
Start enteral nutrition early (reduces infections)
Naso-jejunal feeding is preferred
Other drugs : PPIs, glucagon and octreotide
Anticoagulation : With LMWH for prevention of DVT
ERCP : No routine use. Indications -
Biliary pancreatitis
Pain > 48 hours
Features of obstruction / jaundice
Patients with documented stones in CBD
Cholecystectomy
Done in all patients with gallstones before discharge
Patient comes h/o (history) acute onset of vomiting with severe upper backache,
vomiting aggravates on taking food. Pain is better on lying on the abdomen. Patient
is an alcoholic. Serum Creatine : 1.5 mg/dL, Urea : 50 mg/dL; Amylase : 400 mL.
Outline the approach to this patient
Causes of acute pancreatitis // Etiology of acute pancreatitis
Causes (I GET PP SMASHED)
I : Idiopathic
G : Gallstones → specifically at Ampulla of Vater
E : Ethanol → alcohol triggers release of pancreatic enzymes
T : Trauma (e.g. seatbelt injury in children)
P : Pancreas divisum
P : Posterior duodenal ulcer rupture
S : Steroids abuse
M : Mumps (other infections are coxsackievirus, mycoplasma pneumoniae)
A : Autoimmune disease (Typically due to IgG4 related disease (+ Riedel's thyroiditis,
retroperitoneal fibrosis, noninfectious aortitis))
S : Scorpion sting
H : Hypertriglyceridemia/Hyperchylomicronemia, Hypercalcemia
E : ERCP
D : Drugs (e.g. sulfa drugs, NRTIs, protease inhibitors, Didanosine, Corticosteroids,
Alcohol, Valproic Acid, Azathioprine, Furosemide, HCTZ, GLP-1 analogs, 6MP)
Explain 4 complications acute pancreatitis // Complications of acute pancreatitis (x2)
Local
Pseudocyst → It represents a localized peripancreatic collection of pancreatic juice
and debris developing in the lesser sac following inflammatory rupture of the
pancreatic duct.
Pancreatic ascites - May develop due to leakage of fluid from disrupted pancreatic
duct into the peritoneal cavity
Leaking pseudocyst
Necrosis → Necrosis may result in an acute necrotic collection that contains a
variable amount of fluid and necrosis or walled-off necrosis which consists of a
mature, encapsulated collection
Abscess
Pancreatic ascites → Localized collection of pus close to the pancreas and containing
little or no pancreatic tissue
Disruption of main pancreatic duct
Bowel infarction
Massive intraperitoneal hemorrhage
Systemic
Systemic Inflammatory Response Syndrome
Pleural effusion
ARDS
Hypotension and shock
Upper GI bleeding
Obstructive jaundice, splenic or portal vein thrombosis
Oliguria, azotemia, renal artery/vein thrombosis
DIC
Management of acute pancreatitis
Investigations
CBC → Hb may be low. TLC is raised above 15,000 cells/cu.mm due to inflammation
Blood urea, creatine to rule out renal failure
Serum amylase
Normal → 40-80 Somogyi units
Levels suggestive of acute pancreatitis → >400 units
Levels diagnostic of acute pancreatitis → > 1000 units
Serum lipase - more specific but difficult to measure
Serum calcium levels - Hypocalcemia due to hypoalbuminemia or due to usage of
calcium by the process of saponification
Total proteins are low, especially albumin
Plain X-ray abdomen
Sentinel loop sign - one dilated jejunal loop of intestine which is seen in region of
pancreas
Colon cut-off sign - Mild distension of transverse colon with collapsed descending
colon
Abdominal ultrasound - Edematous pancreas, fluid in the abdomen
Contrast enhanced CT (CECT)
Done after 3-5 days in patient who fail to respond to conservative treatment
If CT scan demonstrates infected necrosis, an urgent CT guided FNAC is done and
the report is sent for gram staining
If it comes out to be positive the patient has to be treated urgently
Treatment
Nil per oral (Bowel rest) + Aspiration with Ryle’s tube
IV fluids - Ringer lactate (preferred)
Analgesic to control pain
No role of empirical antibiotics (given if severe pancreatitis / infected necrosis /
necrosis > 50%)
Antibiotic of choice : Meropenem
Nutrition
Early phase : Total Parenteral nutrition
Start enteral nutrition early (reduces infections)
Naso-jejunal feeding is preferred
Other drugs : PPIs, glucagon and octreotide
Anticoagulation : With LMWH for prevention of DVT
ERCP : No routine use. Indications -
Biliary pancreatitis
Pain > 48 hours
Features of obstruction / jaundice
Patients with documented stones in CBD
Cholecystectomy
Done in all patients with gallstones before discharge
Causes of recurrent and chronic pancreatitis
Clinical features of chronic pancreatitis. // Clinical presentations of chronic calcific
pancreatitis
Abdominal pain
Continuous or intermittent
May be referred to the back
Increased on eating heavy meal
Pain may be increased or decreased by taking alcohol
Relieved on stooping forwards
Features of malabsorption - Chronic diarrhea, steatorrhea, weight loss, fatigue
Diabetes mellitus in about 30% patients
Jaundice in later stages
On physical examination - thin, malnourished patient with tenderness in epigastric
region
Indications for Endoscopic retrograde cholangiopancreatography (ERCP)

IRRITABLE BOWEL SYNDROME


What are the criteria for diagnosis of irritable bowel syndrome, and what drugs can
be used to treat diarrhea in IBS?
INFLAMMATORY BOWEL DISEASE
Extra-intestinal complications of inflammatory bowel disease

Ulcerative colitis // Clinical features of ulcerative colitis


Clinical features
More common in females. Female : Male ratio is 2:1.
Age: Common age of presentation is 3rd decade followed by 4th and 2nd decades.
Types (based on severity):
Mild colitis : Refers to < 4 stools/day without systemic signs and symptoms
Moderate colitis : Refers to < 6 stools/day without systemic toxicity
Severe colitis : Refers to > 6 stools/day with systemic toxicity —
Fever > 37.5 C
Pulse rate > 90 /min
Hypoalbuminemia < 3 g/dL
Weight loss > 3 kg
Tenesmus : Due to loss of elasticity of rectum → lumen collapses
Late features: Severe dehydration, malnutrition, anemia, hypoproteinemia
Acute attack : High grade fever, bloody dysentery, distension and tenderness all over
the abdomen.
Investigations
Stool
To rule out various causes of infective diarrhea
Sigmoidoscopy
Inflammatory changes in mucosa
Mucus, pus and blood are visible
Multiple ulcers are visible with bleeding
Barium enema
Contracted colon / pipe stem colon
Absence of haustrations and mucosal irregularity
Pseudopolyposis - stippled apperance
Retrorectal space is increased
Colonoscopy
To confirm diagnosis by biopsy
To rule out carcinomatous changes
Plain X-ray
To rule out megacolon and perforation
C-reactive protein
High in case of acute attack
Electrolytes, albumin levels are low
Treatment
Conservative
Hospitalization and bed rest
Correction of fluids and electrolytes
Blood transfusion to correct anemia and TPN for hypoproteinemia
Sulfasalazine at a dose of 2 g/day.
MOA → Splits into 5-ASA and sulfapyridine. Suppresses activity of prostaglandins E1
and E2 and thus reduces inflammation.
Corticosteroids :
Less severe cases not responding to sulfasalazine are given a trial of oral
prednisolone 60 mg/day. They decrease the frequency of stools. The dose is tapered
off over 3-4 weeks.
In acute attacks IV hydrocortisone 100 mg is used
Immunosuppressive agents - Azathioprine, 6-MP, mycophenolate mofetil,
methotrexate, cyclosporine (4 mg/kg/day)
TNF-alpha inhibitors
Adalimumab, Infliximab can be used
Surgery
Indications
Complications—toxic megacolon, perforation.
Active disease in spite of medical line of management
Severe disease not responding to medical treatment
Dysplasia on biopsy
Steroid dependence
Hemorrhage
Procedures
Restorative proctocolectomy with ileal pouch
This can be done as one- or two-stage procedure.
Total proctocolectomy is done first.
A mucosectomy of the upper anal canal is done.
A pouch is created by anastomosing the loops of ileum. A J-shaped pouch is the
most popular followed by W pouch
The pouch is anastomosed to the dentate line (junction of upper and lower anal
canal) by using stapler or by hand sutures.
Protective ileostomy is done and it can be closed after two months.
Total proctocolectomy followed by permanent ileostomy. Ileostomy is connected to
ileostomy bag. This is the procedure with least complications
Matthew
Discuss etiology, clinical manifestations, complications and management of
Crohn’s disease
Etiology of IBD
Smoking : Protective for UC but risk factor for CD
Psychosocial factors : Major life events such as illness or death in family, divorce or
separation, interpersonal conflict or other major loss are risk are associated with
increased risk of IBD
Intestinal microflora : Characterized be over-aggressive response to luminal bacterial
agents and other products
Nutritional factors : High sugar and fat intake is suspected to associated with ICD
Appendicectomy : Protective for UC but risk factor for CD
Genetic and familial factors
Environmental factors : Good domestic hygiene has been shown to be a risk factor
for CD.
Immunological factors
Inability to down regulate immune or antigen non-specific inflammatory responses
to endogenous luminal antigens
Upregulation of macrophages and T-helper cells in IBD which release pro-
inflammatory cytokines.
Clinical features
Chronic disease with remissions and exacerbation
Major symptoms : Diarrhea (may be associated with blood), abdominal pain and
weight loss
Constitutional symptoms : Malaise, lethargy, anorexia, nausea, vomiting and low
grade fever
Can be complicated by anal and perianal disease : Anal fissures or perianal abscess
Enteric fistula e.g. from intestine to bladder or vagina
Examination
Right iliac fossa tenderness and mass may be present
Investigations
CBP : Anemia due to blood loss, Increased WBC
Hypoalbuminemia
Raised ESR and CRP
Barium meal follow-through -
asymmetric alteration in mucosal patter with deep ulceration
Areas of narrowing and stricturing (String sign)
Skip lesions with normal bowel in between
USG/CT abdomen
To determine the thickness of bowel wall and mesentery
To check for intra-abdominal and para-intestinal abscess
To rule out other pathologies
Colonoscopy
Superficial and deep ulceration with cobblestone appearance
Skip lesions
Rectal sparing
Colonic biopsy : Confirmation of diagnosis
Crypt abscesses
Branching of crypts
Atrophy of glands
Treatment
General measures
Cigarette smoking to be stopped
Diarrhea control : Loperamide 2-4 mg QID
Anemia correction : B12/Folic acid/Iron supplementation
5-ASA agents
Induce and maintain remission
Sulfasalazine is the commonly used drug
Folic acid supplementation to be given along
Newer drugs : Mesalamine, osalazine and balsalazide
Glucocorticoids
Can be used in acute exacerbation and have no role in maintenance therapy
Prednisolone 60 mg/day can be used
Immunosuppressive agents
Azathioprine, 6-MP, methotrexate, cyclosporine, mycophenolate mofetil and
tacrolimus
Nutritional therapies
Bowel rest and TPN can be used in active CD to induce remission
TNF-alpha inhibitors
Drugs : Infliximab, Certolizumab, Adalimumab, Golimumab, Etanercept
Surgical management
Indicated in stricture and obstruction unresponsive to medical therapy
Clinical Manifestations of Crohn’s Disease
Chronic disease with remissions and exacerbation
Major symptoms : Diarrhea (may be associated with blood), abdominal pain and
weight loss
Constitutional symptoms : Malaise, lethargy, anorexia, nausea, vomiting and low
grade fever
Can be complicated by anal and perianal disease : Anal fissures or perianal abscess
Enteric fistula e.g. from intestine to bladder or vagina
Examination
Right iliac fossa tenderness and mass may be present
Extras

H. PYLORI
Spectrum of H. pylori induced disease. Mention four drugs useful in the treatment of
H. pylori infection.
Asymptomatic infection: Some people who are infected with H. pylori do not
experience any symptoms.
Non-ulcer dyspepsia: H. pylori infection can cause symptoms such as abdominal
pain, bloating, and nausea, but does not cause ulcers. This is sometimes referred to
as non-ulcer dyspepsia.
Peptic ulcer disease: H. pylori infection is a major cause of peptic ulcer disease, which
is characterized by sores in the lining of the stomach or small intestine. Ulcers can
cause symptoms such as abdominal pain, nausea, and weight loss.
Gastric cancer: H. pylori infection can increase the risk of developing gastric cancer,
which is cancer of the stomach.
Other complications: H. pylori infection can also cause complications such as gastric
bleeding, perforation of the stomach or duodenum, and gastric outlet obstruction.
Treatment of H. pylori infection
General measures
Avoid cigarette smoking, aspirin and NSAIDs
Alcohol to be avoided
Triple therapy (BD daily for 10-14 days)
Clarithromycin 500 mg
Amoxicillin 1000 mg / Metronidazole 500 mg
PPI (Pantoprazole 40 mg)
Quadruple therapy (14 days)
Tetracycline 500 mg
Omeprazole 20 mg
Metronidazole 500 mg
Colloidal bismuth solution 525 mg
Management of H. pylori infection (x2)
General measures
Avoid cigarette smoking, aspirin and NSAIDs
Alcohol to be avoided
Triple therapy (BD daily for 10-14 days)
Clarithromycin 500 mg
Amoxicillin 1000 mg / Metronidazole 500 mg
PPI (Pantoprazole 40 mg)
Quadruple therapy (14 days)
Tetracycline 500 mg QID
Omeprazole 20 mg BD
Metronidazole 500 mg QID
Colloidal bismuth solution 525 mg QID
Atrophic gastritis (x2)
Most common cause : H. pylori infection
Other causes : smoking, spicy food, continuous ingestion of drugs, reflux of bile into
stomach
Most commonly involves the antrum of stomach
Loss of cells, atrophy and mucosal thinning (metaplasia)
Symptoms
Epigastric pain
Dyspepsia
Early satiety
Investigations
Esophagoduodenoscopy
Biopsy
Peripheral smear : Megaloblastic anemia
Serum vitamin B12 levels : decreased
Schilling’s test : To test vitamin B12 absorption
Treatment
PPIs : Omeprazole (20-40 mg/day), lansoprazole (15-30 mg/day)
H2 blockers : Cimetidine (400 mg QID), Ranitidine (150 mg QID)
OTHERS
Diagnosis of coeliac disease // Celiac sprue
Inflammatory condition of small intestine precipitated by the ingestion of gluten
containing foods (wheat, barley, rye) in individuals with genetic predisposition
Etiology
Inflammatory damage to intestinal mucosa is due to gluten protein. The toxic
component in gluten is gliadin
HLA-DQ2 association
Pathogenesis
Glutens are partially digested in small intestine to related gliadin and other peptides
Gliadin → Contains glutamine → Deamidated by tissue transglutaminase (tTG) →
Glutamic acid residues → recognized by local intestinal T cells as foreign → Immune
response → Antibodies formed such as antigliadin, anti-endomysial, and anti-tTG →
Damage to intestinal mucosa resulting in maldigestion and malabsorption
Clinical features
Usually occurs in infancy after weaning to gluten containing foods
Diarrhea, abdominal distension and bloating after eating
Weight loss
Growth retardation
Features of vitamin and mineral deficiencies
Increased incidence of other autoimmune disorders like thyroid disease, type I
diabetes, primary biliary cirrhosis, and Sjogren’s syndrome
Extraintestinal manifestations
Rash (dermatitis herpetiformis)
Neurological disorders (myopathy, epilepsy)
Psychiatric disorders (depression)
Reproductive disorders (infertility, spontaneous abortion)
Investigations
Duodenal / Jejunal biopsy
Jejunum predominantly affected
Absence of villi, making the mucosal surface flat
Histology :
Crypt hyperplasia
Chronic inflammatory cells in the lamina propria
Subtotal villous atrophy
Serological markers : anti-tissue transglutaminase IgA detected, antigliadin IgA and
IgG
Tests for malabsorption
Treatment
Diet :
Lifelong gluten free diet (wheat, rye and barley)
Lactose free diet is advisable because of secondary lactase deficiency
Supplementation of vitamins and minerals in cases of patients with these deficiency
Pneumococcal vaccination (because of splenic atrophy) every 5 years
Steroids may be helpful in refractory cases
Parenteral nutrition
Chronic diarrhea
Mention four causes of diarrhea with blood and mucus. Mention the treatment of
Amoebic dysentery.
IBD : Ulcerative colitis and Crohn’s Disease
Bacterial infections : Salmonella, E. coli
Colorectal cancer
Diverticulitis
Amebic dysentery
Clinical features
Could be asymptomatic
Dysentery
Lower abdominal pain, malaise and mild diarrhea
Pass up to 10-12 stools per day
Severe infections : Toxic megacolon
Extraintestinal manifestations (Liver disease)
RUQ pain : Dull or pleuritic in nature
Fever
Investigations
Stool examination : Trophozoites visualized
Stool culture
Colonoscopy : Flask shaped ulcers
USG : To identify amebic liver abscess
Aspiration of abscess : Anchovy sauce pus
Serological tests : ELISA and indirect hemagglutination
Treatment
Metronidazole 400 mg TID x 7 days
Newer agents : Tinidazole, secnidazole or ornidazole are equally effective
Luminal amebicides : Diloxanide furoate, iodoquinol, or paromomycin
Aspiration of liver abscess using pigtail catheter
Wilson's disease
Mention four diseases caused by smoking, and four caused by alcohol
Four diseases caused by smoking
Lung cancer
COPD
Hypertension
Esophageal cancer
Four diseases caused by alcohol
Alcoholic liver disease
Pancreatitis
Oral cancer
Wernicke-Korsakoff Syndrome
Extras
Tropical Sprue
Chronic diarrheal disease possibly of infectious origin, that involves the small
intestine and is characterized by malabsorption of nutrients, especially folic acid and
vitamin B12
Etiology
Likely to be due to infectious agent because it responds to antibiotics
Implicated bacteria → E. coli, Klebsiella, Enterobacter
Clinical features
Anorexia
Abdominal distension
Weight loss
Investigations
Endoscopy and mucosal biopsy
Flattening of duodenal folds and scalloping
Jejunal biopsy shows partial villous atrophy (less severe than celiac disease)
Exclude other causes of diarrhea particularly Giardia which can mimic tropical sprue
Treatment
Broad spectrum antibiotics and folic acid can cure the condition particularly if the
patient leaves the tropical area and does not return
Antibiotic used : Tetracycline 1 g daily x 6 months
Whipple’s Disease
Chronic multisystem disease caused by gram-positive bacteria Tropheryma whippelii
Disease affects gastrointestinal tract, joints, central nervous system and
cardiovascular system
Clinical features
Insidious onset
Diarrhea
Steatorrhea
Abdominal pain
Weight loss
Migratory large-joint arthropathy
Fever
Dementia
Ophthalmologic symptoms
Culture negative endocarditis
Investigations
Small intestine biopsy (and other organs) : PAS positive macrophages containing the
small bacilli
Treatment
TMP-SMX double strength x 1 year
Alternatives : penicillin and chloramphenicol

Nervous System Disorders


MENINGITIS
A 45 year old patient was admitted with fever of 2 weeks duration. Since one week
he has head ache and vomiting. On examination he has neck stiffness and
papilledema. How will you approach this patient?
History
Ask about the following symptoms -
Fever duration, severity, temperature measurements (if taken)
Mental status : lethargy, personality changes etc.
Headache : type of headache, duration
Vomiting
Others : Photophobia, seizures, focal neurological deficits (cranial nerves, motor
disturbance)
Examination
Examination of cranial nerves, sensory system and motor system. Specific signs to be
elicited :
Brudzinski’s neck sign : Passive neck flexion, while patient is in supine position,
produces involuntary flexion of hips and knees
Brudzinski’s leg sign : Passive flexion of one leg at the hip joint produces automatic
flexion of other leg
Kernig’s sign : Extension of knee from flexed thigh position causes passive resistance.
This is due to the spasm of hamstring muscles due to the inflamed sciatic nerve as it
passes through the spinal theca
Papilledema
Investigations
Blood counts : WBC elevated
Blood cultures : May help to identify the causative organism in most cases of
meningitis
Serum procalcitonin : Elevated in bacterial meningitis.
Lumbar puncture and CSF analysis : Test of choice to diagnose meningitis. Every
patient should have LP done unless the procedure is contraindicated. CSF patient
should be sent for sugar content, cell type, Gram’s stain, India
CT scan head : A contrast CT shows meningeal enhancement in meningitis. It is also
helpful to rule out other pathologies such as subarachnoid hemorrhage, cerebral
abscess, mass lesion, middle ear and sinus disease.
Treatment
Bacterial meningitis is a medical emergency and treatment should be initiated
immediately as soon as it is suspected.
There are two general principles of antibiotic therapy:
Use of bactericidal drugs effective against the infecting organism
Use of drugs that enter CSF
Empirical antibiotic therapy
Antibiotics can be chosen based on the common organisms in the particular age
group
Neonates and infants
Common organisms → E. coli, GBS, L. monocytogenes, S. pneumoniae
Antibiotics → Ampicillin (L. monocytogenes) + Cefotaxime/Ceftriaxone
Children < 12 years
Common organisms → N. meningitidis, S. pneumoniae, H. influenzae
Antibiotics → Vancomycin + Cefotaxime/Ceftriaxone
Adults < 60 years
Common organisms → S. pneumoniae, N. meningitidis, H. influenzae, GBS
Antibiotics → Vancomycin + Cefotaxime/Ceftriaxone
Adults > 60 years
Common organisms → S. pneumoniae, L. monocytogenes, N. meningitidis, H.
influenzae, GBS
Antibiotics → Ampicillin + Cefotaxime/Ceftriaxone + Vancomycin
Organism specific antibiotic therapy
S. pneumoniae
Ceftriaxone (2 g every 12 hour) + Vancomycin (500 mg every 6 hours) for 14 days
N. meningitidis
Penicillin G (4 million units every 4 hours) for 7 days
GBS
Penicillin G (4 million units every 4 hours) for 2-3 weeks
H. influenzae
Ceftriaxone (2 g every 12 hour) for 7 days
Enterobacteriaceae
Ceftriaxone (2 g every 12 hour) + Gentamicin (1-2 mg/kg every 8 hours) for 3 weeks
Pseudomonas / Acinetobacter
Ceftazidime (2 g every 8 hours) + Gentamicin (1-2 mg/kg every 8 hour) for 3 weeks
Role of steroids
Trials have shown that dexamethasone given shortly before or at the same time as
first dose of antibiotics significantly improves outcome in patients
It reduces CSF synthesis of cytokines, CSF inflammation and cerebral edema
Treatment for raised ICP
Elevation of head end of the bed to 30 to 45 degrees, hyperventilation and
administration of mannitol
How does lumbar puncture help to differentiate between different etiologies of
meningitis?
35 male presented with history of headache and fever since 5 days. On
examination patient is drowsy and has neck stiffness. Mention possible
diagnosis. Outline etiology, clinical features, investigations and management of
above diagnosis.
Differential diagnosis -
Acute pyogenic meningitis
Aseptic meningitis
Etiology
Pyogenic meningitis -
Neonates and infants
Common organisms → E. coli, GBS, L. monocytogenes, S. pneumoniae
Children < 12 years
Common organisms → N. meningitidis, S. pneumoniae, H. influenzae
Adults < 60 years
Common organisms → S. pneumoniae, N. meningitidis, H. influenzae, GBS
Adults > 60 years
Common organisms → S. pneumoniae, L. monocytogenes, N. meningitidis, H.
influenzae, GBS
Aseptic meningitis -
Enteroviruses → Coxsackie A/B, Poliovirus, Echovirus
Mumps virus
Herpes Simplex Virus
HIV
Clinical features
Symptoms -
Triad : Fever, headache, nuchal rigidity
Nausea, vomiting and photophobia
Seizures/convulsion
Cranial nerve palsies with visual field defects, dysphasia and hemiparesis
Meningococcal meningitis : Maculopapular rash that become petechial, found on
trunk, lower limbs, mucous membranes, conjunctiva
Raised ICP :
Cushing reflex : Bradycardia, hypertension and irregular respiration
Papilledema
Dilated poorly reactive pupil
6th nerve palsy
Decerebrate posture
Signs -
Brudzinski’s neck sign : Passive neck flexion, while patient is in supine position,
produces involuntary flexion of hips and knees
Brudzinski’s leg sign : Passive flexion of one leg at the hip joint produces automatic
flexion of other leg
Kernig’s sign : Extension of knee from flexed thigh position causes passive resistance.
This is due to the spasm of hamstring muscles due to the inflamed sciatic nerve as it
passes through the spinal theca
Papilledema
Investigations
Blood counts : WBC elevated
Blood cultures : May help to identify the causative organism in most cases of
meningitis
Serum procalcitonin : Elevated in bacterial meningitis.
Lumbar puncture and CSF analysis : Test of choice to diagnose meningitis. Every
patient should have LP done unless the procedure is contraindicated. CSF patient
should be sent for sugar content, cell type, Gram’s stain, India
CT scan head : A contrast CT shows meningeal enhancement in meningitis. It is also
helpful to rule out other pathologies such as subarachnoid hemorrhage, cerebral
abscess, mass lesion, middle ear and sinus disease.
Treatment
Bacterial meningitis is a medical emergency and treatment should be initiated
immediately as soon as it is suspected.
There are two general principles of antibiotic therapy:
Use of bactericidal drugs effective against the infecting organism
Use of drugs that enter CSF
Empirical antibiotic therapy
Antibiotics can be chosen based on the common organisms in the particular age
group
Neonates and infants
Common organisms → E. coli, GBS, L. monocytogenes, S. pneumoniae
Antibiotics → Ampicillin (L. monocytogenes) + Cefotaxime/Ceftriaxone
Children < 12 years
Common organisms → N. meningitidis, S. pneumoniae, H. influenzae
Antibiotics → Vancomycin + Cefotaxime/Ceftriaxone
Adults < 60 years
Common organisms → S. pneumoniae, N. meningitidis, H. influenzae, GBS
Antibiotics → Vancomycin + Cefotaxime/Ceftriaxone
Neonates and infants
Common organisms → S. pneumoniae, L. monocytogenes, N. meningitidis, H.
influenzae, GBS
Antibiotics → Ampicillin + Cefotaxime/Ceftriaxone + Vancomycin
Organism specific antibiotic therapy
S. pneumoniae
Ceftriaxone (2 g every 12 hour) + Vancomycin (500 mg every 6 hours) for 14 days
N. meningitidis
Penicillin G (4 million units every 4 hours) for 7 days
GBS
Penicillin G (4 million units every 4 hours) for 2-3 weeks
H. influenzae
Ceftriaxone (2 g every 12 hour) for 7 days
Enterobacteriaceae
Ceftriaxone (2 g every 12 hour) + Gentamicin (1-2 mg/kg every 8 hours) for 3 weeks
Pseudomonas / Acinetobacter
Ceftazidime (2 g every 8 hours) + Gentamicin (1-2 mg/kg every 8 hour) for 3 weeks
HSV-1 or HSV-2
Acyclovir
HIV
Start antiretroviral therapy
Role of steroids
Trials have shown that dexamethasone given shortly before or at the same time as
first dose of antibiotics significantly improves outcome in patients
It reduces CSF synthesis of cytokines, CSF inflammation and cerebral edema
Treatment for raised ICP
Elevation of head end of the bed to 30 to 45 degrees, hyperventilation and
administration of mannitol
BACTERIAL MENINGITIS
CSF findings in Bacterial Meningitis // CSF findings in acute bacterial meningitis
Describe the clinical features, investigations and treatment of bacterial
meningitis in a 26 year old patient.
CLINICAL FEATURES
Symptoms -
Triad : Fever, headache, nuchal rigidity
Nausea, vomiting and photophobia
Seizures/convulsion
Cranial nerve palsies with visual field defects, dysphasia and hemiparesis
Meningococcal meningitis : Maculopapular rash that become petechial, found on
trunk, lower limbs, mucous membranes, conjunctiva
Raised ICP :
Cushing reflex : Bradycardia, hypertension and irregular respiration
Papilledema
Dilated poorly reactive pupil
6th nerve palsy
Decerebrate posture
Signs -
Brudzinski’s neck sign : Passive neck flexion, while patient is in supine position,
produces involuntary flexion of hips and knees
Brudzinski’s leg sign : Passive flexion of one leg at the hip joint produces automatic
flexion of other leg
Kernig’s sign : Extension of knee from flexed thigh position causes passive resistance.
This is due to the spasm of hamstring muscles due to the inflamed sciatic nerve as it
passes through the spinal theca
Papilledema
Investigations
Blood counts : WBC elevated
Blood cultures : May help to identify the causative organism in most cases of
meningitis
Serum procalcitonin : Elevated in bacterial meningitis.
Lumbar puncture and CSF analysis : Test of choice to diagnose meningitis. Every
patient should have LP done unless the procedure is contraindicated. CSF patient
should be sent for sugar content, cell type, Gram’s stain, India
CT scan head : A contrast CT shows meningeal enhancement in meningitis. It is also
helpful to rule out other pathologies such as subarachnoid hemorrhage, cerebral
abscess, mass lesion, middle ear and sinus disease.
Treatment
Bacterial meningitis is a medical emergency and treatment should be initiated
immediately as soon as it is suspected.
There are two general principles of antibiotic therapy:
Use of bactericidal drugs effective against the infecting organism
Use of drugs that enter CSF
Empirical antibiotic therapy
Antibiotics can be chosen based on the common organisms in the particular age
group
Neonates and infants
Common organisms → E. coli, GBS, L. monocytogenes, S. pneumoniae
Antibiotics → Ampicillin (L. monocytogenes) + Cefotaxime/Ceftriaxone
Children < 12 years
Common organisms → N. meningitidis, S. pneumoniae, H. influenzae
Antibiotics → Vancomycin + Cefotaxime/Ceftriaxone
Adults < 60 years
Common organisms → S. pneumoniae, N. meningitidis, H. influenzae, GBS
Antibiotics → Vancomycin + Cefotaxime/Ceftriaxone
Neonates and infants
Common organisms → S. pneumoniae, L. monocytogenes, N. meningitidis, H.
influenzae, GBS
Antibiotics → Ampicillin + Cefotaxime/Ceftriaxone + Vancomycin
Organism specific antibiotic therapy
S. pneumoniae
Ceftriaxone (2 g every 12 hour) + Vancomycin (500 mg every 6 hours) for 14 days
N. meningitidis
Penicillin G (4 million units every 4 hours) for 7 days
GBS
Penicillin G (4 million units every 4 hours) for 2-3 weeks
H. influenzae
Ceftriaxone (2 g every 12 hour) for 7 days
Enterobacteriaceae
Ceftriaxone (2 g every 12 hour) + Gentamicin (1-2 mg/kg every 8 hours) for 3 weeks
Pseudomonas / Acinetobacter
Ceftazidime (2 g every 8 hours) + Gentamicin (1-2 mg/kg every 8 hour) for 3 weeks
Role of steroids
Trials have shown that dexamethasone given shortly before or at the same time as
first dose of antibiotics significantly improves outcome in patients
It reduces CSF synthesis of cytokines, CSF inflammation and cerebral edema
Treatment for raised ICP
Elevation of head end of the bed to 30 to 45 degrees, hyperventilation and
administration of mannitol
A 20 year old boy is brought to casualty with history of fever, head ache,
vomiting of 3 weeks duration initially treated in a local hospital. Father also
gives history of altered level of consciousness and deviation of eye ball. a. What
is the most likely diagnosis? b. What clinical signs will you look for? c. How do
you investigate this boy? d. What is the course of treatment?
Acute pyogenic meningitis
Clinical signs-
Brudzinski’s neck sign : Passive neck flexion, while patient is in supine position,
produces involuntary flexion of hips and knees
Brudzinski’s leg sign : Passive flexion of one leg at the hip joint produces automatic
flexion of other leg
Kernig’s sign : Extension of knee from flexed thigh position causes passive resistance.
This is due to the spasm of hamstring muscles due to the inflamed sciatic nerve as it
passes through the spinal theca
Papilledema
Investigations
Blood counts : WBC elevated
Blood cultures : May help to identify the causative organism in most cases of
meningitis
Serum procalcitonin : Elevated in bacterial meningitis.
Lumbar puncture and CSF analysis : Test of choice to diagnose meningitis. Every
patient should have LP done unless the procedure is contraindicated. CSF patient
should be sent for sugar content, cell type, Gram’s stain, India
CT scan head : A contrast CT shows meningeal enhancement in meningitis. It is also
helpful to rule out other pathologies such as subarachnoid hemorrhage, cerebral
abscess, mass lesion, middle ear and sinus disease.
Treatment
Bacterial meningitis is a medical emergency and treatment should be initiated
immediately as soon as it is suspected.
There are two general principles of antibiotic therapy:
Use of bactericidal drugs effective against the infecting organism
Use of drugs that enter CSF
Empirical antibiotic therapy
Antibiotics can be chosen based on the common organisms in the particular age
group
Neonates and infants
Common organisms → E. coli, GBS, L. monocytogenes, S. pneumoniae
Antibiotics → Ampicillin (L. monocytogenes) + Cefotaxime/Ceftriaxone
Children < 12 years
Common organisms → N. meningitidis, S. pneumoniae, H. influenzae
Antibiotics → Vancomycin + Cefotaxime/Ceftriaxone
Adults < 60 years
Common organisms → S. pneumoniae, N. meningitidis, H. influenzae, GBS
Antibiotics → Vancomycin + Cefotaxime/Ceftriaxone
Neonates and infants
Common organisms → S. pneumoniae, L. monocytogenes, N. meningitidis, H.
influenzae, GBS
Antibiotics → Ampicillin + Cefotaxime/Ceftriaxone + Vancomycin
Organism specific antibiotic therapy
S. pneumoniae
Ceftriaxone (2 g every 12 hour) + Vancomycin (500 mg every 6 hours) for 14 days
N. meningitidis
Penicillin G (4 million units every 4 hours) for 7 days
GBS
Penicillin G (4 million units every 4 hours) for 2-3 weeks
H. influenzae
Ceftriaxone (2 g every 12 hour) for 7 days
Enterobacteriaceae
Ceftriaxone (2 g every 12 hour) + Gentamicin (1-2 mg/kg every 8 hours) for 3 weeks
Pseudomonas / Acinetobacter
Ceftazidime (2 g every 8 hours) + Gentamicin (1-2 mg/kg every 8 hour) for 3 weeks
Role of steroids
Trials have shown that dexamethasone given shortly before or at the same time as
first dose of antibiotics significantly improves outcome in patients
It reduces CSF synthesis of cytokines, CSF inflammation and cerebral edema
Treatment for raised ICP
Elevation of head end of the bed to 30 to 45 degrees, hyperventilation and
administration of mannitol
25 year old male presents with fever of 7 days, headache and drowsiness of 2
days. CSF examination shows protein 180 mg/dL, glucose 30mg/dL and
simultaneous blood sugar was 100 mg/dL. CSF cell count was 160 cells/mm3,
with 90% neutrophils, 10% lymphocytes. What is the most likely diagnosis?
Discuss the etiology, risk factors, differential diagnosis, complications, and
treatment of the same
Bacterial Meningitis
Complications
Obstructive hydrocephalus
Thrombophlebitis of leptomeningeal veins leading to venous thrombosis, cerebral
infarction
Chronic adhesive arachnoiditis
Cerebral abscess
Subdural empyema
Focal neurological deficit
Sensorineural hearing loss
Vasculitis of cranial vessel
Epilepsy
Waterhouse-Friedrichsen Syndrome
TUBERCULAR MENINGITIS
Clinical features (symptoms and signs) and complications of tubercular meningitis
Complications of Tubercular meningitis

Diagnosis and treatment of TB meningitis


Diagnosis
CSF examination :
Elevated protein
Decreased glucose concentration
Predominant lymphocytosis
Demonstration of acid-fast bacilli
Brain imaging
CT scan : Meningeal enhancement, obstructive hydrocephalus
MRI : More sensitive in detecting the distribution of meningeal inflammation
Mantoux test : Positive
CXR : May show evidence of pulmonary TB
Other tests : HIV to rule out immunocompromised state, blood sugar, electrolytes,
LFT, RFT, CBP, ESR
Treatment
Anti-tubercular therapy should be started if there is strong clinical suspicion
Two months of intensive therapy with four drugs (HRZE) and continuation phase
lasting 7-10 months, with two drugs (HR)
Steroids : Given in full dose for three weeks and then tapered off for the next three
weeks
Dexamethasone 12 mg/day in divided doses OR
Prednisolone 60 mg/day
STROKE
A 38 year old male came with complaints of weakness of right half of body for
last one day. a. What are the causes for Stroke in Young b. Clinical features of
left MCA territory infarct c. How will you manage this patient?
Causes of stroke in young
Hypercoagulable states
Protein C deficiency
Protein S deficiency
Factor V Leiden
Antithrombin III deficiency
Prothrombin 20210 mutation
Homocysteinemia
Nephrotic syndrome
APLA
Polycythemia vera
Connective tissue disorders
Systemic vasculitis (PAN, Wegner’s granulomatosis, Takayasu arteritis, Giant Cell
Arteritis)
Systemic lupus erythematosus
Infections: Syphilis, meningitis, tuberculosis, HIV
Drugs : Cocaine, amphetamine
Cardiac disorders
Atrial fibrillation
Atrial myxoma
Infective endocarditis
Libman-Sacks endocarditis
MI
CNS lesions
AV malformations
Aneurysms
Neoplasm
Bleeding diathesis
Thrombocytopenia
Hemophilia
Liver failure
Clinical features of MCA infarct
Contralateral hemiparesis (worse in arm and face than in the leg)
Dysarthria
Hemianesthesia
Contralateral homonymous hemianopia
Aphasia (if dominant hemisphere affected)
Hemi-spatial neglect syndrome (if non-dominant hemisphere affected)
Investigations
CT scan head : Imaging modality of choice in acute stroke because it can be done
fast and is easily available
Infarct appears as hypointense area
May not be visible for 24-48 hours in CT scan
Can also exclude hemorrhage, and other pathologies like neoplasms, abscesses and
other conditions that mimic stroke
Contrast CT is more useful in subacute infarcts and can also visualize venous
structures
MRI brain
Is more sensitive than picking up infarcts in certain areas of brain like brainstem and
cortex
More sensitive in picking up early brain infraction than CT scan
CT angiogram and MR angiogram
Can be done to identify the exact location of vessel block
Carotid and vertebral artery doppler
ECG and Echocardiogram : To rule out any heart problem
Blood Tests : Blood sugar, urea, creatinine, electrolytes, hemoglobin, cell count,
coagulation profiles, lipid profile, toxicology screen
Treatment
Initial management
Assess ABCs (airway, breathing and circulation)
Secure airway
Monitor oxygenation
Provide respiratory ventilatory support
IV thrombolysis
Recombinant tPA (tissue plasminogen activator) has been show to improve the
outcome if given within 4.5 hours of stroke
tPA is contraindicated in the presence of high BP (>185/110), recent major surgery,
prior stroke or head injury within 3 months and GI bleeding preceding 3 weeks
Endovascular techniques
Intra-arterial thrombolysis and endovascular thrombectomy
Can be used in ischemic stroke due to large vessel occlusions such as MCA, ICA and
basilar artery
Antithrombotic treatment
Antiplatelet agent, aspirin should be given as soon as the diagnosis of ischemic
stroke is confirmed. A loading dose of aspirin 325 mg should be given followed by
150 mg daily lifelong. However withhold these agents before and for 24 hours after
thrombolytic therapy
Clopidogrel is not useful in the immediate management but can be used along with
aspirin to patients at high risk of developing subsequent stroke The dose is 75 mg
daily
Anticoagulation
Indicated in cardioembolic stroke (e.g. atrial fibrillation)
Heparin or LMWH can be given subcutaneously and later changed to oral therapy
using warfarin
Supportive measures
Prevention of complications of bedridden patients
Infections (pneumonia, UTI and skin) : Prophylactic antibiotics
DVT with PE : Subcutaneous heparin and pneumatic compression stockings
Fever should be treated with antipyretics and surface cooling
Blood glucose should be monitored and kept below 110 mg/dL
Cerebral edema : IV mannitol, raising head end of bed, hyperventilation
Rehabilitation
Physical, occupational and speech therapy
To return the patient to home and to maximize recovery.
Mention four causes of stroke in young

Clinical features of Wallenberg syndrome (x2)


Secondary prevention of stroke
ABCD2 score can help in identifying the stroke risk. A score of <4 is associated with a
minimal risk whereas >6 is high-risk for a stroke.
A 60 year old male presented with sudden onset of weakness of left half of the
body with deviation of angle of the mouth to the right side. He was a known
Hypertensive and had associated head Ache, vomiting and altered sensorium.
What is the likely diagnosis? Discuss the etiology, clinical features,
investigations and treatment of the same
Hemorrhagic stroke involving the right middle cerebral artery likely due to
hypertension
Etiology
Head injury
AV malformation or aneurysm rupture
Cavernous angioma
Capillary telangiectasias
Hypertension (usually causes hemorrhage into putamen)
Large infarct (bleeding can occur into infarct)
Cortical venous sinus thrombosis
Cerebral amyloid angiopathy
Drugs (cocaine, amphetamine)
Anticoagulant therapy
Brain tumor
Vasculitis
Sepsis (may cause small petechial hemorrhage in the brain)
Moyamoya disease
Coagulopathy
Clinical features
Sudden onset headache, focal neurological deficits and impaired consciousness
Headache and vomiting is also present due to raised intracranial pressure
Seizures are common in lobar hemorrhages involving cerebral cortex
Symptoms and signs may vary depending upon the location of bleed
Putamen : hemiplegia, hemisensory loss, homonymous hemianopia, gaze palsy,
stupor and coma
Cerebellum : Inability to walk due to imbalance, vomiting, occipital headache, neck
stiffness, gaze palsy and facial weakness
Thalamus : hemiparesis, hemisensory loss and occasional transient homonymous
hemianopsia
Cerebral lobes : Unilateral hemiparesis and hemisensory deficits. Speech
impairment can occur if dominant hemisphere is involved.
Pons : Deep coma due to disruption of reticular activating system. Pinpoint pupils.
Investigations
CT scan head
Size and location of hematoma
Extension into ventricular system
Presence of surrounding edema
Shift of brain contents
Hyperacute blood will appear hyperdense. Over weeks it will appear isodense and
later becomes hypodense
MRI head : More accurate for detection of chronic ICH
Others : Blood sugar, urea, creatine, electrolytes, lipid profile and complete
hemogram
Treatment
Admit the patient in ICU for continuous neurological and hemodynamic monitoring
All anticoagulant and antiplatelet drugs should be discontinued for at least one or
two weeks after the onset of hemorrhage
Blood pressure control
Maintain the systolic between 160-140 mmHg
Drugs used: IV nitroprusside, nicardipine or labetalol
Surgery: Surgical evacuation for all cerebellar hemorrhages > 3 cm diameter as they
are associated with a high risk of brainstem compression and obstructive
hydrocephlus
Managing raised ICP
Hyperventilation
Mannitol
Inj. furosemide 20 mg IV every 6 hours
Elevation of head end of table
Prevention of seizures
Inj. phenytoin 15 mg/kg body weight loading dose given as IV infusion over 30 mins,
then 100 mg every 8th hourly
Hemostatic therapy:
Given if patient presents within 3 hours of onset
Treatment with activated recombinant factor VIIa may stop ongoing hemorrhage and
hematoma enlargement
General measures : Take care of ABCs, DVT prophylaxis, nutrition etc.
Neurological deficits in right internal capsule infarct and their anatomical basis
A right internal capsule infarct is caused when the right middle cerebral artery
circulation is affected specifically the M1 component.
The neurological deficits are
Contralateral dense hemiplegia (Upper and Lower limb paralysis)
Contralateral upper motor neuron seventh nerve palsy (Only lower part of face
involved)
Contralateral hemisensory loss
Contralateral Homonymous hemianopia
Write an emergency plan for investigation and treatment of left sided weakness of 1
hour duration in a male patient aged 45 years.
Mention four causes of sudden onset of convulsions with altered sensorium.
Discuss the clinical features and treatment of hypertensive intracerebral
Hemorrhage.
Causes of sudden onset of convulsion with altered sensorium
Epileptic seizures
Trauma (such as head injury)
Stroke
Infections (such as meningitis or encephalitis)
Toxins or ingestions of poisons
Metabolic disturbances (such as hypoglycemia or hypernatremia)
Drug overdose/withdrawal
Brain tumors
Eclampsia
Hypertensive intracerebral hemorrhage
It refers to bleeding within the brain parenchyma.
It usually results from spontaneous rupture of small penetrating artery deep in the
brain
The most common sites are the basal ganglia (especially the putamen), thalamus,
cerebellum and pons. Cortical bleeds are rare.
Clinical features
Sudden onset headache, focal neurological deficits and impaired consciousness
Headache and vomiting is also present due to raised intracranial pressure
Seizures are common in lobar hemorrhages involving cerebral cortex
Symptoms and signs may vary depending upon the location of bleed
Putamen : hemiplegia, hemisensory loss, homonymous hemianopia, gaze palsy,
stupor and coma
Cerebellum : Inability to walk due to imbalance, vomiting, occipital headache, neck
stiffness, gaze palsy and facial weakness
Thalamus : hemiparesis, hemisensory loss and occasional transient homonymous
hemianopsia
Cerebral lobes : Unilateral hemiparesis and hemisensory deficits. Speech impairment
can occur if dominant hemisphere is involved.
Pons : Deep coma due to disruption of reticular activating system. Pinpoint pupils.
Investigations
CT scan head
Size and location of hematoma
Extension into ventricular system
Presence of surrounding edema
Shift of brain contents
Hyperacute blood will appear hyperdense. Over weeks it will appear isodense and
later becomes hypodense
MRI head : More accurate for detection of chronic ICH
Others : Blood sugar, urea, creatine, electrolytes, lipid profile and complete
hemogram
Treatment
Admit the patient in ICU for continuous neurological and hemodynamic monitoring
All anticoagulant and antiplatelet drugs should be discontinued for at least one or
two weeks after the onset of hemorrhage
Blood pressure control
Maintain the systolic between 160-140 mmHg
Drugs used: IV nitroprusside, nicardipine or labetalol
Surgery: Surgical evacuation for all cerebellar hemorrhages > 3 cm diameter as they
are associated with a high risk of brainstem compression and obstructive
hydrocephlus
Managing raised ICP
Hyperventilation
Mannitol
Inj. furosemide 20 mg IV every 6 hours
Elevation of head end of table
Prevention of seizures
Inj. phenytoin 15 mg/kg body weight loading dose given as IV infusion over 30 mins,
then 100 mg every 8th hourly
Hemostatic therapy:
Given if patient presents within 3 hours of onset
Treatment with activated recombinant factor VIIa may stop ongoing hemorrhage and
hematoma enlargement
General measures : Take care of ABCs, DVT prophylaxis, nutrition etc.
TIA
Define TIA, mention the types of TIA and differentiate an embolic from an ischemic
stroke.
Transient ischemic attack (TIA) is characterized by a brief episode of neurological
dysfunction (sudden loss of function) in which symptoms and signs resolve
completely after a brief period within 24 hours (usually within 30 minutes)

What is Amaurosis fugax and mention its clinical significance


Amaurosis fugax refers to transient loss of vision in one or both eyes.
Clinical significance
It can be manifestation of an underlying pathology. The various causes are -
Mono-ocular loss
Retinal artery emboli (carotid artery disease, cardiac emboli)
Retinal vein occlusion
Retinal vasospasm and retinal migraine
Optic neuropathy
Papilledema
Optic nerve compression
Idiopathic
Binocular
Migraine
Seizure
Vertebrobasilar ischemia
Hypotension
Investigation
Ophthalmologic evaluation
ESR and CRP to exclude giant cell arteritis
Carotid doppler
MR angiogram to rule out carotid artery dissection
ECG and ECHO to rule out cardiac disease
EEG if seizures are suspected
Coagulation profiles for hypercoagulable states
CBC to screen for polycythemia vera and essential thrombocythemia
GUILLAIN BARRE’ SYNDROME
Guillain Barre’ syndrome (x2)
Guillain Barre’ syndrome is a heterogenous group of immune mediated conditions. It
is the most common acute, severe fulminant polyradiculopathy/polyneuropathy.
Clinical features
Progressive symmetric muscle weakness - Weakness can vary from mild weakness of
legs to complete paralysis of all extremity, facial, respiratory and bulbar muscles
Weakness usually starts in the lower limbs and then ascends upwards to involve
trunk and upper limbs (ascending paralysis)
Absent/depressed deep tendon reflexes
Facial nerve involvement (LMN type)
Severe respiratory muscle weakness may lead to respiratory failure and requires
ventilatory support
Extraocular movements may be affected (oculomotor involvement) - Diplopia
Paresthesia in hands and feet
Autonomic neuropathy occurs in majority of patients and manifests as bradycardia,
urinary retention, fluctuating blood pressure, orthostatic hypotension, arrhythmias,
ileus and loss of sweating
Investigations
Nerve conduction studies (NCS) and electromyography (EMG) : To confirm the
diagnosis and also to know the type of GBS.
Delayed distal latencies
Slowed nerve conduction velocities
Conduction block
Decreased recruitment
Rapid firing motor units in weak muscles
CSF analysis
Protein elevated with normal WBC count (albumino-cytologic dissociation)
Antibodies : Against nerve components may be detected in the blood of GBS
patients
Treatment
Plasmapheresis removes the circulating antibodies and helps in fast recovery. Four
sittings of plasmapheresis are recommended.
IVIG neutralizes circulating antibodies. Dose = 0.4 g/kg daily for 6 days
Supportive therapy — bowel and bladder care, adequate nutrition, monitoring of
respiratory failure and providing ventilatory support if required, cardiac monitoring
and physiotherapy are all important.
A 30 years old male with history of diarrhea 2 weeks ago, presents with acute
onset paraparesis of 3 days duration, with occasional tingling and numbness in
the legs. a. What is the likely diagnosis? b. Discuss the clinical features and the
complications of the diagnosis. c. Add a note on the management of the above
condition
Guillain Barre’ Syndrome
Clinical features
Progressive symmetric muscle weakness - Weakness can vary from mild weakness of
legs to complete paralysis of all extremity, facial, respiratory and bulbar muscles
Weakness usually starts in the lower limbs and then ascends upwards to involve
trunk and upper limbs (ascending paralysis)
Absent/depressed deep tendon reflexes
Facial nerve involvement (LMN type)
Severe respiratory muscle weakness may lead to respiratory failure and requires
ventilatory support
Extraocular movements may be affected (oculomotor involvement) - Diplopia
Paresthesia in hands and feet
Autonomic neuropathy occurs in majority of patients and manifests as bradycardia,
urinary retention, fluctuating blood pressure, orthostatic hypotension, arrhythmias,
ileus and loss of sweating
Investigations
Nerve conduction studies (NCS) and electromyography (EMG) : To confirm the
diagnosis and also to know the type of GBS.
Delayed distal latencies
Slowed nerve conduction velocities
Conduction block
Decreased recruitment
Rapid firing motor units in weak muscles
CSF analysis
Protein elevated with normal WBC count (albumino-cytologic dissociation)
Antibodies : Against nerve components may be detected in the blood of GBS
patients
Treatment
Plasmapheresis removes the circulating antibodies and helps in fast recovery. Four
sittings of plasmapheresis are recommended.
IVIG neutralizes circulating antibodies. Dose = 0.4 g/kg daily for 6 days
Supportive therapy — bowel and bladder care, adequate nutrition, monitoring of
respiratory failure and providing ventilatory support if required, cardiac monitoring
and physiotherapy are all important.
SEIZURES & EPILEPSY
Evaluation (investigations) of first seizure in an adult
When a patient is seen shortly after a seizure, the first priorities are
attention to vital signs
respiratory and cardiovascular support
treatment of seizures if they resume
If this is the patient’s first seizure, then the emphasis will be to establish whether the
reported episode was a seizure rather than another paroxysmal event
In case of generalized, tonic-clonic seizures (GTCS), features supporting organicity
are postictal confusion, tongue bite, history of fall or having sustained injury, etc.
Identify risk factors and precipitating events
Sleep deprivation
Systemic diseases
Electrolyte and metabolic derangements (like hypoglycemia, hypo/hypercalcemia,
hyponatremia, hypomagnesemia)
Acute infections
Drugs that lower seizure threshold (like penicillin, quinolones, anti-psychotics,
lithium, amphetamine, barbiturates, cocaine) threshold or alcohol or illicit drug use
should be identified
General physical examination
Look for signs of system illness or infection
Careful examination of skin :
Neurocutaneous markers —
Tuberous sclerosis → Shagreens patch, ash-leaf macules, adenoma sebaceum
Neurofibromatosis → Café-au-lait spots, plexiform neurofibroma, neurofibroma
Signs of chronic liver disease
Signs of renal disease
Organomegaly may indicate storage disorders
Decide whether anticonvulsant therapy is required in addition to treatment for any
underlying illness
If no metabolic or infectious causes found then look for
Focal features of seizures
Focal neurological deficits
Any other neurological dysfunction / mental retardation
Unusual features such as, prolonged duration of seizures (>6 hours), more than six
seizures, status epilepticus, or a prolonged postictal state
If present, usually antiepileptic therapy is required. Then MRI and EEG are done to
find any mass lesion/stroke/degenerative lesion. Treatment of underlying cause
besides antiepileptic therapy.
If absent, it is idiopathic epilepsy and antiepileptic therapy continued
Treatment of status epilepticus (x6)
Ensure airway patency - Give oxygen
Monitor vitals, SpO2
Secure IV access
Investigations - RBS, Electrolytes, AED levels, LFT, RFT, CBC
Early 5-30 minutes

Established and early refractory: 30-40 minutes - IV antiepileptics with cardiac


monitoring (Any one of the following)
Phenytoin 15-20 mg/kg at 50 mg/min
Fosphenytoin 15 mg/kg at 50-100 mg/min
Phenobarbitone 10 mg/kg at 100 mg/min
Valproic acid 30 mg/kg over 2-5 minutes
Levetiracetam 2-3 g over 15 minutes
Late refractory > 48 hours
Clinical features (symptoms and signs) of temporal lobe epilepsy
MESIAL TEMPORAL LOBE EPILEPSY
Clinical features
Focal seizures with a remitting and relapsing course
Aura → Abdominal (feeling that food is regurgitating) > Visual > Sensory
Complex automatism → Lip smacking
Post-ictal confusion
Anterograde amnesia → No memory of the episode
Investigations
EEG → Temporal spikes
MRI → Hippocampal sclerosis
Management
Anteromedial resection of the temporal lobe
HEADACHES & MIGRAINE
Discuss the pathophysiology, clinical features and treatment of an acute attack
of migraine with aura. // Prophylaxis and management of migraine // Management
of migraine (x3)
Cluster headache.
OTHERS
Discuss the etiopathogenesis, clinical features, investigations and treatment of
idiopathic Parkinson’s disease. // Clinical features and management of Parkinson’s
disease // Medical management of Parkinson’s disease
Parkinsonism is a syndrome consisting of a variable combination of tremor, rigidity,
bradykinesia, and a characteristic disturbance of gait and posture
Idopathic Parkinson’s disease is a chronic progressive disorder in which idiopathic
parkinsonism occurs without evidence of more widespread neurological involvement.
Clinical features and treatment of multiple sclerosis
Multiple sclerosis is an autoimmune disorder characterized by multiple
demyelinating lesions in the brain and spinal cord
Etiology
Autoimmune disorder where T cells are activated and destroy myelin sheath
Infection by a latent virus (e.g. EBC) could be a trigger
Genetic factors (HLA-DR2 association)
Environmental factors → Temperate climate (European countries) and tropical climate
(Asian countries)
Clinical features
Patient profile : Young adult with two or more clinically distinct episodes of CNS
dysfunction with partial resolution
Cranial nerves : Optic neuritis is a common presentation leading to central scotoma
Motor system :
Due to corticospinal tract lesions
Upper limb weakness
Paraplegia
Paraparesis
UMN signs → Spasticity, exaggerated DTR, clonus, and extensor plantar response
Sensory system
Due to demyelinating lesions in the spinothalamic, posterior column or dorsal roots.
Numbness, tingling, pins and needles, tightness, coldness, radicular pains etc.
Cerebellum
Ataxia and incoordination
Spinal cord
Bowel, bladder and sexual dysfunction leading to urgency, urinary incontinence,
constipation or fecal incontinence, erectile dysfunction
Neuromyelitis optica is a variant of MS characterized by involvement of only optic
nerve and spinal cord
Lhermitte’s sign (Barber chair phenomenon) : Electric shock like sensation that runs
down the back and into the limbs on flexing the neck
Diagnosis
Should be suspected when multiple areas of CNS are involved at different times
MRI of brain and cervical cord is investigation of choice. It shows demyelinating
lesions.
Evoked potential recordings show prolongation and can detect subclinical
involvement of the visual, auditory and somatosensory pathways
CSF examination : lymphocytosis or increased protein concentration
CSF electrophoresis : oligoclonal bands
Treatment
Acute attacks
Methylprednisolone 1 g IV for 3 days followed by oral prednisolone, 60 mg per day
for 1 week after which it is tapered over next 2-3 weeks
Plasmapheresis and IVIG
Preventing relapses
Two forms of recombinant beta interferon, interferon beta-1a and interferon beta-1b
Glatiramer acetate
Immunosuppressive therapy with cyclophosphamide, azathioprine, methotrexate,
cladribine
Natalizumab
Symptomatic therapy
Spasticity — physiotherapy, baclofen, tizanidine, diazepam, inj. botulinum toxin
Ataxia — isoniazid, clonazepam
Sensory symptoms — carbamazepine, gabapentin
Spastic bladder — anticholinergics like oxybutynin or propantheline
Fatigue — amantadine
Impotence — sildenafil
Depression — imipramine, amitriptyline
Bell’s palsy (x2)
Unilateral isolated LMN type of facial paralysis
Though to be due to latent herpes virus infection which are reactivated from cranial
nerve ganglia
Causes swelling and compression of nerve within tight petrous bone facial canal
Clinical manifestations
Symptoms
Hyperacusis
Decreased production of tears, saliva and altered taste
Facial or retro-auricular pain
Signs
Paralysis of all muscles of facial expression on the affected side
Dropping of corner of mouth
Eye closure is weak
Drooling of saliva from angle of mouth
Action of the levator anguli oris on the normal side, makes the angle of mouth to
deviate to the opposite side of the lesion, when the patient shows his teeth
When the closure of the eyelid is attempted, the eye on the paralyzed side rolls
upwards (Bell’s phenomenon)
Due to exposure of the cornea, patient may develop exposure keratitis and corneal
ulceration
Treatment
Steroids - Prednisolone 1 mg/kg/day for 5-7 days then tapered next week
Antivirals for 5-7 days
Acyclovir 800 mg 5 times a day
Valacyclovir 800 mg BD
Surgical decompression - if no resolution of symptoms after 2 weeks
Causes of ptosis
Unilateral ptosis
Third cranial nerve lesion
Lesion of cervical sympathetic pathway (Horner’s syndrome)
Lesions of the upper eyelid
Bilateral ptosis
Myopathies
Myasthenia gravis
Bilateral Horner’s syndrome
Snake bite
Botulism
Trigeminal neuralgia - clinical recognition and treatment
Acute cerebellar ataxia
Lesion of
Midline vermis → Truncal ataxia
Cerebellar hemispheres → Limb ataxia of ipsilateral side
Gait/Limb ataxia
Patients will tend to stand with feet well apart and are often frightened to stand
Patients tend to reel to the side of unilateral lesion or from side to side if central or
bilateral lesion (drunken gait)
Tandem gait demonstrates minor degrees of gait ataxia
Instability may increase if eyes are closed but patients do not fall. This is not a true
Romberg’s test
Truncal ataxia
Patients cannot sit or stand unsupported and tend to fall backwards
Midline cerebellar lesion, or may be a feature of post-chickenpox cerebellar
syndrome
Truncal tremor may evident - constant jerking of trunk and head (tibutation)
The outstretched arm tends to be held hyper pronated at rest and at a slightly higher
level than unaffected side (Riddoch’s sign) and rebounds upwards if gently pressed
downwards and then suddenly released by the examiner
Finger-nose and heel-knee-shin tests will demonstrate even mild limb ataxia, with
terminal intention tremor and dysmetria (past pointing).
Other signs
Pendular knee jerk → Three or more swings at the knee when knee reflex is elicited
Tremor → Unilateral or bilateral intention tremor
Nystagmus → Coarse nystagmus worse on looking towards the side of lesion with
fast component towards the affected side (as opposed to INO)
Dysdiadochokinesia → Inaccuracies in performing rapid alternating movements
Cerebellar dysarthria → Scanning dysarthria is a jerky and explosive speech with
separated syllables may be demonstrated by asking the patient to repeat ‘baby
hippopotamus’
Macorgraphia → Writing may be larger than normal

Sub acute combined degeneration of spinal cord


It is myelopathy that develops due to nutritional deficiency of vitamin B12 deficiency,
including pernicious anemia. There is degeneration of dorsal and lateral spinal
(corticospinal) columns, hence called combine degeneration. Degeneration is due to
defect in myelin formation
Clinical features
Subacute onset
Posterior column degeneration : paresthesia, ataxia associated with loss of vibration
and position sense. Romberg’s sign is positive
Corticospinal tract degeneration : weakness, spasticity, extensor plantar response,
clonus, paraplegia and even fecal and urinary incontinence
Ankle jerk may be absent due to associated peripheral neuropathy but knee jerk is
brisk
Other deficits : memory loss, irritability and dementia
Macrocytic anemia of Vitamin B12 deficiency
Investigations
Serum Vitamin B12 level will be low
CBC shows megaloblastic anemia
MRI of the spinal cord and brain may show hyperintense lesions in the white matter
Treatment
Inj. Vitamin B12 (IM)
1 mg every day for one week followed by
1 mg every week for one month followed by
1 mg every month lifelong
Diagnostic approach to a patient with vertigo
Vertigo is a false sensation of movement of the self or the environment, usually a
spinning or wheeling sensation.
Approach to a case with vertigo
Clinical history and findings
Peripheral vertigo should be distinguished from central vertigo (more serious)
Ask the patient to describe what he exactly feels. This will help in differentiating
vertigo from other causes of dizziness such as lightheadedness
Nausea, vomiting, and imbalance usually accompanies vertigo
H/O recurrent episode in the past suggest BPPV
Ask about triggers and relievers i.e. whether triggered by head/body position change
which suggests peripheral vertigo
Tinnitus and hearing loss suggest middle ear pathology and vertigo of peripheral
origin
Presence of nystagmus should be noted.
Peripheral lesions → Horizontal nystagmus with rotatory component. Inhibited by
fixation of eyes onto object.
Central lesions → Vertical nystagmus with other signs of brainstem or cerebellar
dysfunction. Not inhibited by fixation of eyes on object.
H/O loss of consciousness, focal neurological findings and cerebellar signs suggests
a central cause of vertigo
Investigations
Audiological tests
Caloric stimulation
Electronystagmography
CT scan or MRI
Brainstem auditory evoked potential studies
Symptomatic treatment
Scopolamine, cinnarizine, betahistine, meclizine, dimenhydrinate, diphenhydramine,
prochlorperazine, promethazine, metoclopramide and domperidone
Extras
LACUNAR STROKE
Etiology
Micro-atheroma (commonest cause)
Lipohyalinosis
Small emboli
Clinical features
Pure motor variety is the most frequent lacunar stroke syndrome
Hemiparesis
No cortical signs (aphasia, agnosia, apraxia etc.)
No sensory deficit
Sometimes the weakness may affect only the arm or leg
Investigations
CT scan - less sensitive in picking up lacunar infarcts
MRI scan - more sensitive than CT scan
CT angiography or MR angiography to rule out major

Endocrinology, Metabolic disorders incl.


DM and Nutrition
PITUITARY
What is Sheehan’s syndrome? Explain the clinical presentation.
Sheehan’s syndrome is a potentially life-threatening complication due to infarction of
pituitary gland following postpartum hemorrhage.
Clinical features
Failure to lactate
Failure to regain menstruation after delivery
Other features of hypopituitarism develop over months or years
Few patients may present acutely with hypotension, hyponatremia and
hypothyroidism
Diagnosis
Demonstration of low levels of trophic pituitary hormones (FSH, LH, ACTH, LH,
Prolactin, GH)
Low T4, cortisol, testosterone, IGF-1
MRI : Early stages may show hypertrophied pituitary, Later stages, atrophic pituitary
and empty sella
Treatment
Control of hemorrhage
Volume replacement
Administration of deficient hormones
Gigantism
Gigantism refers to the manifestation of excessive secretion of growth hormone prior
to closure of epiphyseal growth plates in long bones before the onset of puberty.
Etiology
Pituitary tumor (somatotroph pituitary adenoma) is the most common cause
Other tumors : pancreas, lungs and adrenal glands (due to production of GH or
GHRH by them)
Clinical features
Arthropathy
Big boggy hands
Carpal tunnel syndrome
Diabetes
Enlarged tongue, heart and throat
Fields : Bitemporal hemianopia
Gynecomastia, galactorrhea
Hypertension
Increased side of shoes, hat, gloves, dentures, rings
Jaw enlargement and prognathism
Investigations
Biochemical
GH levels during an oral glucose tolerance test
Normal subjects : GH suppresses to below 2 mU/L
Acromegaly : Failure to suppress or paradoxical rise
Blood glucose levels may be high due to insulin resistance
Prolactin elevated in 30% patients
Imaging
CT or MRI of brain demonstrates pituitary adenoma
Skull X-rays : Cortical thickening, enlargement of frontal sinuses
X-ray hands : Tufting of terminal phalanges and soft tissue thickening
Colonoscopy : For screening colonic neoplasms
Management
Surgical : Trans-sphenoidal resection of the adenoma is first line.
Radiotherapy : As second line if acromegaly persists after surgery
Medical :
Somatostatin analogues : octreotide, lanreotide
Dopamine agonists : bromocriptine, cabergoline
GH receptor antagonist : pegvisomant
THYROTOXICOSIS
Twenty five year old girl presented with weight loss and diarrhea. On
examination she had irregular pulse and prominent eyes. a. What is probable
diagnosis and what other history and examination will you do? b. Write what
complications she can develop? c. How do you investigate her? d. Discuss
various treatment options
Grave’s Disease (Primary thyrotoxicosis)
Complications/Clinical features
Thyroid
Diffuse or nodular enlargement, warmth and bruit
Gastrointestinal system
Weight loss
Increased appetite
Vomiting
Diarrhea
Steatorrhea
Cardiovascular system
Sinus tachycardia
Atrial fibrillation
‘Scratchy’ mid-systolic murmur (Means-Lerman Scratch)
Exertional dyspnea
Palpitations
Angina
Nervous system
Nervousness
Irritability
Tremors
Emotional lability
Skin and integumentary system
Soft warm and moist skin
Increased sweating
Pruritus, palmar erythema
Pretibial myxedema
Alopecia
Clubbing
Reproductive system
Amenorrhea or oligomenorrhea
Repeated abortions
Infertility
Loss of libido
Impotence
General
Heat intolerance
Fatigue
Gynecomastia
Eyes

Investigations
TSH levels → Very low and undetectable
Serum T3 and T4 levels → Raised in most cases
IV TRH → No TSH response
I-131 uptake by thyroid gland → May be increased
TSH receptor antibody (TRAb) / Long acting thyroid stimulating antibody (LATS) →
Present
Minor LFT abnormalities
Mild hypercalcemia
Glycosuria
Three treatment options -
Antithyroid drugs
Thionamides : e.g. propylthiouracil, carbimazole, methimazole
Mechanism of action → inhibit TPO enzyme and prevent binding of iodine to
tyrosine. Propylthiouracil additionally inhibits peripheral conversion of T4 to T3.
Dose
Propylthiouracil → 100-600 mg/day
Carbimazole → 5-15 mg TID for 3-4 weeks followed by 10 mg TID for 4-8 weeks.
Maintenance dose 5-20 mg daily.
Adverse effects → Rashes, urticaria, fever, arthralgia, agranulocytosis
Potassium perchlorate
Mechanism of action → It reduces uptake of iodine. Used temporarily in iodine
induced thyrotoxicosis.
Adverse effect → More toxic and produces red cell aplasia
Potassium iodide
Mechanism of action : Decreases synthesis of T3 and T4 and inhibits hormone
release. Makes thyroid gland firm and less vascular
Indications : Preparation for thyroidectomy and thyrotoxic crisis
Beta blockers
Used for immediate relief and control of symptoms due to sympathetic overactivity
such as anxiety, palpitation, increased bowel activity, lid retraction, tremors.
Drug of choice → Propranolol 20-40 mg QID
Surgical treatment
Subtotal thyroidectomy is the treatment of choice
Preparation
Make the patient euthyroid by using anti-thyroid drugs
Use potassium iodide (60 mg BD x 2 weeks) to make the gland firm and reduce
vascularity
Radioactive iodine
I-131 concentrated in thyroid, emits gamma-ray radiation from within the follicles
and destroy thyroid cells. Taken orally as sodium salt of 131I dissolved in water or as
pills. Patients must be euthyroid before treatment
Dose : 185-555 MBq
Indications :
Age > 40 years
Young patients with short life span due to some other reason
Young individuals who are sterilized
Treatment of thyroid ophthalmopathy
Medical
Methylcellulose or Hypromellose eye drops to aid lubrication for grittiness
Use of tinted glasses to reduce lacrimation
Papilledema, loss of visual acuity and visual field defect need emergency treatment
and are treated by systemic prednisolone (30-120 mg daily). If there is no response
orbital decompression has to be performed.
Surgical
Lateral tarsorrhaphy for corneal ulcers.
Corrective eye muscle surgery for persistent diplopia
Outline etiology, clinical features, investigation and management of Graves
disease
Etiology
Age : Peak incidence at 20-40 years of age
Sex : Female to male ratio = 5:1
Autoantibodies
LATS (Long acting thyroid stimulating antibodies) / TRAb (TSH receptor antibodies) :
IgG antibodies targeted against the TSH receptors on follicular cell of thyroid.
Stimulate thyroid hormone production and gland enlargement
Ophthalmopathy and Dermopathy : Due to immunologically mediated activation of
fibroblasts in the extraocular muscles and skin. This along with accumulation of
glycosaminoglycans and water causes edema initially. Later, fibroblasts cause fibrosis.
Genetic association : HLA-DR3 associated
May be triggered by viral or bacterial infection
Complications/Clinical features
Thyroid
Diffuse or nodular enlargement, warmth and bruit
Gastrointestinal system
Weight loss
Increased appetite
Vomiting
Diarrhea
Steatorrhea
Cardiovascular system
Sinus tachycardia
Atrial fibrillation
‘Scratchy’ mid-systolic murmur (Means-Lerman Scratch)
Exertional dyspnea
Palpitations
Angina
Nervous system
Nervousness
Irritability
Tremors
Emotional lability
Skin and integumentary system
Soft warm and moist skin
Increased sweating
Pruritus, palmar erythema
Pretibial myxedema
Alopecia
Clubbing
Reproductive system
Amenorrhea or oligomenorrhea
Repeated abortions
Infertility
Loss of libido
Impotence
General
Heat intolerance
Fatigue
Gynecomastia
Eyes
Investigations
TSH levels → Very low and undetectable
Serum T3 and T4 levels → Raised in most cases
IV TRH → No TSH response
I-131 uptake by thyroid gland → May be increased
TSH receptor antibody (TRAb) / Long acting thyroid stimulating antibody (LATS) →
Present
Minor LFT abnormalities
Mild hypercalcemia
Glycosuria
Three treatment options -
Antithyroid drugs
Thionamides : e.g. propylthiouracil, carbimazole, methimazole
Mechanism of action → inhibit TPO enzyme and prevent binding of iodine to
tyrosine. Propylthiouracil additionally inhibits peripheral conversion of T4 to T3.
Dose
Propylthiouracil → 100-600 mg/day
Carbimazole → 5-15 TID for 3-4 weeks followed by 10 mg TID for 4-8 weeks.
Maintenance dose 5-20 mg daily.
Adverse effects → Rashes, urticaria, fever, arthralgia, agranulocytosis
Potassium perchlorate
Mechanism of action → It reduces uptake of iodine. Used temporarily in iodine
induced thyrotoxicosis.
Adverse effect → More toxic and produces red cell aplasia
Potassium iodide
Mechanism of action : Decreases synthesis of T3 and T4 and inhibits hormone
release. Makes thyroid gland firm and less vascular
Indications : Preparation for thyroidectomy and thyrotoxic crisis
Beta blockers
Used for immediate relief and control of symptoms due to sympathetic overactivity
such as anxiety, palpitation, increased bowel activity, lid retraction, tremors.
Drug of choice → Propranolol 20-40 mg QID
Surgical treatment
Subtotal thyroidectomy is the treatment of choice
Preparation
Make the patient euthyroid by using anti-thyroid drugs
Use potassium iodide (60 mg BD x 2 weeks) to make the gland firm and reduce
vascularity
Radioactive iodine
I-131 concentrated in thyroid, emits gamma-ray radiation from within the follicles
and destroy thyroid cells. Taken orally as sodium salt of 131I dissolved in water or as
pills. Patients must be euthyroid before treatment
Dose : 185-555 MBq
Indications :
Age > 40 years
Young patients with short life span due to some other reason
Young individuals who are sterilized
Treatment of thyroid ophthalmopathy
Medical
Methylcellulose eye drops to aid lubrication for grittiness
Use of tinted glasses to reduce lacrimation
Papilledema, loss of visual acuity and visual field defect need emergency treatment
and are treated by systemic prednisolone (30-120 mg daily). If there is no response
orbital decompression has to be performed.
Surgical
Lateral tarsorrhaphy for corneal ulcers.
Corrective eye muscle surgery for persistent diplopia
Mention the etiology of thyrotoxicosis. Discuss the diagnosis and treatment of
Grave’s disease
Etiology
Primary hyperthyroidism
Grave’s disease
Toxic multinodular goiter (Plummer’s disease)
Toxic adenoma
Iodine excess (Jod Basedow phenomenon)
Activating mutation of TSH receptor
Thyrotoxicosis without hyperthyroidism
Subacute thyroiditis, Hashitoxicosis
Amiodarone induced
Radiation induced
Thyrotoxicosis facitia
Struma ovarii
Infarction of thyroid gland
Secondary hyperthyroidism
TSH secreting pituitary adenoma
hCG mediation hyperthyroidism
Gestational thyrotoxicosis
Investigations
TSH levels → Very low and undetectable
Serum T3 and T4 levels → Raised in most cases
IV TRH → No TSH response
I-131 uptake by thyroid gland → May be increased
TSH receptor antibody (TRAb) / Long acting thyroid stimulating antibody (LATS) →
Present
Minor LFT abnormalities
Mild hypercalcemia
Glycosuria
Three treatment options -
Antithyroid drugs
Thionamides : e.g. propylthiouracil, carbimazole, methimazole
Mechanism of action → inhibit TPO enzyme and prevent binding of iodine to
tyrosine. Propylthiouracil additionally inhibits peripheral conversion of T4 to T3.
Dose
Propylthiouracil → 100-600 mg/day
Carbimazole → 5-15 TID for 3-4 weeks followed by 10 mg TID for 4-8 weeks.
Maintenance dose 5-20 mg daily.
Adverse effects → Rashes, urticaria, fever, arthralgia, agranulocytosis
Potassium perchlorate
Mechanism of action → It reduces uptake of iodine. Used temporarily in iodine
induced thyrotoxicosis.
Adverse effect → More toxic and produces red cell aplasia
Potassium iodide
Mechanism of action : Decreases synthesis of T3 and T4 and inhibits hormone
release. Makes thyroid gland firm and less vascular
Indications : Preparation for thyroidectomy and thyrotoxic crisis
Beta blockers
Used for immediate relief and control of symptoms due to sympathetic overactivity
such as anxiety, palpitation, increased bowel activity, lid retraction, tremors.
Drug of choice → Propranolol 20-40 mg QID
Surgical treatment
Subtotal thyroidectomy is the treatment of choice
Preparation
Make the patient euthyroid by using anti-thyroid drugs
Use potassium iodide (60 mg BD x 2 weeks) to make the gland firm and reduce
vascularity
Radioactive iodine
I-131 concentrated in thyroid, emits gamma-ray radiation from within the follicles
and destroy thyroid cells. Taken orally as sodium salt of 131I dissolved in water or as
pills. Patients must be euthyroid before treatment
Dose : 185-555 MBq
Indications :
Age > 40 years
Young patients with short life span due to some other reason
Young individuals who are sterilized
Treatment of thyroid ophthalmopathy
Medical
Methylcellulose eye drops to aid lubrication for grittiness
Use of tinted glasses to reduce lacrimation
Papilledema, loss of visual acuity and visual field defect need emergency treatment
and are treated by systemic prednisolone (30-120 mg daily). If there is no response
orbital decompression has to be performed.
Surgical
Lateral tarsorrhaphy for corneal ulcers.
Corrective eye muscle surgery for persistent diplopia
Define thyrotoxicosis. Discuss etiology, clinical features, diagnosis and
treatment of Grave's disease
What are the treatment options for a 25 year old married woman with Graves’
disease?
Three treatment options -
Antithyroid drugs
Thionamides : e.g. propylthiouracil, carbimazole, methimazole
Mechanism of action → inhibit TPO enzyme and prevent binding of iodine to
tyrosine. Propylthiouracil additionally inhibits peripheral conversion of T4 to T3.
Dose
Propylthiouracil → 100-600 mg/day
Carbimazole → 5-15 TID for 3-4 weeks followed by 10 mg TID for 4-8 weeks.
Maintenance dose 5-20 mg daily.
Adverse effects → Rashes, urticaria, fever, arthralgia, agranulocytosis
Potassium perchlorate
Mechanism of action → It reduces uptake of iodine. Used temporarily in iodine
induced thyrotoxicosis.
Adverse effect → More toxic and produces red cell aplasia
Potassium iodide
Mechanism of action : Decreases synthesis of T3 and T4 and inhibits hormone
release. Makes thyroid gland firm and less vascular
Indications : Preparation for thyroidectomy and thyrotoxic crisis
Beta blockers
Used for immediate relief and control of symptoms due to sympathetic overactivity
such as anxiety, palpitation, increased bowel activity, lid retraction, tremors.
Drug of choice → Propranolol 20-40 mg QID
Surgical treatment
Subtotal thyroidectomy is the treatment of choice
Preparation
Make the patient euthyroid by using anti-thyroid drugs
Use potassium iodide (60 mg BD x 2 weeks) to make the gland firm and reduce
vascularity
Radioactive iodine
I-131 concentrated in thyroid, emits gamma-ray radiation from within the follicles
and destroy thyroid cells. Taken orally as sodium salt of 131I dissolved in water or as
pills. Patients must be euthyroid before treatment
Dose : 185-555 MBq
Indications :
Age > 40 years
Young patients with short life span due to some other reason
Young individuals who are sterilized
Treatment of thyroid ophthalmopathy
Medical
Methylcellulose eye drops to aid lubrication for grittiness
Use of tinted glasses to reduce lacrimation
Papilledema, loss of visual acuity and visual field defect need emergency treatment
and are treated by systemic prednisolone (30-120 mg daily). If there is no response
orbital decompression has to be performed.
Surgical
Lateral tarsorrhaphy for corneal ulcers.
Corrective eye muscle surgery for persistent diplopia
Thyrotoxic crisis
Explain FOUR complications of Graves’ disease
Orbitopathy (ophthalmopathy): Inflammation and fibrosis of the orbital tissues can
result in proptosis, diplopia, and, in severe cases, vision loss.
Cardiac complications: Hyperthyroidism can increase the risk of atrial fibrillation, as
well as increase the demand on the cardiovascular system, leading to increased risk
of myocardial infarction or stroke.
Osteoporosis: Chronic hyperthyroidism can result in decreased bone density and an
increased risk of fractures.
Psychiatric comorbidities: Graves' disease can have a significant impact on quality of
life due to physical symptoms, as well as the psychological burden of the disease,
leading to increased risk of anxiety, depression, and stress-related disorders.
A 22 year old lady presents with a history of palpitations, anxiety and sweating
for the last few weeks before her final exams and she says she has lost some
weight. On questioning further she is found to have irregular periods. a. Give
two differential diagnoses b. How do you investigate this patient? c. Outline the
treatment.
Primary thyrotoxicosis (Grave’s disease), Panic Disorder
INVESTIGATIONS
TSH levels → Very low and undetectable
Serum T3 and T4 levels → Raised in most cases
IV TRH → No TSH response
I-131 uptake by thyroid gland → May be increased
TSH receptor antibody (TRAb) / Long acting thyroid stimulating antibody (LATS) →
Present
Minor LFT abnormalities
Mild hypercalcemia
Glycosuria
TREATMENT
Three treatment options -
Antithyroid drugs
Thionamides : e.g. propylthiouracil, carbimazole, methimazole
Mechanism of action → inhibit TPO enzyme and prevent binding of iodine to
tyrosine. Propylthiouracil additionally inhibits peripheral conversion of T4 to T3.
Dose
Propylthiouracil → 100-600 mg/day
Carbimazole → 5-15 TID for 3-4 weeks followed by 10 mg TID for 4-8 weeks.
Maintenance dose 5-20 mg daily.
Adverse effects → Rashes, urticaria, fever, arthralgia, agranulocytosis
Potassium perchlorate
Mechanism of action → It reduces uptake of iodine. Used temporarily in iodine
induced thyrotoxicosis.
Adverse effect → More toxic and produces red cell aplasia
Potassium iodide
Mechanism of action : Decreases synthesis of T3 and T4 and inhibits hormone
release. Makes thyroid gland firm and less vascular
Indications : Preparation for thyroidectomy and thyrotoxic crisis
Beta blockers
Used for immediate relief and control of symptoms due to sympathetic overactivity
such as anxiety, palpitation, increased bowel activity, lid retraction, tremors.
Drug of choice → Propranolol 20-40 mg QID
Surgical treatment
Subtotal thyroidectomy is the treatment of choice
Preparation
Make the patient euthyroid by using anti-thyroid drugs
Use potassium iodide (60 mg BD x 2 weeks) to make the gland firm and reduce
vascularity
Radioactive iodine
I-131 concentrated in thyroid, emits gamma-ray radiation from within the follicles
and destroy thyroid cells. Taken orally as sodium salt of 131I dissolved in water or as
pills. Patients must be euthyroid before treatment
Dose : 185-555 MBq
Indications :
Age > 40 years
Young patients with short life span due to some other reason
Young individuals who are sterilized
Treatment of thyroid ophthalmopathy
Medical
Methylcellulose eye drops to aid lubrication for grittiness
Use of tinted glasses to reduce lacrimation
Papilledema, loss of visual acuity and visual field defect need emergency treatment
and are treated by systemic prednisolone (30-120 mg daily). If there is no response
orbital decompression has to be performed.
Surgical
Lateral tarsorrhaphy for corneal ulcers.
Corrective eye muscle surgery for persistent diplopia
HYPOTHYROIDISM
Complications of hypothyroidism
Cardiovascular dysfunction: Chronic hypothyroidism can result in decreased left
ventricular ejection fraction and increased risk of cardiovascular disease.
Neuropsychiatric manifestations: Hypothyroidism can result in cognitive dysfunction,
including fatigue, decreased memory and concentration, and major depressive
disorder.
Reproductive dysfunction: Women with hypothyroidism may experience menstrual
irregularities, like menorrhagia, as well as decreased fertility.
Myxedema coma: Severe, untreated hypothyroidism can lead to myxedema coma, a
life-threatening state characterized by decreased level of consciousness,
hypothermia, and respiratory failure.
Goiter: Chronic hypothyroidism can result in goiter, an enlargement of the thyroid
gland secondary to compensatory mechanisms aimed at maintaining serum T3 and
T4 levels.
A 40 years old lady comes with H/O tiredness and increased body weight. Her
TSH is 20. What is the diagnosis? Describe other clinical features associated and
its management
Primary Hypothyroidism
CLINICAL FEATURES
General
Weight gain
Lethargy
Cold intolerance
Hoarse voice
Thyroid
Enlargement of gland (Goiter)
Gastrointestinal
Reduced appetite
Constipation
Ileus
Ascites
Macroglossia
Cardiorespiratory
Angina
Bradycardia
Pericardial effusion
Pleural effusion
Dyslipidemia
Neuromuscular
Delayed relaxation of tendon reflexes (Woltman’s sign)
Carpal tunnel syndrome
Depression
Ataxia
Pseudohypertrophy of muscles
Aches and pains
Skin
Myxedema (non-pitting edema of the skin of hands and feet)
Alopecia
Vertigo
Dry flaky skin
Reproductive
Menorrhagia
Infertility
Galactorrhea
Impotence
Hematological
Macrocytosis
Anemia
Misc.
OSA
Hyponatremia
INVESTIGATIONS
Serum TSH : High TSH levels
Serum T4 : Low
Thyroid and organ-specific antibodies may be found
Increased AST from muscle or liver
Increased LDH and CK : Myopathy
Hypercholesterolemia and hypertriglyceridemia
Hyponatremia : Due to increased ADH and impaired free water clearance
Anemia : Normochromic normocytic
ECG : Sinus bradycardia, Low voltage QRS
CXR : May reveal enlarged cardiac shadow
TREATMENT
Treated with T4
Levothyroxine sodium once daily dosage (1.6-1.8 mcg/kg/day)
Starting dose
For patients without heart disease
< 50 years → 1 mcg/kg/day
> 50 years → < 50 mcg/day. Dose increased by 25 mcg, if needed at 6-8 weeks
interval
For patients with heart disease
12.5-25 mcg/day and increase by 12.5-25 mcg/day, if needed at 6-8 weeks interval
Dose adjustments
Should be taken on an empty stomach with water, ideally an hour before from
breakfast
Malabsorption requires increased dose
Pregnancy : 25-50% increase in dose
Age : In elderly start with half dose
Monitoring
Goal : To normalize TSH and to restore T4 within normal range
Monitoring done using clinical methods and thyroid function tests after 6 weeks on a
steady dose
Complete suppression of TSH should be avoided because it may cause AFib and
osteoporosis
Lifelong therapy
Treatment and monitoring of primary and secondary hypothyroidism
Treated with T4
Levothyroxine sodium once daily dosage (1.6-1.8 mcg/kg/day)
Starting dose
For patients without heart disease
< 50 years → 1 mcg/kg/day
> 50 years → < 50 mcg/day. Dose increased by 25 mcg, if needed at 6-8 weeks
interval
For patients with heart disease
12.5-25 mcg/day and increase by 12.5-25 mcg/day, if needed at 6-8 weeks interval
Dose adjustments
Should be taken on an empty stomach with water, ideally an hour before from
breakfast
Malabsorption requires increased dose
Pregnancy : 25-50% increase in dose
Age : In elderly start with half dose
Monitoring
Goal : To normalize TSH and to restore T4 within normal range
Monitoring done using clinical methods and thyroid function tests after 6 weeks on a
steady dose
Complete suppression of TSH should be avoided because it may cause AFib and
osteoporosis
Lifelong therapy
A postmenopausal lady of 60 yrs presents to the emergency department with
altered sensorium. Her Son gives history of recent weight gain of 10 Kgs. She
had also developed hoarseness of voice, intolerance to cold and constipation
over the previous one month. a. What is the most likely diagnosis? b.
Enumerate the etiology and other clinical manifestations of the above
endocrinal disease c. Briefly discuss about the management of this patient.
Myxedema coma
ETIOLOGY
Develops in patients with long-standing severe untreated hypothyroidism
Precipitating factors :
Infections
Cardiac failure
Hypoxia, hypercapnia
Hyponatremia
Drugs : (BLAND)
Beta-blockers, barbiturates
Lithium
Amiodarone, anesthetic agents
Narcotics
Diuretics
INVESTIGATIONS
Low free serum T4 levels
High TSH levels
Serum creatine phosphokinase is markedly raised
Hypoglycemia and hyponatremia
Decreased WBC and hematocrit
ECG : Low voltage, sinus bradycardia
Blood gases : Respiratory acidosis
Hypothyroidism findings
Thyroid and organ-specific antibodies may be found
Increased AST from muscle or liver
Increased LDH and CK : Myopathy
Hypercholesterolemia and hypertriglyceridemia
CXR : May reveal enlarged cardiac shadow
TREATMENT
Hypothyroidism → Large initial IV dose of 300-500 mcg T4; if no response within 48
hours, add T3
Hypocortisolemia → IV hydrocortisone 200-400 mg daily
Hypoventilation → Intubation and mechanical ventilation
Hypothermia → Gentle warming of patient with blankets, no active rewarming
Hyponatremia → Mild fluid restriction, 3% saline
Hypotension → Cautious volume expansion with crystalloid or whole blood
Hypoglycemia → Glucose administration
Precipitating event → Identification and elimination by specific treatment
Other measures
Monitor cardiac output and blood pressure
Whenever needed, cautious use of IV fluids, high flow oxygen or assisted ventilation
Myxedema coma management
INVESTIGATIONS
Low free serum T4 levels
High TSH levels
Serum creatine phosphokinase is markedly raised
Hypoglycemia and hyponatremia
Decreased WBC and hematocrit
ECG : Low voltage, sinus bradycardia
Blood gases : Respiratory acidosis
Hypothyroidism findings
Thyroid and organ-specific antibodies may be found
Increased AST from muscle or liver
Increased LDH and CK : Myopathy
Hypercholesterolemia and hypertriglyceridemia
CXR : May reveal enlarged cardiac shadow
TREATMENT
Hypothyroidism → Large initial IV dose of 300-500 mcg T4; if no response within 48
hours, add T3
Hypocortisolemia → IV hydrocortisone 200-400 mg daily
Hypoventilation → Intubation and mechanical ventilation
Hypothermia → Gentle warming of patient with blankets, no active rewarming
Hyponatremia → Mild fluid restriction, 3% saline
Hypotension → Cautious volume expansion with crystalloid or whole blood
Hypoglycemia → Glucose administration
Precipitating event → Identification and elimination by specific treatment
Other measures
Monitor cardiac output and blood pressure
Whenever needed, cautious use of IV fluids, high flow oxygen or assisted ventilation
HYPERALDOSTERONISM
Primary Hyper Aldosteronism
ETIOLOGY
Adrenal adenoma (Conn’s syndrome)
Bilateral hyperplasia of zona glomerulosa
Idiopathic
CLINICAL FEATURES
Hypertension
Tetany : Due to metabolic alkalosis
Muscle weakness : Due to hypokalemia
Polyuria and polydipsia
INVESTIGATIONS
Serum potassium : Low
Urinary potassium : High considering hypokalemic state
Plasma aldosterone concentration : Elevated and not suppressed by 0.9% saline
infusion or fludrocortisone administration
Plasma renin : Low
Plasma Aldosterone to Renin ratio : Increased (> 20)
Aldosterone suppression test
Others
Oral salt loading test : Suppression of aldosterone below 8.5 ng/dL rules out primary
hyperaldosteronism
Oral captopril test : No significant decrease in aldosterone
CT/MRI : To detect adenoma and hyperplasia
Adrenal vein catheterization : To detect hypersecretion (unilateral or bilateral)
Dexamethasone suppression
Lowers plasma aldosterone transiently in adenoma
Prolonged suppression in glucocorticoid sensitive hyperaldosteronism
Measurement of 18-OH cortisol levels
Very high in adenomas and glucocorticoid sensitive hyperaldosteronism
Slightly raised in idiopathic hyperplasia
TREATMENT
Potassium supplementation
Definitive treatment : Adrenalectomy, bilateral if multiple tumor present
Aldosterone antagonists : Spironolactone (up to 400 mg/day) and eplerenone (less
side effects).
What is Conns syndrome? Discuss clinical and lab features
Conn’s syndrome refers to aldosterone secreting adrenal adenoma
CLINICAL FEATURES
Hypertension
Tetany : Due to metabolic alkalosis
Muscle weakness : Due to hypokalemia
Polyuria and polydipsia
INVESTIGATIONS
Serum potassium : Low
Urinary potassium : High considering hypokalemic state
Plasma aldosterone concentration : Elevated and not suppressed by 0.9% saline
infusion or fludrocortisone administration
Plasma renin : Low
Plasma Aldosterone to Renin ratio : Increased (> 20)
Aldosterone suppression test
Others
Oral salt loading test : Suppression of aldosterone below 8.5 ng/dL rules out primary
hyperaldosteronism
Oral captopril test : No significant decrease in aldosterone
CT/MRI : To detect adenoma and hyperplasia
Adrenal vein catheterization : To detect hypersecretion (unilateral or bilateral)
Dexamethasone suppression
Lowers plasma aldosterone transiently in adenoma
Prolonged suppression in glucocorticoid sensitive hyperaldosteronism
Measurement of 18-OH cortisol levels
Very high in adenomas and glucocorticoid sensitive hyperaldosteronism
Slightly raised in idiopathic hyperplasia
TREATMENT
Potassium supplementation
Definitive treatment : Adrenalectomy, bilateral if multiple tumor present
Aldosterone antagonists : Spironolactone (up to 400 mg/day) and eplerenone (less
side effects).
ADRENAL INSUFFICIENCY
Addison’s Disease a. Define Addison’s disease. b. Enumerate various causes of
Addison’s disease. c. Discuss clinical features of Addison’s disease. d. How do
you manage case of Addison’s disease?
Addison’s disease or chronic adrenocortical insufficiency is an uncommon disorder
resulting from progressive destruction of the entire adrenal cortex.
ETIOLOGY
Primary adrenocortical insufficiency (adrenal causes)
Auto-immune adrenal insufficiency
Auto-immune polyglandular syndrome 1
Auto-immune polyglandular syndrome 2
Metastatic malignancy : Lung, breast, stomach carcinomas or lymphoma
Hemorrhage : Waterhouse-Friedrichsen Syndrome, Anticoagulant therapy
Infections : TB, CMV, HIB
Infarction : APLA, SLE
Infiltrative disorders : Amyloidosis, hemochromatosis, sarcoidosis
Bilateral adrenalectomy
Drugs : Ketoconazole, metyrapone
Kearns-Sayre syndrome
Secondary adrenocortical insufficiency (inadequate ACTH)
Exogenous glucocorticoid therapy
Hypopituitarism
Pituitary apoplexy
Sheehan’s syndrome
Pituitary irradiation
Granulomatous disease of pituitary (TB, sarcoidosis)
Secondary tumor deposits in pituitary (breast, bronchus)
CLINICAL FEATURES
Glucocorticoid deficiency
Fasting hypoglycemia
Increased insulin sensitivity
Muscle weakness
Morning headache
Increased production of proopiomelanocortin → Increased melanin → Pigmentation
of exposed areas, pressure areas like elbows, knees and knuckles, mucous
membranes.
Fatigue, malaise, weakness, weight loss, anorexia, nausea and vomiting
Mineralocorticoid deficiency
Hypotension
Dizziness
Salt craving
Weight loss
Anorexia
Electrolyte anomalies (hyponatremia, hyperkalemia, metabolic acidosis)
Adrenal androgen deficiency
Decreased axillary hair, pubic hair in females
Loss of libido in females
INVESTIGATIONS
8 AM cortisol levels
Levels < 3 mcg/dL suggestive of adrenal insufficiency
Levels > 11 mcg/dL exclude adrenal insufficiency
Plasma ACTH level : Elevated
ACTH stimulation test : Failure to rise in plasma cortisol level following administration
of 250 mcg of synthetic ACTH
Plasma renin activity : High
Radiograph
TB adrenalitis : CXR may show evidence of TB
Plain X-ray of abdomen, CT and MRI may show calcification of adrenal gland
Elevated BUN, hyponatremia and hyperkalemia
Blood sugar : low
Peripheral blood : Mild anemia
TREATMENT
Primary adrenal insufficiency
Acute treatment
NS for volume resuscitation
Look for / treat hypoglycemia by 25% dextrose
Steroids
Loading dose : 50-100 ��/�2mg/m2 hydrocortisone IV/IM
Continue hydrocortisone with 50-100 ��/�2/���mg/m2/day divided 6th or
8th hourly
Long term treatment
Daily glucocorticoid replacement : Hydrocortisone 10-15
��/�2/���mg/m2/day
Daily mineralocorticoid replacement : Fludrocortisone 0.05-0.20 mg/day
Stress conditions
Main goal : To avoid serious consequences of adrenal crisis
Illness
Minor stress (e.g. sore throat, rhinorrhea, T < 38 C) : May not require increase in dose
Moderate stress (e.g. severe URTI) : Double the glucocorticoid replacement dose
Major stress (e.g. T > 38 C and/or vomiting) : Three or four times the glucocorticoid
replacement dose
Surgery
Requirement of GC increases drastically during general anesthesia
For major surgical procedures : 100-150 mg hydrocortisone for major surgical
procedures in divided doses
TB adrenalitis : Treated with antitubercular therapy
Discuss the etiology, clinical features and management of adrenocortical
insufficiency
ETIOLOGY
Primary adrenocortical insufficiency (adrenal causes)
Auto-immune adrenal insufficiency
Auto-immune polyglandular syndrome 1
Auto-immune polyglandular syndrome 2
Metastatic malignancy : Lung, breast, stomach carcinomas or lymphoma
Hemorrhage : Waterhouse-Friedrichsen Syndrome, Anticoagulant therapy
Infections : TB, CMV, HIB
Infarction : APLA, SLE
Infiltrative disorders : Amyloidosis, hemochromatosis, sarcoidosis
Bilateral adrenalectomy
Drugs : Ketoconazole, metyrapone
Kearns-Sayre syndrome
Secondary adrenocortical insufficiency (inadequate ACTH)
Exogenous glucocorticoid therapy
Hypopituitarism
Pituitary apoplexy
Granulomatous disease of pituitary (TB, sarcoidosis)
Secondary tumor deposits in pituitary (breast, bronchus)
Sheehan’s syndrome
Pituitary irradiation
CLINICAL FEATURES
Glucocorticoid deficiency
Fasting hypoglycemia
Increased insulin sensitivity
Muscle weakness
Morning headache
Increased production of proopiomelanocortin → Increased melanin → Pigmentation
of exposed areas, pressure areas like elbows, knees and knuckles, mucous
membranes.
Fatigue, malaise, weakness, weight loss, anorexia, nausea and vomiting
Mineralocorticoid deficiency
Hypotension
Dizziness
Salt craving
Weight loss
Anorexia
Electrolyte anomalies (hyponatremia, hyperkalemia, metabolic acidosis)
Adrenal androgen deficiency
Decreased axillary hair, pubic hair in females
Loss of libido in females
INVESTIGATIONS
8 AM cortisol levels
Levels < 3 mcg/dL suggestive of adrenal insufficiency
Levels > 11 mcg/dL exclude adrenal insufficiency
Plasma ACTH level : Elevated
ACTH stimulation test : Failure to rise in plasma cortisol level following administration
of 250 mcg of synthetic ACTH
Plasma renin activity : High
Radiograph
TB adrenalitis : CXR may show evidence of TB
Plain X-ray of abdomen, CT and MRI may show calcification of adrenal gland
Elevated BUN, hyponatremia and hyperkalemia
Blood sugar : low
Peripheral blood : Mild anemia
TREATMENT
Primary adrenal insufficiency
Acute treatment
NS for volume resuscitation
Look for / treat hypoglycemia by 25% dextrose
Steroids
Loading dose : 50-100 ��/�2mg/m2 hydrocortisone IV/IM
Continue hydrocortisone with 50-100 ��/�2/���mg/m2/day divided 6th or
8th hourly
Long term treatment
Daily glucocorticoid replacement : Hydrocortisone 10-15
��/�2/���mg/m2/day
Daily mineralocorticoid replacement : Fludrocortisone 0.05-0.20 mg/day
Stress conditions
Main goal : To avoid serious consequences of adrenal crisis
Illness
Minor stress (e.g. sore throat, rhinorrhea, T < 38 C) : May not require increase in dose
Moderate stress (e.g. severe URTI) : Double the glucocorticoid replacement dose
Major stress (e.g. T > 38 C and/or vomiting) : Three or four times the glucocorticoid
replacement dose
Surgery
Requirement of GC increases drastically during general anesthesia
For major surgical procedures : 100-150 mg hydrocortisone for major surgical
procedures in divided doses
TB adrenalitis : Treated with antitubercular therapy
CUSHING’S SYNDROME & RELATED
Describe the causes, clinical manifestations and management of Cushing's
syndrome. // Etiology and diagnosis of Cushing's syndrome
Cushing’s syndrome is the term used to describe the clinical state of increased free
circulating glucocorticoid.
Causes of Cushing’s syndrome
ACTH dependent causes
ACTH-secreting pituitary tumor (Cushing’s disease) — 65%
Non-pituitary ACTH-secreting neoplasm — 10%
Pituitary CRH-secreting neoplasm
ACTH independent causes
Adrenal adenoma
Adrenal carcinoma
Micronodular adrenal disease
McCune Albright syndrome
Massive macronodular adrenal disease
Iatrogenic (use of corticosteroids)
Clinical Manifestations
General : Weight gain, central obesity, buffalo hump, moon face, hypertension
Skin : Hirsutism, plethoric appearance, purple striae over abdomen, bruising
Musculoskeletal : Back pain, muscle weakness
Gonadal dysfunction : Menstrual disorders, decreased libido and impotence
Neuropsychiatric : Emotional lability, depression, euphoria, psychosis, irritability
Metabolic : Glucose intolerance, diabetes, hyperlipidemia, polyurea, kidney stones
Skin pigmentation with ACTH dependent causes
Investigations

To confirm the presence or absence of Cushing’s syndrome


48 hour low dose dexamethasone test : Failure to suppress plasma cortisol levels
24 hour free urinary cortisol measurements : Simple but less reliable
Plasma cortisol levels : Circadian rhythm lost. Remain elevated throughout the day.
Overnight dexamethasone suppression test
Late-night salivary cortisol : Screening test. Elevated in patients with Cushing’s
syndrome
To determine the cause
Plasma ACTH level at 8 AM
Low ACTH (< 10 ng/L) : Adrenal tumors, macronodular adrenal hyperplasia or
exogenous administration
Normal levels (10-80 ng/L) : Pituitary
Intermediate levels (80-300 ng/L) : Pituitary dependent disease or ectopic ACTH
Very high levels (>300 ng/L) : Ectopic ACTH syndrome
High dose dexamethasone suppression test
Lack of suppression → Adrenal Cushing’s syndrome (CT adrenals), Ectopic ACTH
syndrome (CT abdomen and chest)
Suppression present → Pituitary Cushing’s syndrome (MRI pituitary)
Plasma potassium levels
Normal in pituitary disease and tumors of adrenal glands
Low in ectopic ACTH syndrome
Bilateral inferior petrosal sinus sampling for ACTH before and after CRH stimulation
Other investigations
Biochemical : Blood glucose, cholesterol and LDL may be raised
Radiological
Plain radiograph of the skull. Radiograph of chest to detect lung cancer
CT scan of anterior mediastinum, upper abdomen and pancreas to rule out tumors
Management
Adrenal tumors
Surgical resection
Adrenal adenomas : Surgical removal. Postoperatively, prednisolone is given as a
replacement therapy till the contralateral adrenal, hypothalamus and pituitary
recover.
Adrenal carcinomas : Surgical resection, irradiation of tumor bed and administration
of adrenolytic drug (mitotane)
Medical adrenalectomy : Ketoconazole (200-400 mg TID), metyrapone (600-4500
mg/day), mitotane, octreotide inhibit steroidogenesis
Cushing’s disease
Treatment of choice : Trans-sphenoidal removal of tumor
Recurrent / Residual ACTH secreting tumors : Radiotherapy and radiosurgery
Medical therapy to reduce ACTH : e.g. bromocriptine, cabergoline, pasireotide
Bilateral adrenalectomy
Done if diagnosis is uncertain.
Followed by pituitary irradiation using Yttrium-90 implantation to prevent the
development of Nelson’s syndrome (enlargement of ACTH-secreting tumor after
bilateral adrenalectomy)
Ectopic ACTH syndrome
Surgical removal of benign tumors (e.g. bronchial carcinoid)
Radiotherapy and Chemotherapy : For malignant tumors
Four side effects of long term corticosteroid use // Adverse effects of corticosteroids
DIABETES MELLITUS
A 45 year old hypertensive is recently detected to have random blood sugar of
250mg/dl. Discuss the clinical features and complications that you will look for,
and investigations which are essential in him. How will you treat this patient? //
Discuss clinical presentation, investigations and treatment of newly detected
type 2 diabetes mellitus
Diabetes mellitus.
Clinical features -
Complications
Acute
Diabetic Ketoacidosis
Hyperglycemic Hyperosmolar State
Hypoglycemia
Somogyi Phenomenon
Dawn Phenomenon
Chronic
Diabetic Nephropathy
Diabetic retinopathy
Atherosclerosis
Myocardial infarction
Gangrene of upper/lower extremities
Renal vascular insufficiency
Cerebrovascular accidents
Diabetic neuropathy
Investigations
Treatment

Treatment

Examples of the different classes of antidiabetic agents -


Thiazolidinediones → Rosiglitazone, pioglitazone
DPP4 inhibitors → Linagliptin. Saxagliptin, Sitagliptin
MOA → Prevent GLP-1 breakdown
GLP 1 agonists → Exenatide, liraglutide
MOA → Decreased glucagon secretion and gastric emptying
Sulfonylurea → glyburide, glimepiride, and glipizide
MOA → Bind ATP dependent K channels on insulin secreting cells in pancreas
thereby increasing release
Meglitinides → Repaglinide and nateglinide
MOA → Bind ATP dependent K channels on insulin secreting cells in pancreas
thereby increasing release
SGLT2 inhibitors → Canagliflozin
MOA → Prevent glucose reabsorption in the PCT thereby increasing excretion
Skin manifestations in Diabetes

Diagnostic criteria for Diabetes mellitus


Lifestyle measures (non-pharmacological management) in Type 2 Diabetes Mellitus
Incretin based therapy for type 2 diabetes mellitus
Indications for insulin in diabetes mellitus
Insulin dosing regimes
DPP4 Inhibitors // Gliptins
Metformin

Treatment of hypoglycemia episode in a diabetic patient


Clinical Manifestations of Hypoglycemia
Occur following a missed meal, too much insulin or unexpected exercise
Two main categories of symptoms
Adrenergic symptoms : Sweating, tremor, tachycardia, anxiety and hunger
Predominate when hypoglycemia is sudden in onset (e.g. insulin reactions)
May not manifest in long standing diabetics due to severe neuropathy
CNS dysfunction : Dizziness, headache, clouding of vision, blunted mental acuity, loss
of fine motor skill, confusion, abnormal behavior, convulsions, loss of consciousness
Predominate when hypoglycemia is of gradual onset.
Daytime hypoglycemic events : sweating, nervousness, tremors and hunger
Nocturnal hypoglycemic events : night sweats, unpleasant dreams, early morning
headache
DIABETIC NEUROPATHY
Discuss the classification, clinical features, investigation and management of
diabetic neuropathy // Management of diabetic neuropathy
Classification
Polyneuropathy
Symmetrical : Sensory and distal
Asymmetrical : Motor and proximal
Polyradiculopathy
Lumbar polyradiculopathy (Diabetic amyotrophy)
Thoracic polyradiculopathy
Mononeuropathies
Cranial mononeuropathy
Peripheral mononeuropathy
Autonomic neuropathy
Cardiovascular
Gastrointestinal
Clinical Features
Distal Symmetric polyneuropathy
Most common type of diabetic neuropathy
Progressive loss of distal sensation
Followed, in severe cases by motor weakness
Symptoms -
paresthesia in the feet and hands
pain in lower limbs
burning sensation in the soles of feet
numbness in the feet
Examination
Loss of vibration sensation
Altered proprioception
Loss of tendon relaxes
Muscle weakness and wasting (advanced cases)
Polyradiculopathy
Lumbar polyradiculopathy (Diabetic amyotrophy)
Most common type of polyradiculopathy in diabetic individuals
Involves L2, L3, L4 nerve roots
Symptoms
Thigh pain
Proximal weakness and wasting in one leg
Thoracic polyradiculopathy
T4 to T12 involvement
Abdominal pain in band like pattern
Mononeuropathies
Severe and of rapid onset
Usually recover
Cranial mononeuropathy
Tolosa hunt syndrome : 3, 4, 6 cranial nerves affected
Bell’s palsy : 7th nerve affected
Peripheral mononeuropathy
Median, femoral and sciatic nerves are commonly involved
Median nerve due to carpal tunnel syndrome
Lateral popliteal nerve compression causes foot drop
Autonomic neuropathy
Cardiovascular
Postural hypotension
Resting tachycardia
Fixed heart rate
Gastrointestinal
Dysphagia
Gastroparesis : Abdominal fullness, nausea and vomiting, delayed gastric emptying
Constipation, diarrhea, incontinence
Genitourinary
Bladder dysfunction
Erectile dysfunction
Retrograde ejaculation
Pupillary
Decreased pupil size
Resistance to mydriatics
Delayed or absent reflexes to light
Management
Good control of diabetes
Pain control —neuropathic pain can be controlled by following drugs
TCAs : Amitriptyline
Anticonvulsants : Gabapentin, carbamazepine, phenytoin, pregabalin
Opiates : Tramadol, oxycodone
Duloxetine
Autonomic neuropathy —
Postural hypotension
Full length elastic stockings
Increasing salt intake
Fludrocortisone
Midodrine
Gastroparesis : Prokinetic agents such as metoclopramide and domperidone
Diarrhea : Diphenoxylate, loperamide and broad spectrum antibiotics
Constipation : stimulant laxatives (senna)
Bladder dysfunction : Intermittent self-catheterization
Erectile dysfunction : PDE-5 inhibitors (sildenafil, tadalfil)
Treatment to reverse neuropathy
Alpha-lipoic acid
Carnitine
Methyl-cobalamin
Ruboxistaurin
Mention six causes of peripheral neuropathy and write the treatment of Diabetic
neuropathy
Causes of peripheral neuropathy
Treatment of Diabetic Neuropathy
<answered above>
DIABETIC NEPHROPATHY
Management of diabetic nephropathy
Diagnosis
Screening for microalbuminuria
Measurement of albumin-to-creatine ratio on spot urine test. A ratio > 30 mg
albumin/g creatine is consider elevated
Albumin from a 24 hour urine collection (>30 mg/day is diagnostic)
Renal function tests — urea, creatine may be elevated in advanced disease
USG : To know the size and any parenchymal changes
Prevention
Meticulous control of diabetes can reverse microproteinuria in some patients
Stop smoking and control dyslipidemia
Management
ACE inhibitors or Angiotensin Receptor Blockers can retard the progression at this
stage
Avoid oral hypoglycemic agents partially excreted via kidney (e.g. glibenclamide and
metformin)
Aggressive control of hypertension
The BP should be decreased to less than 140/90 mmHg
Aldosterone inhibitors can be added with ACE inhibitors
Low protein diets (0.8 g/kg/day) may be beneficial
Once the azotaemic phase is reached the management is similar to other forms of
chronic renal failure
DIABETIC KETOACIDOSIS
A 35 year old male patient with type I diabetes mellitus on irregular treatment
presented to the casualty with pain abdomen and vomiting of 2 days duration.
On examination patient had dehydration, tachycardia and hypotension with
deep sighing respiration. a. What is the likely diagnosis and what investigations
will confirm the diagnosis? b. Outline the management of above patient. c.
Enumerate all the acute and long term complications of diabetes mellitus
Diabetic ketoacidosis
Complications
35 years old lady comes with acute onset of vomiting, pain abdomen of one
day duration to the casualty. Also gives history of mild fever, known diabetic on
insulin therapy. Diagnosis and Management of this patient.
Biochemical parameters of diabetic ketoacidosis
Plasma Glucose > 250 mg/dL
Arterial pH < 7.3
Serum bicarbonate < 15 mEq/L
Anion gap > 12 mEq
Ketonemia and Ketonuria
Plasma K levels are normal or slightly raised despite the total body deficit due to
metabolic acidosis which shift K from intracellular compartment to extracellular
compartment
Acute complications of Diabetes Mellitus and treatment of any one of them
55 year old Diabetic male comes with fever, chest pain and cough of 4 days
duration. He also has vomiting and abdominal pain of 1 day duration. On
examination he has left upper lobe pneumonia. Investigation: RBS=454mg/dl,
Creatinine=1.8 mg/dl, Potassium = 5.9, pH=7.12, HCO3=12, urine ketones -
positive. a. What is your complete diagnosis? b. What are the Electrolyte
abnormalities in this patient? c. How will you manage this patient?
VITAMIN DEFICIENCIES
Beriberi
Occurs due to thiamine (B1) deficiency
Thiamine is responsible for carbohydrate metabolism and nerve impulse conduction
Clinical features
Infantile beriberi : Exclusively breastfed infants of thiamine deficient mothers and is
invariably fatal
Dry beriberi :
Affects nervous system
Peripheral neuropathy
Wrist/foot drop
Wernicke’s disease : ophthalmoplegia and ataxia along with confusion
Korsakoff’s psychosis : impaired short-term memory with confabulation
Wet (Oriental) beriberi
Congestive cardiac failure
Pulmonary edema
Peripheral edema (due to lactate and pyruvate)
High output cardiac state with warm extremities
Extreme tachycardia
Venous congestive state with raised JVP and tender hepatomegaly
Dyspnea and circulatory collapse
Marked cardiomegaly
Treatment
Thiamine at a dose of 50-100 mg for 7-14 days IV or IM. Then an oral dose of 10 mg
per day till full recovery.
Korsakoff’s psychosis is irreversible and does not respond to thiamine
Wet Beriberi

Pellagra (x2)
Mention the normal levels of Vitamin D in adults, and treatment of its deficiency
Normal Levels → 20 - 40 ng/mL
Treatment of Vitamin D deficiency →

Clinical features, diagnosis and treatment of Vit D deficiency in adults


OTHERS
Treatment of Polycystic Ovarian Syndrome
Milk alkali syndrome
Etiology → Excessive intake of calcium and absorbable antacids such as calcium
carbonate and milk
Acute form
Triad of hypercalcemia, metabolic alkalosis and renal failure
Treatment with rehydration and if necessary dialysis
Chronic form (Burnett Syndrome)
Soft tissue calcification in kidney and nephrocalcinosis
Progressive renal insufficiency
Treatment options for obesity
45 years old lady with a waist hip ratio of >90; TC/HDL: >6.0; BP >160/90 mmHg,
HbA1c: 7.6. Interpret the report and importance and management
Hypertriglyceridemia

Immunology, Connective Tissue Disorders


and Rheumatology
SYSTEMIC LUPUS ERYTHEMATOSUS
20 year old lady presented with multiple joint pains, photosensitivity rash since 4
months. How do you approach her problem? What investigations will help in
confirming diagnosis?
The clinical features are suggestive of SLE
Investigations
Antibodies
Highly specific antibodies for SLE
Anti dsDNA antibodies - suggestive of renal involvement
Anti-smith antibodies
Highly sensitive antibody
Antinuclear antibodies
Drug induced lupus
Anti histone antibodies
Other tests
CBC → Anemia, leucopenia and thrombocytopenia
aPTT → Prolonged in presence of antiphospholipid antibodies
ESR - rises with disease activity
CRP - usually does not rise with disease activity unless there is infection
Complement levels - low C3
Imaging
CXR
HRCT - fibrotic lung
MRI - spinal cord or brain involvement
Clinical features of Systemic Lupus Erythematosus (x2)

Mnemonic: RASH OR PAIN


R: Malar / Discoid rash
A: Arthritis
S: Serositis (Pericarditis, Pleuritis), Libman-Sack endocarditis
H: Hematological disorder (Pancytopenia)
O: Oral or Nasopharyngeal ulcers
R: Renal manifestations (lupus nephritis)
P: Photosensitivity
A: Anti-nuclear antibodies
I: Immune phenomenon (anti-dsDNA, anti-Sm, antiphospholipid)
N: Neurological Symptoms (seizures etc.)
Diagnostic criteria for SLE (x3)
Presence of 4 of the following 11 yields a sensitivity of 85% and a specificity of 95%
for SLE.

Mnemonic: RASH OR PAIN


R: Malar / Discoid rash (Two separate criteria)
A: Arthritis
S: Serositis (Pericarditis, Pleuritis), Libman-Sack endocarditis
H: Hematological disorder (Pancytopenia)
O: Oral or Nasopharyngeal ulcers
R: Renal manifestations (lupus nephritis)
P: Photosensitivity
A: Anti-nuclear antibodies
I: Immune phenomenon (anti-dsDNA, anti-Sm, antiphospholipid)
N: Neurological Symptoms (seizures etc.)
Complications of Systemic lupus erythematosus
Libman-Sack Endocarditis
Pericarditis
Pleural Effusion
Anemia
Recurrent infections
Increased atherosclerosis leading to coronary artery disease
Hypercoagulability leading to stroke, TIA
Epilepsy
Renal failure
Mention three clinical features and diagnosis of antiphospholipid antibody syndrome
Clinical features
Deep vein thrombosis
Livedo reticularis
Pulmonary embolism
Superficial thrombophlebitis
Stroke, TIA
Libman-Sacks endocarditis
Renal artery, vein or glomerular thrombosis
Recurrent pregnancy losses
Diagnosis
Antiphospholipid antibodies
Lupus anticoagulant
Anti-cardiolipin
Anti-beta-2-glycoprotein-1
False positive VDRL due to anticardiolipin
Falsely elevated aPTT due to lupus anticoagulant
RHEUMATOID ARTHRITIS
Four differential diagnoses for wasting of small muscles of hand
Spinal cord lesions : Motor neuron disease, syringomyelia, intramedullary tumors, C8,
T1 lesions (cervical spondylosis, trauma)
Medial cord lesions of brachial plexus : Pancoast tumor, metastasis, trauma, thoracic
outlet syndrome
Median nerve lesions : Trauma, carpal tunnel syndrome, vasculitis
Ulnar nerve lesions : Trauma, entrapment, leprosy, vasculitis
Muscle disease : Focal amyotrophy
American rheumatology criteria for the diagnosis of Rheumatoid arthritis.

Mention four commonly used disease modifying anti-rheumatic drugs with dose
Methotrexate
2.5 - 7.5 mg/week as a single dose orally
If there is no positive response within 4–8 weeks, and there is no toxicity, the dose
should be increased by 2.5–5 mg/week each month to 15–25 mg/week before
considering, the treatment a failure
Hydroxychloroquine - 200-400 mg daily
Sulfasalazine - 1-3 g daily
Leflunomide - 10-20 mg daily
Treatment of rheumatoid arthritis (x3)
NSAIDs - symptomatic relief. Do not alter the disease process
Glucocorticoids -
They provide symptomatic effect as well as reduce disease progression but their long
term usage is associated with many toxicities
Prednisolone is the most commonly used drug
In ‘bridge therapy’, glucocorticoids are used first to shut off inflammation rapidly and
then to taper them as the DMARD is taking effect
Disease modifying antirheumatic drugs (DMARDs) - They have the ability to greatly
inhibit disease process and to modify the disabling potential of RA
Conventional DMARDs
Methotrexate
2.5 - 7.5 mg/week as a single dose orally
If there is no positive response within 4–8 weeks, and there is no toxicity, the dose
should be increased by 2.5–5 mg/week each month to 15–25 mg/week before
considering, the treatment a failure
Hydroxychloroquine - 200-400 mg daily
Sulfasalazine - 1-3 g daily
Leflunomide - 10-20 mg daily
Biologic DMARDs
They are usually given along with methotrexate or other conventional DMARDs.
Disadvantages: High cost and long-term toxicities, notably infections and
demyelinating syndromes
TNF-alpha blocker
Infliximab
Etanercept
Adalimumab
IL-1 receptor blocker - Anakinra
Anti CD20 - Rituximab
JAK inhibitors - Baricitinib, Tofacitinib
Disease modifying drugs in Rheumatoid Arthritis (x3)
Disease modifying antirheumatic drugs (DMARDs) - They have the ability to greatly
inhibit disease process and to modify the disabling potential of RA
Conventional DMARDs
Methotrexate
2.5 - 7.5 mg/week as a single dose orally
If there is no positive response within 4–8 weeks, and there is no toxicity, the dose
should be increased by 2.5–5 mg/week each month to 15–25 mg/week before
considering, the treatment a failure
Hydroxychloroquine - 200-400 mg daily
Sulfasalazine - 1-3 g daily
Leflunomide - 10-20 mg daily
Biologic DMARDs
They are usually given along with methotrexate or other conventional DMARDs.
Disadvantages: High cost and long-term toxicities, notably infections and
demyelinating syndromes
TNF-alpha blocker
Infliximab
Etanercept
Adalimumab
IL-1 receptor blocker - Anakinra
Anti CD20 - Rituximab
JAK inhibitors - Baricitinib, Tofacitinib
Non biological DMARDs in rheumatoid arthritis with dose and side effects
Conventional DMARDs
Methotrexate
2.5 - 7.5 mg/week as a single dose orally
If there is no positive response within 4–8 weeks, and there is no toxicity, the dose
should be increased by 2.5–5 mg/week each month to 15–25 mg/week before
considering, the treatment a failure
Side effects → GI intolerance, stomatitis, hepatotoxicity, hyperhomocysteinemia
Hydroxychloroquine - 200-400 mg daily
Side effect - Bull’s eye maculopathy
Sulfasalazine - 1-3 g daily
Can trigger sulfa drug allergies
Leflunomide - 10-20 mg daily
Side effect - teratogenic
Biologicals in Rheumatoid arthritis
Biologic DMARDs
They are usually given along with methotrexate or other conventional DMARDs.
Disadvantages: High cost and long-term toxicities, notably infections and
demyelinating syndromes
TNF-alpha blocker
Infliximab
Etanercept
Adalimumab
IL-1 receptor blocker - Anakinra
Anti CD20 - Rituximab
JAK inhibitors - Baricitinib, Tofacitinib
Extra articular manifestations of Rheumatoid arthritis (x3)
Constitutional → Fatigue, weight loss and low grade fevers
Rheumatoid nodules → Firm non-tender nodules on extensor surfaces
Rheumatoid vasculitis → Digital infarcts, cutaneous ulcerations, palpable purpura,
distal gangrene
Pulmonary manifestations
Rheumatoid nodules
Diffuse interstitial fibrois
Bronchiolitis obliterans
Pulmonary arterieis
Caplan’s syndrome
Cardiac manifestations
Pericardial effusions
CAD due to premature atherosclerosis
Neurological manifestations
Peripheral nerve entrapment syndromes - Carpal tunnel syndrome, Tarsal tunnel
syndrome
Myelopathy
Ophthalmological manifestations
Sicca complex
Scleritis
Felty’s syndrome → RA, splenomegaly and neutropenia
Malignancies → Increased risk of lymphomas
Role of DMARDS in Rheumatic conditions
SERONEGATIVE SPONDYLOARTHROPATHIES
Describe the salient features of seronegative spondyloarthropathies. Mention the
types of arthritis grouped under spondyloarthropathies. // Name four seronegative
arthritic disorders and two clinical features of seronegative arthritis.

Clinical features of psoriatic arthritis


Psoriatic arthritis is a chronic, inflammatory, seronegative arthritis characterized by
asymmetric polyarthritis, enthesitis and dactylitis. It is considered to be a subtype of
spondyloarthritis.
Clinical Features:
Articular features
Asymmetric oligoarthritis
DIP joints of fingers and toes are especially affected
Hand deformity results from tenosynovitis and soft tissue contractures
Knees are affected with large effusions
Extra-articular features
Enthesitis : Affects Achilles’ tendon, plantar fascia and pelvic bones
Dactylitis : Sausage digits
Skin lesions : Plaques with silvery white scales
Nail changes : Coarse, irregular pitting, onycholysis, nailbed hyperkeratosis and
splinter hemorrhages
Investigations
ESR/CRP elevated
Rheumatoid factor and ANA usually negative
X-ray : Erosive changes with new bone formation in distal joints
MRI : Soft tissue inflammation
Treatment
General measures : Regular exercise and attention to posture in those with
spondylitis
NSAIDs : First choice for pain relief and control inflammation
Second line drugs : Sulfasalazine, methotrexate or azathioprine. HCQ should be
avoided as it can exacerbate skin lesions
Surgery for severe joint destruction and contractures
A 20 year old male patient with history of dysentery 2 weeks prior, presented with
complaints of pain and swelling of the lower limb joints. How do you approach this
patient?
The history suggests of reactive arthritis
History
Ask for the history of following symptoms
Duration and severity of pain, joints affected. Any history of lower back pain
(sacroiliitis)
Sexual exposure or attack of dysentery in the past 1 to 3 weeks
Any urinary problems
Any eye problems - redness of eyes, discharge, blurry vision, photophobia
Problems related to external genitalia - painless dermatitis of glans penis (circinate
balanitis)
Any buccal ulcers
General features like fever, fatigue and weight loss
Examination
Look for the following signs -
Hyperkeratotic vesicles on the palms and soles (keratoderma blennorrhagica)
Examine the external genitalia for circinate balanitis
Examination of the eye
Investigations
ESR / CRP - raised
CBC : Normocytic normochromic anemia
Synovial fluid analysis : Low viscosity, turbid appearance, giant macrophages (Reiter’s
cells)
Urine culture : May be positive for N. gonorrhea, C. trachomatis
Stool culture : May show causative organisms but usually becomes negative by the
time arthritis develops
Serological testing : Detection of antibodies against the organism may help confirm
previous dysentery
Rheumatoid factor and ANA are negative
X-ray : No features initially. Chronic disease may show periarticular osteopenia, joint
space narrowing, and marginal proliferative erosions.
Management
NSAIDs to control the pain and inflammation of arthritis
Steroids - intra-articular or local corticosteroid injections are helpful in relieving the
symptoms of arthritis or enthesitis.
Sulfasalazine has been shown to be effective in reactive arthritis. Azathioprine and
methotrexate may be required in severe progressive arthritis and intractable
keratoderma blennorrhagica
Chlamydial infection is treated with doxycycline. N. gonorrhea infection is treated
with benzathine penicillin G
GOUTY ARTHRITIS
Acute gouty arthritis // Clinical features and treatment of acute gout // Treatment of
gouty arthritis
Acute gouty arthritis usually follows decades of asymptomatic hyperuricemia. Usually
the first attack occurs in a middle-aged male
Presentation: It appears as sudden excruciating (agonizing) joint pain, swelling,
intense redness and tenderness of the first metatarsophalangeal (MTP) joint (termed
podagra)
Systemic features: The acute attack may be accompanied by fever, chills, and malaise,
leukocytosis and raised ER
Precipitating factors: high-purine foods, alcohol ingestion, starvation, dehydration
certain drugs (e.g. diuretic, salicylates, urate lowering drugs allopurinol and
radiographic contrast agents), trauma and surgery.
Even if not treated, acute attacks of gout are usually self-limited, subside
spontaneously and first several acute attacks last for 5 to 8 days
Treatment of acute attack
NSAIDs - Indomethacin (75 mg immediately, then 50 mg 6–8 hours)
Local ice packs for symptomatic relief
Oral colchicine - dose of 1.2 mg immediately, then 0.6 mg every 6–12 hours
Corticosteroids: If patient cannot tolerate NSAIDs or colchicine, oral prednisolone (20
30 mg/day) or intramuscular or intra-articular depot methylprednisolone may be
tried.
ACTH gel: A single injection of intramuscular ACTH gel (25 to 80 IU) is useful for
terminating an acute attack of gout.
Prophylaxis
Dietary advice - Avoid alcohol intake especially bear which is rick in purines and
fructose
Avoid meat and shellfish
Controlled weight reduction
Drugs
Xanthine Oxidase inhibitors - Decrease production
Allopurinol 300–900 mg daily
Febuxostat 40–120 mg/day
Uricase analogs - Increase metabolism
Pegloticase
Rasburicase
Uricosuric agents - Increased excretion
Probenecid
Benzbromarone
Lesinurad
Sulfinpyrazone
MYASTHENIA GRAVIS
Myasthenic crises
A rapid and severe deterioration of myasthenia is called ‘myasthenic crisis’ can bring
patient to the brink of respiratory failure and quadriparesis in hours.
A respiratory infection or a sedative medication with NM (neuromuscular) block may
be the reason.
It can develop at any time after the diagnosis of myasthenia.
Anticipate if patient is restless, anxious with diaphoresis and develops tremor.
Require respiratory support
Diagnosis and treatment of myasthenia gravis // Treatment of myasthenia gravis
Diagnosis
Serological tests → Antibodies against AChR or MuSK (muscle specific receptor
tyrosine kinase)
Pharmacological tests
Anticholinesterase test → Drugs inhibiting AChE allow ACh to interact repeatedly
with limited number of AChRs producing strength improvement in myasthenic
muscles
Edrophonium (Tensilon) test
Edrophonium is a rapidly acting acetylcholinesterase inhibitor
Onset of action is rapid (30 seconds) and lasts for short duration (5 minutes)
When the test is positive, it produces substantial improvement in weakness, ptosis,
diplopia, nasal voice, etc. within 30 seconds and lasts for up to 5 minutes. Positive
test is highly suggestive of MG
Neostigmine/Prostigmine test
Ice on eyes/Ice pack test: Apply an ice pack on eye for 3 to 5 minutes. The response
is positive when there is increase in at least 2 mm of the palpebral fissure from
before to after the test.
Electrodiagnostic tests
Repeated nerve stimulation - Rapid reduction in the amplitude of the evoked
response during repeated stimulation
Single fiber electromyogram
Treatment
Oral cholinesterase - Help weakness but do not change the natural history of
myasthenia
Pyridostigmine: Onset of action is within 15–30 minutes and lasts for 3–4 hours. Dose
30–60 mg/3 to 4 times daily
Thymectomy: : It is an expert surgical procedure that should be carried out in all
patients with generalized MG between the ages of puberty and at least 55 years.
Immunosuppression: IVID, plasmapheresis, glucocorticoids, cyclosporine, tacrolimus,
azathioprine, mycophenolate mofetil
VASCULITIS
Polyarteritis nodosa (x2)
Systemic, necrotizing vasculitis of small and medium-sized muscular arteries that
spares the smallest blood vessels (e.g. arterioles, venules or capillaries).
It characteristically involves the renal and visceral arteries
Unique features
Confinement of the disease to the arterial rather than the venous circulation
Absence of granulomatous inflammation
Clinical features
Peak incidence between 40 to 50 years of age
Subacute onset of constitutional vague symptoms such as fever, weight loss,
weakness, malaise and arthralgias are present in > 50% of cases
Absence of pulmonary artery involvement helps distinguish PAN from most cases of
vasculitis.
Diagnosis
Anemia, thrombocytosis, and raised acute phase reactants
p-ANCA: Present in about 20% of cases
Hepatitis B surface antigen (HBsAg): It may be positive
Arteriograms: Demonstrates aneurysms of small and medium-sized arteries in the
kidney, liver, and visceral vasculature.
Biopsy → Characteristic vasculitis with fibrinoid necrosis
Treatment
Glucocorticoids and cytotoxic agents (Cyclophosphamide)
PAN with Hep B - Antiviral therapy used in combination with glucocorticoids and
plasma exchange
Enumerate common vasculitis. Mention treatment of Giant cell arteritis
Treatment of Giant Cell Arteritis

Serological tests for systemic vasculitis


OSTEOPOROSIS
Osteoporosis // Treatment of osteoporosis
Osteoporosis is defined as a bone mineral density (BMD) of 2.5 standard deviations
(SDs) below the young healthy adult mean value (T-score ≤−2.5) or lower as
measured by DEXA (dual-energy X-ray absorptiometry).
Clinical features
Risk of fractures
Pathological fractures
Spine - Vertebral crush fractures
Hip fractures
Wrist fracture - Colles’ fracture
Investigation
Plain radiograph
Bone mineral density
DEXA scan
Quantitative ultrasound
Quantitative CT scan
Treatment
General measures
Sufficient calcium intake (1200-1500 mg/day) and Vitamin D
Cessation of smoking
Reduction in alcohol intake
Increased weight bearing exercise
Pharmacological
Antiresorptive drugs
Bisphosphonates - Alendronate, Pamidronate, Zoledronate, Etidronate
Calcitonin - directly inhibits osteoclastic bone resorption activity
Raloxifene - Selective estrogen receptor modulator
Bone forming agents
Teriparatide - acts like parathyroid hormone and stimulates osteoblasts
Others
Denosumab - RANKL inhibitor - prevents osteoclast activation
Strontium ranelate - prevents resorption as well as increases bone formation
Dual energy Xray absorptiometry (DEXA) scan
It is the most commonly used technique to measure BMD.
Two low dose X-rays are directed to the area of interest.
It measures areal bone density (mineral/surface area rather than a true volumetric
density), usually spine (lumbar) and hip (proximal femur).
It uses low doses of radiation and is the ‘gold standard’ for the diagnosis of
osteoporosis.
Results from DXA are expressed as either the T-score (with reference to normal bone
density in a younger individual of same sex and race but) or the Z-score (with
reference to normal bone density in an individual of same age, sex and race)
ANAPHYLAXIS
A patient was given penicillin injection, soon he became anxious and developed
difficulty in breathing, light headedness, palpitation and dizziness. He was pale and
had low blood pressure. What is the diagnosis and how will you manage the patient?
Systemic anaphylaxis due to penicillin
Clinical features
Cutaneous - Itching, Hives, Angioedema, Flushing
Respiratory - Bronchospasm, dyspnea and wheeze, laryngeal edema, pulmonary
edema
Gastrointestinal - Nausea, vomiting, abdominal cramps, diarrhea
Cardiovascular - tachycardia, hypotension, arrhythmias, shock and collapse
Central Nervous System - confusion, feeling of impending doom, altered mental
status
Treatment
Prevent further contact with allergen
Ensure airway patency
Oxygen alone via a nasal catheter or along with nebulized salbutamol/albuterol
Administer adrenaline IM into the thigh 0.3 mL 1:1000 solution
Repeat at 5, 10 minute intervals if response is inadequate
In severe cases, 1 mL of 1:100,000 of solution is given intravenously
Administer antihistamines chlorphenamine 10 mg IM or slow IV injection
Administer corticosteroids: Hydrocortisone 200 mg IV stat
Clinical features and treatment of anaphylaxis (x2)
Clinical features
Cutaneous - Itching, Hives, Angioedema, Flushing
Respiratory - Bronchospasm, dyspnea and wheeze, laryngeal edema, pulmonary
edema
Gastrointestinal - Nausea, vomiting, abdominal cramps, diarrhea
Cardiovascular - tachycardia, hypotension, arrhythmias, shock and collapse
Central Nervous System - confusion, feeling of impending doom, altered mental
status
Treatment
Prevent further contact with allergen
Ensure airway patency
Oxygen alone via a nasal catheter or along with nebulized salbutamol/albuterol
Administer adrenaline IM into the thigh 0.3 mL 1:1000 solution
Repeat at 5, 10 minute intervals if response is inadequate
In severe cases, 1 mL of 1:100,000 of solution is given intravenously
Administer antihistamines chlorphenamine 10 mg IM or slow IV injection
Administer corticosteroids: Hydrocortisone 200 mg IV stat
OTHERS
Graft versus host reaction
Duchenne Muscular Dystrophy
Caused by a mutation of the gene responsible for producing dystrophin
Duchenne dystrophy is present at birth, but the disorder usually becomes apparent
between ages 3 and 5, by age 12, most patients are wheelchair dependent.
Muscle selectively involved in order of appearance of weakness is iliopsoas,
quadriceps, Gluteus-Pretibial muscles, muscles of pectoral girdle, muscles of leg and
forearm
Clinical features
Gower’s maneuver while getting up from the floor
toe walking due to foot drop
lordotic posture
Pseudohypertrophy of the calves
Cardiomyopathy
Mental impairment
Investigations
CPK elevated to 20-100 times normal
Muscle biopsy
EMG
Examples and use of therapeutic monoclonal antibodies
Misc. Topics
GENETICS
What is X-linked recessive Disease? Mention TWO examples
Location of mutant gene: X-chromosome; no male to male transmission
Required number of defective genes:
Males - only one copy of mutant gene
Females - two copies required
Sex affected - Males are more frequently affected
Pattern of inheritance - Transmission is through female carriers.
Females can be affected if affected father and carrier mother

X linked genetic disorders


Males more commonly affected than females
Affected males will have only carrier daughters
Female carriers are generally unaffected, or if affected, they have milder symptoms

Write three characteristics of autosomal recessive inheritance. Give two examples of


diseases following autosomal recessive inheritance pattern.
Location of mutant gene → Autosomes
Required number of defective genes → Two copies
Sex affected → Both males and females equally
Consanguineous marriage is a common predisposing factor
Examples

Autosomal dominant disorder


Draw a pedigree chart for boy with hemophilia, representing his parents and a sister
// Pedigree chart of hemophilia
Genetic counselling
GERIATRICS
Risk factors for falls in elderly (x2) .// Frequent falls in the elderly // Mention causes
of frequent falls in an elderly person,
Falls are a common cause of morbidity and mortality among the elderly
Falls increase in frequency with advancing age due to multiple age related changes
which can produce alteration of gait and decreased balance. These changes include :
environmental hazards - poor lighting
decreased strength from loss of muscle mass
decreased visual and hearing acuity
decreased proprioception
slowed reaction time
Pathologies associated with falls
conditions like parkinsonism
ophthalmologic diseases
arthritis
foot problems
neurological illness
stroke
peripheral neuropathy
dizziness and disequilibrium
Evaluation of falls
Thorough medical history
Physical examination should include neurological examination that tests gait,
balance, reflexes, sensory impairment, and extremity strength.
Because falls may be associated with acute illness, patients should be assessed for
infections, myocardial infarction and gastrointestinal tract hemorrhage
Orthostatic hypotension although common among elderly, also may indicated
medication effect or hypovolemia from hemorrhage or dehydration
Prevention and treatment
Reduction in environmental hazards — provide adequate lighting, remove obstacles
from floors, eliminate slippery floors, use appropriate footwear
Assistive devices (walking stick, etc.) — improve gait and balance
Review of the medications — avoid drug-drug interactions and eliminate potentially
offending drugs
Treatment of medical problems that may contribute to falls (cataracts, postural
hypotension, postprandial hypotension, Parkinson’s disease)
Management of postural hypotension in elderly
Discuss specific nutritional advice for elderly
Emphasize nutrient-dense foods: Include plenty of fruits, vegetables, whole grains,
lean protein sources and low-fat dairy products.
Hydration: Ensure elderly adults stay hydrated, drinking enough water and other
beverages to prevent dehydration.
Vitamin and mineral supplementation: Consider supplementing with vitamin D,
calcium, and B12 as deficiencies are common in the elderly population.
Sodium restriction: Limit high-sodium foods and include more potassium-rich foods
such as bananas, potatoes, and leafy greens to help regulate blood pressure.
Fiber-rich foods: Encourage a diet high in fiber to support digestive health and
prevent constipation.
Healthy fats: Incorporate healthy fats such as olive oil, nuts, and avocados to support
brain function and overall health.
Avoid processed and high-fat foods: Limit highly processed foods, added sugars, and
saturated and trans fats.
Regular physical activity: Encourage regular physical activity to support overall health,
physical function, and prevent chronic conditions.
Address loss of appetite and taste changes: Offer small, frequent meals, and provide
a variety of flavors and textures to stimulate appetite.
Monitor weight: Regular monitoring of weight and body composition can help
identify changes in nutrition status and provide early intervention if needed.
Clinical assessment of elderly people.

Psychiatry
Write a note on common Psychological problems in elderly
Depression: The elderly are at increased risk for depression, especially after the loss
of a spouse or friends.
Anxiety: Anxiety disorders are common in older adults and can range from general
anxiety to specific phobias.
Dementia and Alzheimer's disease: Dementia, including Alzheimer's disease, is a
progressive decline in cognitive function that can affect memory, thinking, and
behavior.
Delirium: Delirium is a sudden onset of confusion that can result from a physical
illness, medication, or other underlying medical conditions.
Sleep disorders: Sleep problems such as insomnia, sleep apnea, and restless leg
syndrome are common among older adults.
Substance abuse: Substance abuse and addiction can affect older adults, particularly
those with a history of alcohol or drug abuse.
Isolation and loneliness: Isolation and loneliness can lead to depression, decreased
physical and cognitive functioning, and a decline in overall health.
Adjustment disorders: Adjustment disorders can occur in response to significant life
changes, such as retirement or moving to a care facility.
Write a note on emerging role of Information technology in Medicine
Information technology (IT) has been transforming healthcare in recent years, playing
a crucial role in improving the quality of patient care, reducing medical errors, and
increasing efficiency. Some of the key areas where IT is making a significant impact in
medicine include:
Electronic Health Records (EHRs): EHRs provide a centralized, digital repository of
patient information, allowing for improved patient data management and
accessibility.
Telemedicine: Telemedicine enables remote consultations between patients and
healthcare providers, improving access to care for those in remote or underserved
areas.
Artificial Intelligence (AI): AI is being used to analyze large amounts of medical data
to support decision-making and improve patient outcomes.
Wearable technology: Wearable devices such as smartwatches and fitness trackers
can provide real-time health monitoring and data, enabling early detection of health
issues.
Robotic Surgery: Robotic surgical systems are being used to perform minimally
invasive procedures with greater precision and accuracy, leading to improved patient
outcomes.
Clinical Decision Support Systems (CDSSs): CDSSs use algorithms and data analytics
to provide real-time support for clinical decision-making, reducing medical errors
and improving patient outcomes.
Acute psychosis
Name FOUR first rank symptoms of acute schizophrenia and FOUR negative
symptoms of chronic schizophrenia
First rank symptoms

Negative symptoms
Alogia - lack of words including poverty of speech
Anhedonia
Affective flattening - decreased expression of emotion
Attention impairment
Avolition-apathy
Schizophrenia (x2)
Schizophrenia is a heterogeneous group of disorders characterized by perturbations
of language, perception, thinking, social activity, affect, and volition. It has no
pathognomonic features
Clinical features

Investigations
Full neurological examination - Gait and motor
Mini mental state examination
Blood- CBC, LFT, RFT, TFT
Urine drug screen
EEG is suspicion of temporal lobe epilepsy
Brain imaging and findings
Management
Hospital admission in case of first episode to permit a full physical or psychiatric
assessment
Drug treatment
Typical antipsychotics - Chlorpromazine, fluphenazine, thioridazine, haloperidol
Atypical antipsychotics - Aripiprazole, Olanzapine, Risperidone, Ziprasidone
Mechanism of action → Blockade of D2 receptors
Aripiprazole is a partial D2 receptor agonist
Psychological treatment
Psychoeducation
Cognitive behavioral therapy
Social skill training
Family therapy to reduce expressed emotion
Write a note on Bipolar affective disorder
Bipolar disorder is characterized by episodes of elevated mood interspersed with
episodes of depression (both manic and depressive).
Elevated mood when mild or short-lived is known as hypomania or when severe or
chronic is called mania
Types
Bipolar I → Characterized by occurrence of
one or more manic or mixed episodes
one or more major depressive episode
Bipolar II → Characterized by occurrence of
one of more major depressive episodes
one hypomanic episode
no manic or mixed episode
Cyclothymic disorder
Numerous episodes of hypomania and depression
Duration of at least two years in adults and one year in adolescents and children
Symptom free duration is not more than 2 months at a time
No major depressive, manic or mixed episodes during initial two years. After the
initial two years they may occur.
Medication/substance induced bipolar disorder
Bipolar disorder due to another medical condition
Treatment
Antidepressants
Tri-cyclic antidepressants
Types
NA + 5 HT reuptake inhibitors → Imipramine, amitriptyline, clomipramine, doxepin
Predominantly NA uptake inhibitor → Desipramine, nortriptyline, amoxapine,
roboxetine
Side effects
Anticholinergic — dry mouth, bad taste, constipation, epigastric fullness, urinary
retention, blurred vision, palpitation
Sedation, mental confusion, weakness
Increased appetite and weight, fine tremors, postural hypotension
Selective serotonin reuptake inhibitors (SSRIs)
Drugs → Citalopram, escitalopram, fluoxetine, paroxetine, sertraline
Side effects
Gastric upset, nausea, anorexia
Interference with ejaculation
Nervousness, restlessness, insomnia
Reversible inhibitor of MAO-A (RIMAs)
Drugs → Moclobemide, clorgyline (Isocarboxazid, phenelzine, tranylcypromine)
Side effects
Increased appetite (phenelzine)
Decreased appetite (tranylcypromine)
Hepatotoxicity, SLE, Cheese reaction
Atypical antiderpessants
Drugs → mirtazapine, venlafaxine, duloxetine, bupropion, trazodone
Side effects
Priapism (trazodone)
Bone marrow suppression
Hepatotoxicity
Clinical features of depression
Major depressive disorder : Five or more of the below symptoms to be present for at
least two weeks and one symptom must be depressed mood or loss of interest or
pleasure.
Mood : Depressed mood most of the day, nearly everyday (dysphoria)
Interest : Marked decrease in interest and pleasure in most activities (anhedonia)
Sleep : Insomnia or hypersomnia
Psychomotor : Psychomotor agitation or retardation
Appetite : Increased or decreased leading to weight gain or loss
Guilt : Feeling or worthlessness or inappropriate guilt
Energy : Fatigue or low energy nearly everyday
Suicidality : Recurrent thoughts of death, suicidal ideation, suicidal plan, suicide
attempt
Concentration : Decreased concentration or increased indecisiveness
Obsessive compulsive disorder // Obsessive compulsive disorder- clinical features
and management (x2)
Obsessive-compulsive disorder (OCD) is characterized by obsessive, recurrent,
unwanted thoughts (which cause marked anxiety) and compulsions (which serve to
neutralize or relieve anxiety).
Obsessions : These are persistent thoughts, ideas, impulses or images that seem to
invade a person’s consciousness
Compulsions : Repetitive behaviors (e.g. hand washing, ordering, checking) or metal
acts (e.g. praying, counting, repeating words silently) that the individual feels driven
to perform in response to an obsession
Symptoms
Cleaning : fears of contamination
Symmetry : arranging items symmetrically and repeating, ordering and counting
compulsions
Forbidden or taboo thoughts : Aggressive, sexual and religious obsessions
Harm : e.g. thoughts about harm on oneself or others
Treatment
Drugs
SSRIs → paroxetine, citalopram or escitalopram
SNRIs → clomipramine
Cognitive behavioral therapy (CBT)
Causes of Dementia.

Differential diagnosis and investigation in dementia // Investigations in a patient with


dementia

Investigations
Neuroimaging studies (CT and MRI) :
Alzheimer’s Disease : Normal in early disease. In late stages, diffuse cortical atrophy
becomes apparent, and detailed MRI shows atrophy of hippocampus.
Vascular dementia : MRI may reveal hypointensities and focal atrophy suggestive of
old infarcts
PET-scan : Test of choice. It shows hypoperfusion in bilateral parietotemporal cortex
CSF markers : Raised tau proteins, low B42 amyloid and elevated ceramide levels
Routine investigations : Blood chemistry, CBC, tests for syphilis, serum B12 levels and
thyroid functions
Panic attacks
Occurrence of recurrent unexpected attacks of intense anxiety
May occur with or without agoraphobia (avoidance of situations where a person may
feel trapped and unable to escape)
Period of intense fear or discomfort which develops abruptly and reaches a peak
within 10 minutes
In between the attacks the patient is free from anxiety
Age of onset lies between late adolescence and mid 30s
Patients have persistent concern of having attack and they worried about the
implications of attack.
Clinical features (PANICS)
Palpitations, paresthesia or depersonalization
Abdominal distress
Nausea
Intense fear of dying or losing control
Chest pain, choking or chills
Shortness of breath, sweating or shaking
Treatment :
First line : SSRIs + CBT
Acute attacks : Benzodiazepines
Anxiety Neurosis
Anxiety disorders, as the term suggests, are characterized by unrealistic, irrational
fear or anxiety of disabling intensity, worrisome thoughts, avoidance behavior and
the somatic symptoms of autonomic arousal.
Types
Panic disorder
Occurrence of recurrent unexpected attacks of intense anxiety
Attacks begin abruptly and peak within 10 minutes
Clinical features (PANICS)
Palpitations, paresthesia or depersonalization
Abdominal distress
Nausea
Intense fear of dying or losing control
Chest pain, choking or chills
Shortness of breath, sweating or shaking
Phobic anxiety disorder
Defined as anxiety triggered by specific things, situation or environments for > 6
months duration
Treatment : Systematic desensitization therapy
Generalized anxiety disorder
Defined as anxiety unrelated to a specific person, situation or event for > 6 months
duration
Diagnosis (three of the following six) - Worry WARTS
Wound up (irritability)
Worn out (fatigue)
Absentminded (difficulty concentrating)
Restlessness
Tension in muscles
Sleep disturbance
Social anxiety disorder
Characterized by exaggerated fear of embarrassment in public situations
Treatment : SSRIs or venlafaxine + CBT
Separation anxiety disorder
Childhood disorder characterized by overwhelming fear of separation from home or
an attachment figure lasting > 4 weeks
Post-traumatic stress disorder
Clinical features (DREAMS)
Disinterest in usual activities,
Re-experience (intrusive) of event, Reckless behavior
Event preceding symptoms
Avoidance of associated stimuli, Angry outbursts
Month or more of symptoms
Sympathetic arousal (Hypervigilance), Sleep disturbance
Obsessive compulsive disorder
<Already answered>
Delirium tremens.
It is an acute organic disorder characterized by confusion, restlessness, incoherence,
inattention, anxiety or hallucinations which may be reversible with treatment.
Causes

Clinical features
Sudden onset, short episode
Global impairment of cognitive functions (memory, attention, orientation, thinking
etc.)
Variability in state of arousal
Reduced responsiveness is interspersed with periods of excited outbursts
Sleep/wake cycle disturbed
Impaired perception: Misperceives surrounding and attendants, hallucinations.
Disturbance of emotion: Agitation, fear, depression, anxiety.
Psychomotor changes: Hyperactivity, restlessness, repetitive (plucking, tossing).
Investigations
CBC
Urine analysis
Blood glucose, urea
Serum electrolytes
LFTs
RFTs
TFT
Arterial blood gas
CXR
ECG
VDRL, HIV testing
Cranial CT or MRI scan
Treatment
Identify the cause and treat accordingly
Drugs
Delirium tremens :
Diazepam (10-20 mg QID)
Other causes :
Chlorpromazine (50-100 mg TID)
Haloperidol (5-10 mg TID)
Supportive medical and nursing care
Classify antipsychotic drugs
Extras
ALZHEIMER’S DISEASE
Most common cause of dementia in the world
Etiology
Genetic factors
Two groups
Early onset disease with AD inheritance
Late onset disease with polygenic inheritance
Mutations
Point mutation in amyloid precursor protein
Mutations in gene presenilin-1 (PS1) and presenilin-2 (PS2)
Inheritance of one of the apolipoprotein ε4
Environmental risk factors
Age
Female gender
Head trauma and vascular risk factors
Some studies have shown long term consumption of NSAIDs to be protective
Clinical features
Memory impairment
Language problem (aphasia) : anomia commonly
Apraxia : impaired ability to carry out skilled, complex, organized motor activities
Agnosia : failure to recognize objects
Frontal executive function impaired : organizing, planning and sequencing
Parietal presentation : Visuospatial difficulties and difficulty with orientation in space
Myoclonic jerks
Generalized seizures
Diagnostic criteria
Memory impairment
Any one or more of the following
Aphasia
Agnosia
Apraxia
Disturbance in executive function
Investigations
Neuroimaging studies (CT and MRI) : Normal in early disease. In late stages, diffuse
cortical atrophy becomes apparent, and detailed MRI shows atrophy of
hippocampus.
PET-scan : Test of choice. It shows hypoperfusion in bilateral parietotemporal cortex
CSF markers : Raised tau proteins, low B42 amyloid and elevated ceramide levels
Treatment
Supportive
Cognitive rehabilitation
Exercise programs
Occupational therapy
Drugs
Cholinesterase inhibitors : Donepezil, galantamine, rivastigmine and tacrine
Memantine : Blocks overexcited NMDA channels
Statins : protective effect against vascular dementia
Antioxidants : Vitamin E, selegiline
Ginkgo biloba

You might also like